Jump to content

Wikipedia:Reference desk/Science

From Wikipedia, the free encyclopedia

This is an old revision of this page, as edited by 128.104.112.117 (talk) at 23:21, 12 May 2009 (→‎electrons and molecules). The present address (URL) is a permanent link to this revision, which may differ significantly from the current revision.

Welcome to the science section
of the Wikipedia reference desk.
Select a section:
Want a faster answer?

Main page: Help searching Wikipedia

   

How can I get my question answered?

  • Select the section of the desk that best fits the general topic of your question (see the navigation column to the right).
  • Post your question to only one section, providing a short header that gives the topic of your question.
  • Type '~~~~' (that is, four tilde characters) at the end – this signs and dates your contribution so we know who wrote what and when.
  • Don't post personal contact information – it will be removed. Any answers will be provided here.
  • Please be as specific as possible, and include all relevant context – the usefulness of answers may depend on the context.
  • Note:
    • We don't answer (and may remove) questions that require medical diagnosis or legal advice.
    • We don't answer requests for opinions, predictions or debate.
    • We don't do your homework for you, though we'll help you past the stuck point.
    • We don't conduct original research or provide a free source of ideas, but we'll help you find information you need.



How do I answer a question?

Main page: Wikipedia:Reference desk/Guidelines

  • The best answers address the question directly, and back up facts with wikilinks and links to sources. Do not edit others' comments and do not give any medical or legal advice.
See also:



May 6

Wouldn't adding any potential energy to a system make something infinitely massive?

Since lifting an object increases it's energy, wouldn't it therefore increase it's mass? And wouldn't this in turn increase it's energy even further, making a cycle? Since energy is rest mass + kinetic+other forms of energy etc. Also: Should I be using it's or its in my first sentence? That one has always bugged me, since I am showing ownership of the property, but I'm not sure if ownership transfers over to pronouns. If they don't then the un-contracted form being it is, would not make sense in that sentence. —Preceding unsigned comment added by 141.218.35.51 (talk) 00:47, 6 May 2009 (UTC)[reply]

You mean its. "It's" means "it is", "its" means "belonging to it". "Its" is the equivalent of "his" and "hers" - you don't write "hi's" or "her's", so you don't write "it's". That's how I remember it, so it might work for you. Yes, lifting an object does increase its mass, but that doesn't increase its energy, it just increases the amount of energy you need to add to the system to lift it. --Tango (talk) 01:08, 6 May 2009 (UTC)[reply]

Thanks for the quick response to my grammatical question, I knew it felt wrong. "when energy of any kind is added to a resting body, or to a system of bodies, the increase in the mass as seen by a single observer (or as seen from any given inertial frame) is equal to the energy added, divided by c2." This was taken from the energy-rest mass equivalence article here. Wouldn't this cause the lifted object to experience increased mass? —Preceding unsigned comment added by 141.218.35.51 (talk) 01:19, 6 May 2009 (UTC)[reply]

Under ordinary conditions (Earth-like gravity), if there were an increase in mass (I'm not sure there is), the increase in mass would be negligibly small. The extra energy to raise that would be near-infinitesimally small. The extra energy to raise that would be ludicrously small, etc. (See Geometric series.) Now if that mass started at the event horizon of a black hole, there might be more of a problem, but I'll leave that to the next responder. -- Tcncv (talk) 02:43, 6 May 2009 (UTC)[reply]
Lifting an object increases the energy of the system of object and Earth. The energy is neither located in the lifted object nor in Earth, it is contained in the Gravitational field. It is in principle possible to calculate the precise distribution of energy in space by solving the Einstein field equations of general relativity for the two bodies and inspecting the resulting vector field of gravitation (way beyond my abilities). Afaik, gravitation is self coupled, so the infinite looping self attraction effect you describe should actually exist in the field and be part of the solution automatically. Nevertheless, energy conservation applies normally, the kinetic energy of lift is converted to gravitaitonal energy of precisely the same amount. The center of mass of the energy also remains wherever it was before due to momentum conservation. —Preceding unsigned comment added by 84.187.80.238 (talk) 03:16, 6 May 2009 (UTC)[reply]
I just looked it up a bit further. Quoting from Stress-energy tensor: "However, in general relativity there is not a unique way to define densities of gravitational field energy and field momentum." So, it's not that simple to locate. —Preceding unsigned comment added by 84.187.80.238 (talk) 03:35, 6 May 2009 (UTC)[reply]

So I don't really understand where the mass is contained, if it is the gravitational field, what is the form in which it takes? 141.218.35.51 (talk) 03:45, 6 May 2009 (UTC)[reply]

I would guess virtual gravitons. But quantum gravity is not well worked out even at a theoretical level. --Trovatore (talk) 04:25, 6 May 2009 (UTC)[reply]

With regard to the infinite cycle problem, you have to be careful about how you are changing the potential energy of the object in your thought experiment. If I am holding a book on earth, and you define the system to be me+the book+earth, there is no energy being added to the system when I raise the book higher off the ground, I'm simply converting the electromagnetic potential energy stored in my muscles into another form of energy. So in that example, the relativistic mass does not change. Adding energy to the system would mean transferring it from an outside source, for example hitting the planet with light from the sun, or redefining the system to only encompass the Earth + the book. Truthforitsownsake (talk) 12:47, 6 May 2009 (UTC)[reply]

Gravity well thought experiment

To expand on the prior topic: Suppose we start with two masses in space which are gravitationally attracted to each other. As they approach they each gain kinetic energy and loose potential energy (potential energy becoming negative). When they collide, they merge and the potential energy becomes heat which eventually radiates away. With that loss of energy is a loss of mass as observed from outside the system. Now, suppose I were to descend into their gravitational well and measure their mass from there. Would my measurements equal the sum of the original two masses? To put it another way, would my observations from within the gravity-well be different than those at a distance? I know general relativity predicts gravitational time dilation, but what else changes? (..in layman's terms please, sans any stress-energy tensors and the like.) -- Tcncv (talk) 04:21, 6 May 2009 (UTC)[reply]

Yes. Measuring the mass (by measuring gravitational attraction) of the objects near them gives their individual rest mass. Measuring the combined mass of the system from far away returns the sum of masses of the objects minus binding energy. This is actually a good definition of a bound system: If the combined mass of the system is smaller than the sum of mass of the parts, it is bound. The mass loss from the collision will not be detectable before the radiated energy passes beyond the observers position. If you would keep it near the masses, by using mirrors for example, the mass of the whole thing would never change at all, because even as radiation the energy still attracts the observer, just like before. —Preceding unsigned comment added by 192.109.111.157 (talk) 18:16, 6 May 2009 (UTC)[reply]
Thank you. -- Tcncv (talk) 03:49, 7 May 2009 (UTC)[reply]
Confusion alert. What is true is that the sum of the two masses infinitely far away equals the sum of the masses after colliding PLUS the E=mc^2 mass equivalent of the heat radiation from the collision (as 192 said, the binding energy) (including any gravitational radiation, which may be zero in this case, I'm not sure (I seem to remember that a gravitational dipole doesn't radiate ((This whould be a new question)))). The confusion is your statement "observations from within the gravity-well be different than those at a distance". The size of the gravity well is infinite, there is no "outside it" (except at the conventional 'infite'distance). —Preceding unsigned comment added by 128.32.146.97 (talk) 03:57, 8 May 2009 (UTC)[reply]

Pls help me wid my hw?

q1-if light bulbs are connected to other bulbs just like in a series circuit such as in Christmas lights,they get dimmer .why is that? q2-if light bulbs are connected to other bulbs just like in a parallel circuit such as in household wiring .they stay bright .why? q3-how long light bulb last ?what happens to light bulb when it gives it's last glimmer I HAVESEARCHED EVERYWHERE google ,wikipedia ,my textbooks but i couldn't find it —Preceding unsigned comment added by Srijan89 (talkcontribs) 07:40, 6 May 2009 (UTC)[reply]

  • Power is voltage times current. If you connect (identical) light bulbs in a series, the voltage drops in equal parts on each bulb. Thus there is less of a potential at each bulb, the current is lower, and hence less light is emitted. Another way of looking at this is to look at the resistance of the chain - it goes up, hence the current goes down.
  • In parallel (and, strictly speaking, connected to a perfect power source with perfectly conducting cables), the same voltage is applied to each bulb, and hence they all glow with the same brightness.
  • It depends on usage and model, but the way they die is usually by breaking the filament, either mechanically or by vaporization.
--Stephan Schulz (talk) 07:52, 6 May 2009 (UTC)[reply]
See also Series and parallel circuits —Preceding unsigned comment added by 82.44.54.169 (talk) 07:54, 6 May 2009 (UTC)[reply]

Noisy flies

Recently, in the last week, there were two occurences of the presence of a noisy fly in my house. The first, was precisely five days ago where the presence of the fly was known to me for sometime. During the morning and afternoon, it appeared that the fly was attracted to food, like other flies. It was extremely noisy, large and slow, making it easy to kill. However, it was most difficult to determine its location, as often it would sit still unless disturbed. At night, I happened to notice the fly, flying around a light source; it seemed quite far away from my position. I walked along (in my house) with no notice of it, for at least 10-15 metres. Then, to my surprise, I found it idle on my neck. I could not see it and therefore could not determine the exact second when it landed there, but having placed my hand on my neck alarmed me. The fly, to this sudden movement, responded by making a loud buzzing noise - extremely loud to my ear considering that it was on my neck. Within an instance, I noticed it on the wall in front of me. It came a surprise to me that a fly could travel such great distance in so little time, when, in my experience, such flies had been easy to kill and recognizable by their lack of agility. I approached the fly, and within an instant, killed it.

The next time that the presence of such a fly was beknownst to me, was merely a few hours ago. It seemed that the fly was attracted to any light source, often flying near the windows through which the light of the sun shone. It was therefore easy for me to trap the fly behind some curtins. Upon doing this, I heard a great buzzing noise - although I was certain that this fly was actually a fly, rather than a bee. The fly seemed to fly at a low altitude, making it somewhat inconspicuous. In the end, however, I killed it, in the same manner as that of the previous incident.

Based on the previous observations (concisely: loud buzzing noise, slow to move but quick in flight, large, inconspicuous, attracted to light - all in comparison to the average fly), what is the exact species of this fly? Based on my understanding of flies, I would predict this fly to be either a deer fly or a horse fly. However, despite being on my neck for sometime, the fly did not bite me. On the other hand, the flies which I have described seem more aggressive compared to other flies. It also seems that the ratio between the presence of these flies in my house, to the presence of the average fly in my house, is about 1:1. Furthermore, rarely is there such a fly during the summer season. Does anyone have any specific ideas regarding the species to which this fly belongs and the exact name of this fly? Thankyou for any remarks, and apologies for the somewhat long and narrative-like question that I am asking. --PST 10:09, 6 May 2009 (UTC)[reply]

Photo? location? SpinningSpark 10:43, 6 May 2009 (UTC)[reply]

Sounds to me like 1 of the Calliphorid flies or "dumb house flies". They would naturally overwinter in the burrows of animals but your walls or basement seem fine to them. As they warm up, like all ectotherms, they become active. They are attracted to light then since it would mean the way out of the animals burrow. As to why they landed on you, well they do lay thier eggs on other bugs and even eartworms but i doubt they thought you were one of thier potential hosts. Likely just seeking warmth. The Buzzing is likely just a predator avoidance thing, annoying yes, but harmless and effective. Most adults are pollen feeders. I would leave a light on near the door then just scoop the little buggers outside in the a.m. to continue thier important work!

I know the above is unsourced but hey, you asked!!! 67.193.179.241 (talk) 15:34, 6 May 2009 (UTC) Rana sylvatica[reply]

Franz Kafka, is that you as the OP? I'd recognize your prose anywhere.
No man, I am not the op. 67.193.179.241 (talk) 12:44, 8 May 2009 (UTC) Rana sylvatica[reply]

Cartridge specifications

Hi. In the 7.62x39mm cartridge (for example) what does the 7.62 stand for? I thought it was the bullet diameter, but apparently it's not. Also, why do they use 2 measurements instead of 1, like 9mm or .45? And what is shown by those specifications (9mm etc)? Thanks in advance —Preceding unsigned comment added by 122.255.2.65 (talk) 11:59, 6 May 2009 (UTC)[reply]

See Caliber tells you everything you need to know. SpinningSpark 12:10, 6 May 2009 (UTC)[reply]
See 7.62x39mm, believe it or not. —Tamfang (talk) 06:49, 10 May 2009 (UTC)[reply]

Can you explain me this?

First sorry for the title that doesnt says anything, but english is not my first language and I hope that you will understand what I am trying to know.The question:Why does a Cell-phone when lying on a table and ringing(vibrating) move? The phone should represent an Inertial frame and should not move as long as no force from outside is acting on it or am I wrong? Please explain. Thank You. DST —Preceding unsigned comment added by DSTiamat (talkcontribs) 12:51, 6 May 2009 (UTC)[reply]

If there weren't any other forces acting on it then you would be right, it would vibrate but its centre of mass would remain stationary. However there are other forces - friction with the table, gravity (particularly if the table isn't perfectly level), maybe even air resistance. --Tango (talk) 13:15, 6 May 2009 (UTC)[reply]
You are assuming that the vibrations are not directional. Every action has an equal and opposite reaction. So, if the internal little weight moves quickly in one direction, the rest of the phone will move slightly in the other. Most of the vibrators spin. So, they are moving in a circle. The phone, therefore, moves in a circle in the opposite direction. Combine that motion with the friction between the phone and the table and you get a seemingly random drift across the table. -- kainaw 13:21, 6 May 2009 (UTC)[reply]
If the little vibrator inside spins in one direction and the phone the other way would this apply even if the phone would be in vacuum(no friction whatever) and whit 0 gravity acting on it?

"So, if the internal little weight moves quickly in one direction, the rest of the phone will move slightly in the other"-Does this not violate the conservation of momentum? I understand that THIS IS conservation of momentum but if u see the Phone as one sytem who suddenly begins moving?Or should it not be seen this way? Explain please —Preceding unsigned comment added by DSTiamat (talkcontribs) 13:51, 6 May 2009 (UTC)[reply]

The phone is a single system with stored energy (the battery). This is no different that wondering why a car moves when you press your foot down on the accelerator. The stored energy (released by combusting gas) causes motion. In the phone, the stored energy in the battery is released to cause motion. -- kainaw 13:55, 6 May 2009 (UTC)[reply]

The comparation is not the best because a car INTERRACTS whit the road through the weels where the phone does not(vacuum etc), i think it has something to do whit the moving of its centre of mass as Tango said, the momentum in case of the car is clearly conserved ( car moves one way, earth other etc) —Preceding unsigned comment added by DSTiamat (talkcontribs) 13:58, 6 May 2009 (UTC)[reply]

The vibrator moves one way and the phone moves the other - that's conservation of momentum. The vibrator will then move back to where it was (that's what "vibrate" means!) and the phone will do likewise. That results in the phone jiggling around on the spot. Its orientation may change, but its position will remain constant. Until you introduce external influences, that is. If you get more friction while the phone is moving in one direction than when it is moving back, for example, you'll end up with the phone moving slightly. If that same movement happens with each vibration, the phone will move across the table. --Tango (talk) 14:05, 6 May 2009 (UTC)[reply]
Yes Tango i understand perfectly what you are explaining it is logical i figured this out myself BUT lets pretend that the vibrator only does one half cicle not moving back (no more vibrator i guess) and the Phone moves a bit one way, now If u see the Phone as an closed system (not knowing whats inside) this would violate the law of conservation of momentum isnt it? Hope that now you are understanding and sorry but the car example has nothing to do whit it. —Preceding unsigned comment added by DSTiamat (talkcontribs) 14:13, 6 May 2009 (UTC)[reply]
I've just fixed your replies, DSTiamat, please use :: before your responses, not spaces -- MacAddct1984 (talk • contribs) 14:35, 6 May 2009 (UTC)[reply]
The phone's battery is powering the vibrator, allowing it to move the phone. If you applied your logic, DSTiamat, nothing would be able to move unless acted upon by an outside force. -- MacAddct1984 (talk • contribs) 14:41, 6 May 2009 (UTC)[reply]
Most vibrators spin, they do not vibrate. So, the vibrator does not "return". It just spins round and round. The phone spins in the opposite direction. The vibration is caused by the offset weight of the vibrator. I feel that the questioner's problem is the assumption that the motion is being created out of nothing. I was pointing out that it isn't created out of nothing. It is energy stored in the battery, converted to motion by the little motor in the vibrator. It is not safe for work to search for pictures of cell phone vibrators - so I did it for you and here is a page with a picture of some.-- kainaw 14:40, 6 May 2009 (UTC)[reply]
If you don't know what is inside the phone, then you can't tell whether momentum is conserved or not. The moving phone only appears to be violating conservation of momentum because you are assuming there are no moving parts inside it, so you are assuming that its centre of mass is moving as its casing moves. If you reverse your reasoning and assume conservation of momentum instead, so you assume that the phone's centre of mass is not moving, then you can infer from the movement of its casing that the phone must contain a vibrating part, moving in the opposite direction to its casing. Gandalf61 (talk) 14:42, 6 May 2009 (UTC)[reply]
Thank you Gandalf really, shed some light, the other 2 battery posts are really off, showing that they didnt grasped my question. —Preceding unsigned comment added by DSTiamat (talkcontribs) 14:44, 6 May 2009 (UTC)[reply]
Sorry about the misunderstanding. IIRC, a vibrator in a phone works is like an unbalanced centrifuge. When a centrifuge spinning at high speeds is unbalanced, it will shake violently. So a vibrator acts on the same principle, just on a smaller scale. -- MacAddct1984 (talk • contribs) 15:00, 6 May 2009 (UTC)[reply]
The confusion here is in the word "move" - which is a little vague. There are three kinds of motion involved here:
  • The cellphone bodily moves across the table. This is only possible because of friction with the table - and only if the friction is more as it moves (say) left to right than right to left. With wood-grain surfaces, that's possible. So as the phone moves one way, it slips easily - when it moves the opposite way, it doesn't overcome 'static' friction - so it wobbles gradually in one direction.
  • The cellphone vibrates because inside there is a tiny rotary electric motor with an offset weight on it. the center of gravity of the phone moves in a small circle RELATIVE TO THE BODY OF THE PHONE. So if your phone was suspended in a zero-g vacuum - it would still appear to vibrate (move in a small circle) because although it's center of gravity wouldn't move at all - the body of the phone would need to move in order to keep the center of gravity stationary.
  • Because the vibrator is a rotating frame of reference - a phone floating in space would spin in the opposite direction to the vibration motor inside. But when the phone is constrained by the desktop, it can't do that.
SteveBaker (talk) 15:12, 6 May 2009 (UTC)[reply]

Thank you Gandalf and SteveBaker really clever guys they understood what I was actually asking.

 TY, DST  —Preceding unsigned comment added by DSTiamat (talkcontribs) 15:59, 6 May 2009 (UTC)[reply] 


The reason the phone doesn't end up where it started if the vibrator moves quickly in one direction and slowly back (slowlyback-QUICK-slowlyback-QUICK) is beecause the slow movements leave it STATIONARY because of friction. It's the same as a jar. if you're in a jar and hit a basketball against the glass again and again and again and again, you could slowly move in that direction. That's because when it gets back to you, you catch it slower, so that there is literally 0 movement of the glass jar (due to friction of the ground) but when you throw it against the glass the quicker movement is able to overcome friction so that you move 0.001 meters for example. Do it enough times and you end up moving in that direction. But what about in space? In space if you are strapped in the center of an object and you keep throwing a basketball at the same wall and catch it on it's rebound back to you, you will NOT budge. When you catch the basketball again strapped to the center, even if you're doing it MORE SLOWLY, there is no FRICTION that makes this "more slow" movement = 0.
In conclusion: it's quite simple: your cell phone moves because friction makes it so that a force that would normally result in slow movement in fact results in 0 movement. So whereas in a vacuum it would vibrate like this: sachay slowly to one side, quickly to the other, slowly to one side, quickly to the other (remaining in the same net location), on the table it does this: sachays quickly to one side, remains still, sachays quickly to one side, remains still, ending up moving in that direction. The "remains still" part is where the vibrator, in space, would be countering the movement it just made, as it moves back so it can make it again... 79.122.21.123 (talk) 19:11, 6 May 2009 (UTC)[reply]

If the motor inside the phone did move faster left-to-right than right-to-left, then I'd agree with you - but the thing that makes it vibrate is a very simple rotary motor - there isn't likely to be much variation in speed when going left-to-right versus right-to-left.
However, I think I have a MUCH better explanation. Thinking a bit more carefully - I think the reason is this:
  • The motor rotates at constant speed - moving an eccentric mass around in a small vertical circle. It can't be a horizontal circle because the motors they use (at least, all the ones I've seen) are long and thin and could not be mounted with their long axis perpendicular to the screen. If it DID use a horizontal circle, the phone would go into a flat spin every time it rang...and that's not what we see.
  • We know that the center of gravity of the phone is trying to remain stationary - Newtons' laws.
  • Now consider the motion of the eccentric mass that's attached to the vibration motor. Let's suppose it's travelling on the downward part of it's counter-clockwise circular motion:
    • As the eccentric mass moves downwards towards the table - the body of the phone has to move upwards in order to keep the center of gravity stationary. Despite gravity, this lifts the body of the phone a tiny distance off the table (or at least reduces the pressure it exerts onto the table).
    • Then - as the mass rotates a further quarter turn (counter-clockwise) - it's moving to right. Because there is little or no force between phone and table - there is little or no friction the phone moves easily to the left.
    • However, gravity starts to pull the phone back towards the table and in the next quarter turn of the motor, the mass goes upwards - pushing the body of the phone yet harder down. Now the phone is pushing hard against the table.
    • In the fourth quarter turn the mass is moving to the left - but the phone is pushed hard against the table - so it doesn't slide so easily to the right.
  • Hence (in this scenario), the phone moves across the table from right to left in little 'hops' - even if the frictional forces are entirely symmetrical and the vibration motor rotates at constant speed.
Of course if the table isn't flat the phone may start to bounce around and hit the table in more complicated ways - with individual corners of the phone hitting or jumping - so this idealised motion may not be what happens in practice.
SteveBaker (talk) 23:13, 6 May 2009 (UTC)[reply]

that's very long steve, could you summarize it in a sentence the way I did with m ine? 94.27.231.41 (talk) 11:17, 7 May 2009 (UTC)[reply]

I don't see any previous post from your present IP address. But no - what I wrote is about as concise as I can make it and still convey my explanation adequately. SteveBaker (talk) 13:02, 7 May 2009 (UTC)[reply]
(sarcasm alert) How about, "The phone hops like a tiny little bunny." Most Some people like things simplified to the point of cuteness at the expense of truthfulness or understanding. -- kainaw 21:36, 7 May 2009 (UTC)[reply]

Spider Question

I've got a huge, hairy black spider in my kitchen with yellow and black triangle markings down its back. Anyone know what it is? Thx in advance --Anonymous07921 (talk) 13:41, 6 May 2009 (UTC)[reply]

Where is your kitchen? It helps to limit the search to spiders that live in your area. -- kainaw 13:43, 6 May 2009 (UTC)[reply]
Is the spider in a web? on the wall? Running around on the floor? How big is huge? 3" across? 6"? more? 65.121.141.34 (talk) 13:49, 6 May 2009 (UTC)[reply]
My kitchen is in Lawrence, Kansas ;). It was trying to climb my trash can (don't ask me why). And its about as big as my palm. --Anonymous07921 (talk) 14:16, 6 May 2009 (UTC)[reply]
Does it look like a garden spider? -- kainaw 14:23, 6 May 2009 (UTC)[reply]
No, it looked poisonous.....it was huge, fat, hairy, and, in addition to the triangle markings, had a yellow and black stripe on its head. --Anonymous07921 (talk) 14:26, 6 May 2009 (UTC)[reply]
Poisonous things get you sick when ingested/absorbed; venomous creatures bite you and inject venom. Sorry, it's a pet peeve of mine. -- MacAddct1984 (talk • contribs) 15:32, 6 May 2009 (UTC)[reply]
Perhaps the OP was considering the culinary properties of the spider. Why assume that his concern was being bitten by a large arachnid after all? 65.121.141.34 (talk) 15:43, 6 May 2009 (UTC)[reply]
Culinary? Lol, yeah, it was in my kitchen :D. I'll get some seasoning and a frying pan and make some roast spider like that guy from Cast Away :D --Anonymous07921 (talk) 16:54, 6 May 2009 (UTC)[reply]
That is a cultural view. In Cambodia, fried tarantulas are rather popular. But for the record, I did not expect that you were seriously contemplating consuming the spider. 65.121.141.34 (talk) 16:56, 6 May 2009 (UTC)[reply]
But still, i have acute arachnaphobia, to the point where I'm repulsed so much by any kind of spider that I don't even want to squash it with a boot......I generally try to avoid any kind of spider, especially ones as bid as my palm....*shudders*....yeah, well, cambodia can keep their fried tarantulas ;) --Anonymous07921 (talk) 16:58, 6 May 2009 (UTC)[reply]
I strongly suggest deep fried spiders to pan fried. Some beer batter and horseradish sauce will go nicely. As for the type, the only yellow and black spiders I've ever seen in Kansas City are the garden spiders (which can get very large and bristly) and jumping spiders (which are very small). The four dangerous spiders in the are are neither large or yellow/black. They are the brown recluse, black widow, hobo, and yellow sac spiders. -- kainaw 17:02, 6 May 2009 (UTC)[reply]
Lol....I've seen several brown recluses running around outside, and there's ocassionally a black widow in my freakin' bathtub *shudders again*, but it probably is a garden spider --Anonymous07921 (talk) 17:06, 6 May 2009 (UTC)[reply]
If it is a garden spider, let it outside. They make huge webs and eat tons of nasty bugs. I try to keep them happy around my house in Charleston because they keep the palmetto bugs in check. I just wish they'd handle the mosquitoes better. Gotta figure out how to invite a few bats over. -- kainaw 17:20, 6 May 2009 (UTC)[reply]
Lol, just make sure they're not vampire bats ;)....ok, spider's back outside --Anonymous07921 (talk) 17:43, 6 May 2009 (UTC)[reply]
Nest box#Bats. --Sean 18:42, 6 May 2009 (UTC)[reply]
My guess is a wolf spider, but without a picture it's hard to say. Looie496 (talk) 18:31, 6 May 2009 (UTC)[reply]
No, it's not a wolf spider.....it was probably a garden spider --Anonymous07921 (talk) 18:52, 6 May 2009 (UTC)[reply]
We learn something every day – thanks, MacAddct! —Tamfang (talk) 06:59, 10 May 2009 (UTC)[reply]

Google "camel spider bite" and click on images....that guy's leg looks pretty messed up. --Anonymous07921 (talk) 18:58, 6 May 2009 (UTC)[reply]

I would suggest taking a picture with a digital camera; all though this portal is brilliant for just about everything i feel the best person to ask would be the old guy in your neighbourhood who has lived there all his life; he may smell a little funny and start droaning on about the war while forcing you to have a game of Dominoes for his wisdom; but he will no doubt be able to tell you what type of spider it is and if it is going to sink its fangs into your flesh!!!!Chromagnum (talk) 05:21, 7 May 2009 (UTC)[reply]

Scientists have discovered many stars that have planets at great distance using things like stars wobbling from planet gravity or a bit of dimming from a planet "eclipsing" the star. At what range from our solar system, would our scientists be able to detect that our own solar system has planets using their current methods? 65.121.141.34 (talk) 13:59, 6 May 2009 (UTC)[reply]

Comment this sounds a little like a homework problem Dougofborg(talk) 14:39, 6 May 2009 (UTC)[reply]
No it doesn't! Not even remotely! SteveBaker (talk) 14:51, 6 May 2009 (UTC)[reply]
Gotta agree with Steve there. This is a bizarre question to pose for homework! Vimescarrot (talk) 14:53, 6 May 2009 (UTC)[reply]
Well, there are several techniques - but the one that uses doppler shift in the star's light as the star is moved slightly by the planetary gravitation ought to work at any range - providing enough light can be obtained from the star to do some sort of spectrographic analysis...and that's more a matter of how long you're prepared to point the telescope at the star than it is to do with distance. Similarly - the methods that rely on the planet eclipsing the star (and the star eclipsing the planet) will also work at almost any range if you're prepared to wait long enough. However, those tricks don't work well for very small planets because the amount of spectral shift is just too small - so planets like earth, mercury, mars and venus (yes, yes, AND pluto) are probably too small for us to detect at any range unless they are very close to the star. Uranus is probably too far from the sun - so even though it's quite large, it's not very detectable. However, Jupiter and (to a lesser degree) Saturn ought to be pretty detectable over extremely large distances if your alien astronomers are patient enough. So far, the most distant exoplanet we've found is 22,000 light years away(!) - so I imagine that we'd be able to at least detect Jupiter and perhaps Saturn at those kinds of distances. However, even as close as a dozen light years, we don't yet have the technology to detect Earth. Detecting something like Haumea (on the Wikipedia front page today!) would be utterly impossible by any known means - until very recently, we couldn't even detect it orbiting our own star when we're 9 light-minutes away! SteveBaker (talk) 14:51, 6 May 2009 (UTC)[reply]
Pluto is no longer a planet, Steve. Sorry to break the news. :( Vimescarrot (talk) 14:56, 6 May 2009 (UTC)[reply]
Pluto is not an IAU planet. It's still a planet. I decline to recognize the authority of some silly organization like that to dictate my usage of language by way of a vote.
By the way, not only is Pluto a planet, but so are Titan and the four largest moons of Jupiter, and I'd probably throw our own Moon in there too. However I'm not so sure about the gas giants. How can something be a planet if there's nowhere to land? --Trovatore (talk) 17:28, 6 May 2009 (UTC)[reply]
Well, the real planets are the Sun, the Moon, Mercury, Venus, Mars, Jupiter, and Saturn. Everything else is some newcomers opinion. --Stephan Schulz (talk) 17:36, 6 May 2009 (UTC)[reply]
The range of detection will depend on the size of the planet (larger ones can be found farther away) and the size of the star (smaller ones have their planets more easily found). Our article on extrasolar planets and our list of extrasolar planets probably provide a good baseline, though. Going off planets within two masses of Jupiter, the farthest known system is MOA-2007-BLG-400L, 20000 light years distant. That's also more or less the limit of finding any planets. On the other hand, said star is only about 1/3 as massive as the sun, and said planet orbits at about the distance of Venus. Quick back-of-the-envelope calculations suggest that Jupiter's influence on the sun would be about 1/100 that of this system, so I'd ballpark about 200 light years for detecting a solar system with a sun-sized star and a Jupiter-sized planet with Jupiter's orbital characteristics. — Lomn 14:55, 6 May 2009 (UTC)[reply]
I expect that would largely depend on just how hard and how long we'd try to find planets around that particular star we call the Sun. OGLE-2006-BLG-109L is a star 4,900 light years away, where two gas giants were discovered similar enough to Jupiter and Saturn. Equendil Talk 14:55, 6 May 2009 (UTC)[reply]
"Current techniques" have a wide range of resolution capabilities. Keck observatory states that the angular resolution ranges "0.04 to 0.4 arcseconds for individual telescopes, depending on target and instruments used" - that's a variation of a factor of 10x using the same facility! As always, you have to define the hypothetical situation very carefully. With current technology, we have the capability to build much much higher resolution optical telescopes (but we just don't have the science research budget to do it). Consequently, your range of answers will vary by orders of magnitude unless you select a specific technique, specific observational platform, etc. Nimur (talk) 15:01, 6 May 2009 (UTC)[reply]

I would also imagine that it is going to depend on where you are relative to the solar plane? If you are looking down on the solar system, a planet would never move in front of the sun, making planets harder to detect and greatly reducing your range then if you are in line with that plane right? 65.121.141.34 (talk) 15:08, 6 May 2009 (UTC)[reply]

Yes, depending on technique. Such a scenario rules out eclipses but should be ideal for observing gravitational wobble. — Lomn 15:36, 6 May 2009 (UTC)[reply]
Not really, as "gravitational wobble" is not usually detected via the parallax (which is very very small), but via spectroscopic shifts. In other words, you want the star to wobble towards you and back again, not to wobble around on a plane perpendicular to your vantage point. --Stephan Schulz (talk) 16:41, 6 May 2009 (UTC)[reply]

Can our hops, or released bodies from the ground cause slip with respect to the Earth rotation?

Hi, Wikipedia

Nowadays I'm trying to figure out how a mass body behaves when changing its position normal to the earth surface and I hope you can help me in this topic and confirm the correct statement with evidences or equations. I assume ideal conditions where no air, or any object might contribute in friction losses. I will be so grateful if someone studies the following equations carefully and confirms whether or not possible.

Estimated 3D view of the Earth rotating with a body moving vertically to represent the body anticipated slip. Tariq A 6-May-2009

One example is an oscillating ball vertically or a repelled magnet that is released from the ground to become free in a vacuum at a specific height. I'm afraid I'm not that confident of the mathematical model to be used because I lack for advanced mechanics. I tried using simple equations with the following assumptions:

1- Initial circular velocity of the floating body must be the same of that derived from angular velocity when the body was still on the ground. This is due to the conservation of energy that was only Kinetic.

2- No friction of any means exist.

3- Gravity variation with height is negligible.

According to the previous assumptions, my equation went this way:

rad/s,

If h is varying with time this will lead to a differential equation or can be approximated by converting h into an average value (hav = 2/3*hmax) and so the previous equation can be used as follows:

And the time required to cause a full rotation slip will be when

where Tr is the Time required to cause 1 full rotation offset (lag).

and so:

In terms of day time, T it will be:

and finally can be simplified to:


or more approximated,

—Preceding unsigned comment added by Email4mobile (talkcontribs) 09:02, 7 May 2009 (UTC)[reply]


Where T is the normal day = 24*3600 second.


--Email4mobile (talk) 19:41, 6 May 2009 (UTC)[reply]

Is coriolis effect what you are looking for? --Tango (talk) 19:44, 6 May 2009 (UTC)[reply]

I don't think it is related to coriolis effect. To make my question more clear; assume you're bouncing repeatedly at the Equator, normal to the Surface and forget about the latitude now. If we try to average your jumps (average potential energy) and returns to the ground; there results a fixed point above the ground where you're initiating your jumps by let's say height H. because your initial speed at the equator was the same rotational speed for the Earth it is supposed to be constant with height and hence, the angular velocity must be less at H which will cause a lag or offset proportional with time. This time may be several tenths of thousands of years (if hav=0.3m then T will be around 58000 years)to complete lagging rotation. This is of course true only if my calculations and analysis was correct. --Email4mobile (talk) 21:18, 6 May 2009 (UTC)[reply]

Are you saying that the jumping will induce a drag on the rotation of the Earth? Or are you referring to an effect akin to a Foucault pendulum? --Jayron32.talk.contribs 01:42, 7 May 2009 (UTC)[reply]

It seems to be independent of coriolis and Foucault pendulum effects because as you can see from the last simplified equation; the lag time (by one rotation) is only a function of the vertical hop (h)! —Preceding unsigned comment added by Email4mobile (talkcontribs) 08:52, 7 May 2009 (UTC)[reply]

Stirring vs. jiggling

This is partly scientific, partly sociological.

I was told many years ago by my chemistry teacher that the quickest and most efficient way of mixing a soluble solid (such as sugar) into a liquid (such as tea or coffee) is to stir it in a roughly circular motion, and that "jiggling" the spoon up and down, or even side to side, in the liquid is less efficient. That always rang true with me, and it seems logical. I always stir, never jiggle. Firstly, I want to know if what I was told is scientifically correct.

If so, what could explain the phenomenon that "jiggling" is something that, in my experience, is done more by men than by women, who prefer to stir. Men are supposed to be the logical/scientific ones, yet women seem to have the right approach here. If it has something to do with men not wanting to be seen doing something in a "feminine" way, that's probably for a different desk, but any comments would be appreciated. -- JackofOz (talk) 21:46, 6 May 2009 (UTC)[reply]

It sounds completely unlikely to me. Certainly, the turbulance caused by the spoon will be sensitively dependent on the shape of the spoon, the size of the container and the precise speed and trajectory of the spoon. There is no reasonable way to calculate that - so this would have had to have been determined experimentally...but it seems highly unlikely that they'd have tried a huge range of spoons, containers, stirring speeds AND techniques...so in all likelyhood this is junk science of some kind. I'm also completely unaware of any sexual bias in stirring techniques...assuming I'm not alone in this unawareness - it's unlikely that men (in general) are specifically stirring in one way or another because they want to be different from women. I think you're WAY over-thinking this. SteveBaker (talk) 22:42, 6 May 2009 (UTC)[reply]
I should point out that the tip given by the chemistry teacher was not about tea, coffee and sugar per se. It was about dissolving some granulated solid chemical (NaCl, probably) into a liquid chemical, in a flask, in a lab. We were being shown how to conduct some experiment. Some students started using a glass stirrer and just shook it around in the liquid, but she said not to do it that way but stir it in a circular motion. I just extended that principle to tea and coffee; but I always did it that way anyway, because my mother taught me to stir, not to shake or jiggle. Mothers are the ones who generally do most of the cooking at home, so they ought to have a few ideas about the best ways of doing these things. My partner, who's a chef, says he was also taught at cooking school to stir rather than jiggle when preparing food. -- JackofOz (talk) 00:15, 7 May 2009 (UTC)[reply]
Chemistry-lab glass stirrers are typically thinnish rods. Moving such a rod semi-randomly (i.e. "jiggling it") in a beaker of liquid would only generate small-scale currents in the liquid, both because the rod is has a small cross-section and because the movements caused by the jiggles would partly cancel each other out. Stirring in a consistent circular direction, on the other hand, would cumulatively build up a larger-scale circular swirl that would catch up more of the solute more quickly.
Stirring with a spoon is somewhat different, because the shape of the spoon creates large scale movements in the liquid much more easily: even jiggles will have significant effects. Nevertheless larger-scale movements (so long as they don't slop the beverage out of the cup!) would still mix things a little faster.
Circular stirring creates a whirlpool effect that tends to concentrate solutes in the centre of the vessel, slowing their dissolution: this can easily be observed with a transparent solvent (such as tea without milk) in a transparent vessel (such as a glass teacup); to counteract this, when stirring beverages circularly I myself tend to reverse the direction of stir a couple of times. An alternative method is to move the spoon back and forth along a diameter of the vessel, which avoids any local concentrations and is, I suspect intuitively, the most efficient method, but generates more uncontrollable movements in the liquid level, risking spillage.
One often stirs beverages in a social situation, where efficiency is not the sole factor: considerations of etiquette also apply. Jiggling one's spoon is more likely to cause a vulgar clatter than gentle circular stirring, and side-to-side movements more likely to slop. These factors might matter more at a family meal on the best tablecloth and less in an all-male works canteen.
Both the chemical and the sociological aspects of this question would be eminently suitable for research. The dissolution rates of standardised amounts of suitable solutes and solvents in identical vessels, varying the style of stirring and nature of stirrer, could be investigated. Equally, a survey of techniques tabulated by sex, approximate age, class, ethnicity and locality (for example) could be carried out in public cafeterias and the like. Perhaps these would make suitable Science Fair Projects. 87.81.230.195 (talk) 02:55, 7 May 2009 (UTC)[reply]
There are other considerations too besides effectiveness. Something like NaCl is going to be soluble enough to dissolve quickly regardless of how you mix it; it probably doesn't matter much if it takes 5 seconds to fully dissolve or 7 seconds. However, swirling is less likely to cause solution to splash out of the container, which will cause an unmeasurable loss of material, and will introduce error into any quantitative measurement. So even if it WERE more effiecient to dissolve by jiggling, it is messier and probably thus is a bad idea regardless. --Jayron32.talk.contribs 03:30, 7 May 2009 (UTC)[reply]
I'm glad you mentioned locality, 87. Another thing I've also noticed is that people living in rural areas tend to "jiggle" their tea/coffee with their spoon more than city people, who tend to stir it. And younger people tend to jiggle, older people tend to stir. Completely OR, but that is my experience. Don't ask me why I notice these things, I just do.
Jayron, this lesson from my 1968 chemistry class is one of the few things I remember about the course, but it's clearly imprinted on my brain for some reason. The reason the lecturer gave was that it would dissolve more quickly by stirring than any other way, but I can see that there would be other good reasons for stirring too. The lecturer was an American woman, if that's at all relevant. -- JackofOz (talk) 04:49, 7 May 2009 (UTC)[reply]

Russian "quarantine" after spaceflight?

An article on Gamasutra about a lawsuit mentions that Richard Garriott was in "quarantine" in Russia after his spaceflight. Quarantine? Really? Is this an accurate description? Do they really quarantine cosmonauts after a flight, and why? Tempshill (talk) 22:02, 6 May 2009 (UTC)[reply]

Hmm maybe not a perfect description, although the NASA used to quarantine astronauts when they came back from the moon (see Lunar Receiving Laboratory). I suspect in this case it was more "observation" than quarantine, but don't really know. TastyCakes (talk) 22:16, 6 May 2009 (UTC)[reply]
See here also. TastyCakes (talk) 22:21, 6 May 2009 (UTC)[reply]
Right - but that stopped in the early 1970s, hence my surprise the Russians may still feel the need to quarantine stuff that comes back from space. Tempshill (talk) 00:05, 7 May 2009 (UTC)[reply]

Birds tapping on windows

My partner and I have a pet cockatiel named George, in a large cage. It's perfectly happy there, and it's outside the house. A few weeks ago, a hawk was attacking the cage and frightening George, so my partner decided to put her (yes, George is a girl, a fact we only discovered when she started laying eggs) into a smaller cage and bring her inside the house. It was near a window where she and other birds outside could see each other. One bird in particular, a magpie, started to tap on the window with its beak. This would happen maybe a dozen times in the space of a few minutes, usually in the morning. I'd eventually get sick of it, shoo the maggie away, and that would be the end of that. Until next morning. We decided the hawk problem was no longer an issue, so we've now returned George to her normal cage outside. But this damn magpie still taps on the window every morning, as if George is still inside the house, when she plainly isn't. When George was inside, once I'd shooed the magpie away, it wouldn't return till the next day. But now, when I shoo it away, it comes back after a few minutes and starts tapping again. This goes on for a few hours now, whereas before it was only a few minutes. I can't swear it's the same magpie every time, but it looks like it might be. What's going on here? Is the magpie remembering the time when George was inside the house, and not connecting the bird that's outside now with the bird that used to be inside - or is it doing something it might have done anyway? I sort of doubt the latter, because we have a lot of birds around the house, but this window-tapping never happened until George was brought inside. -- JackofOz (talk) 22:42, 6 May 2009 (UTC)[reply]

Are you sure it was a magpie and not a raven? Edison (talk) 23:58, 6 May 2009 (UTC)[reply]
Oh, how droll. -- JackofOz (talk) 00:18, 7 May 2009 (UTC)[reply]
Could it be coincidence and nothing to do with George? Maybe it's only seeing its own reflection? Either way maybe you can cure it of the habit by putting white out on the window (the stuff they use on greenhouses)? David D. (Talk) 02:24, 7 May 2009 (UTC)[reply]
It's not uncommon for birds to attack their own reflections in a window. Maybe the first time the bird was trying to attack George, but got confused by its own reflection and keeps returning because it has a "memory" of an "intruder magpie" there. Looie496 (talk) 02:49, 7 May 2009 (UTC)[reply]
I'd say that it's pretty likely that the bird is just going after its own reflection. I had a Common Blackbird doing the same thing to my window on four separate occasions yesterday. Most bird species do not understand the 'reflection = me' concept, though interestingly, the European Magpie apparently does. I assume that we're talking about the Australian Magpie here though? --Kurt Shaped Box (talk) 03:32, 7 May 2009 (UTC)[reply]
Yes. They're clearly less intelligent than their European cousins. -- JackofOz (talk) 04:34, 7 May 2009 (UTC)[reply]
Try hanging a bit of aluminum foil outside at the window. Magpies (check post about a week ago) like shiny things. It may have been after your bird's mirror (If she has one) or other shiny things or reflections. Funnily enough a lot (all?) of them don't like aluminum foil. That may deter them from going after other shiny things it sees in your window. As a plus it doesn't cost more than a bit of string and a couple of minutes to put in place. Another common recommendation, cutting out the Silhouette of a hawk, may not work; because in the battle of evolution quite a few bird species now flock and attack hawks and thus may actually attack your window rather than leave it alone. BTW (OR) We have a stupid woodpecker that for some strange reason loves attacking our gutters. It has actually managed to peck a hole in one place. Sometimes there's no accounting for a bird brain :-) 71.236.24.129 (talk) 17:47, 7 May 2009 (UTC)[reply]


May 7

IQ

What is the lowest recorded IQ?

Well, I guess the IQ of someone in a persistent vegetative state would be zero. There are degrees of mental disability ranging (in terms of IQ) all the way from that to normality. --Tango (talk) 00:35, 7 May 2009 (UTC)[reply]
As measured across long periods of time, comparing one IQ score to another would be pointless. Usually when people talk about IQ score, they are referring to the Wechsler Adult Intelligence Scale which is a scale from 0-200, which is normallized to a median score of 100. It is also further normalized to a normal distribution, which defines the exact percentages of people who should score any given score along the way. What this means is that as more people take the test, the scores will drift over time. A 100 in 1970 would not necessarily mean a 100 today. The absolute number means almost nothing, what the number tells you is how well you score relative to other people who have taken the same test. Also, so many people are administerred such a test that it is almost impossible to track down any such "lowest score ever"; and even if so, there may be privacy concerns with releasing that data. --Jayron32.talk.contribs 01:39, 7 May 2009 (UTC)[reply]
Combine that with people having an 'off-day' when they took the test - people who were severely distracted by some other thing happening in their lives - and people deliberately trying to get a low score (eg to upset their parents) - and who knows what could emerge. This one is not answerable. SteveBaker (talk) 03:09, 7 May 2009 (UTC)[reply]
Not sure about the zero/persistent vegetative state. In theory wasn't IQ originally for children and consisting of the quotient of mental age over biological age? So a newborn baby is more responsive than someone in [persistent vegetative state]]. An IQ of zero ought to mean like a newborn baby, not below this? --BozMo talk 11:07, 7 May 2009 (UTC)[reply]
For children, yes, I believe that is true, but when the system was extended to adults that definition became pretty much meaningless. It is now defined so that the distribution of IQs is normal. --Tango (talk) 13:21, 7 May 2009 (UTC)[reply]
The articles Imbecile and Mental retardation#IQ_below_70 give some more information. 78.145.24.191 (talk) 11:21, 7 May 2009 (UTC)[reply]
Using the same statistically based scale, different IQ tests are created for optimum resolution of selected ranges of the scale. Thus a common IQ test may not give meaningful results at very low IQ.
In defining the IQ scale the assumption of a normal distribution does not rule out the theoretical possibility of individuals with negative IQ. Cuddlyable3 (talk) 19:39, 7 May 2009 (UTC)[reply]
True, but assuming a standard deviation of 15, we would expect the world population to include only 13 people with an IQ less than 10. Since the number of people in a persistent vegetative state is in the 10,000s or more, and I struggle to see how you could get a lower IQ, that makes negative IQs pretty much non-existent. Human intelligence clearly isn't normally distributed that low down, there is a minimum possible which plenty of people attain. --Tango (talk) 19:52, 7 May 2009 (UTC)[reply]
The IQ scale was never meant to include people in a persistant vegetative state, nor would it apply to a newborn baby. You have to be able to actually complete the test procedure (which involves interacting with the examiner and the test materials) to receive a score. There are certainly ways to estimate a "developmental quotient" (see Bayley Scales of Infant Development) based on the motor and verbal skills you would expect an infant or toddler to have at a given age, but the predictive value -- i.e. what that individual's IQ will be as an adult -- is not perfect. It gets much more accurate as the child grows older and is able to demonstrate different abilities. Also, I would suspect that since the standard IQ test was designed to be centered around 100, it is much better at discriminating between 95 and 105 than at either of the extremes, say between 20 and 30 or between 180 and 190. So, it's probably more appropriate to calculate how many people would fall below an IQ of 40 (4 standard deviations) and to leave it at that. We're talking about mental retardation with little or no verbal ability. There's realistically going to be very little difference between people with IQs of 25 and 35 and there is even less ability to discriminate between 10 and 15, so how could you really have a meaningful "lowest score"? --- Medical geneticist (talk) 20:54, 7 May 2009 (UTC)[reply]
You wouldn't use a standard IQ test to measure the IQ of someone more than a standard deviation or two from the mean. You need a specialist test that has been calibrated for the range you expect the result to be in. I don't see any reason why you couldn't come up with an IQ test for testing people with IQ's less than 10. (Of course, it would be just as meaningless as any other IQ test as an actual measure of intelligence.) --Tango (talk) 00:23, 8 May 2009 (UTC)[reply]
Of course you can use a standard IQ test to measure a person's IQ that is more than 2 standard deviations from the mean, the results just get less reliable the further out you get in either direction. We're all saying the same thing. However, it is completely untrue that "you wouldn't use a standard IQ test to measure the IQ of someone more than a standard deviation or two from the mean". Developmental pediatricians and child psychologists use these types of tests ALL THE TIME to judge the relative capacity of children with developmental delay and mental retardation. One of the diagnostic criteria for mental retardation IS an IQ below 70, which in itself means that these tests are routinely used to measure IQs less than 2 standard deviations below the mean. On the flip side, "gifted" children (whose IQs would be >130 on the standard scale) also undergo IQ testing to see if they qualify for special classes, etc. NOTE: I'm not saying anything about the "validity" of an IQ score per se. That's a different argument. The OP asked "What is the lowest recorded IQ" and my reply is still that there is no meaningful answer to the question. --- Medical geneticist (talk) 13:16, 8 May 2009 (UTC)[reply]
Of course you would use an IQ test to measure their IQ, but not a standard one. If you give someone a standard IQ test and it comes out as 150 you don't write down "150" on your form, you give them another IQ test that is calibrated for IQs around 150 and get an accurate answer and write that down (similarly if the initial test said 50). You can't have one test that is valid for all IQs, it would need in infinite number of questions (otherwise you can't correctly score people that get either all the questions wrong or all the questions right). That is one of the reasons you shouldn't put any weight by online IQ tests - I once took such a test and it said my IQ was 169. There is no way that test was accurately calibrated in that range, so that number is pretty meaningless (all it says is that I did better than that test can accurately measure, there is no way to say how much better). --Tango (talk) 13:32, 8 May 2009 (UTC)[reply]
Can you give examples of specific IQ tests that are calibrated differently to capture different ranges of IQ? I'm just curious. For all practical purposes, if the initial IQ test (either Stanford-Binet, WISC, or WAIS) measured someone at 50, that's the number. There's simply no point in using a more specially calibrated IQ test, or at least I don't know of any. Feel free to point me to a specific example if you know of one. With regard to IQ test results >130, again is there really any practical use for a more highly calibrated test, other than simple personal curiosity or a specific scientific research question? Do you have specific examples of standardized IQ tests that are calibrated to discriminate higher IQs? Keeping in mind a non-normal distribution at either end of the intelligence scale, how would such a test even be normalized? (By the way, I don't have any confidence whatsoever in "online IQ tests" for a whole host of reasons -- the least of which is proper calibration. I wouldn't even consider that to be within the category of "standard IQ tests".) --- Medical geneticist (talk) 15:46, 8 May 2009 (UTC)[reply]
No, I don't have any examples (you could try some of the super-high IQ societies, though). The only examples I could give would be self-administered tests and they are generally aimed at average people since the process of getting an accurate IQ for someone significantly far away from the mean is too complex to do for yourself. It should be obvious that such specialist tests are required - how would you determine the IQ of somebody that gets either no questions right or all the questions right? The only way is to give them another test with easier/harder questions on it. While it may be unusual to get such an extreme result, it doesn't need to be that extreme to be unreliable. --Tango (talk) 16:03, 8 May 2009 (UTC)[reply]
OK, this will be my final word on the subject since we just seem to be going around in circles and discussing this in tangents. If someone here with first-hand knowledge of the actual procedure of giving a standardized IQ test (such as a child psychologist) can answer the OP's question that would be great. Self-administered tests do not qualify as standardized IQ tests and don't have any relevance to the question of the "lowest recorded IQ". It isn't at all obvious that "specialist tests are required" in the context of a true professionally-administered standardized IQ test such as the Wechsler scale or Stanford-Binet. My guess is that the reason Tango doesn't know of any specific examples of IQ tests calibrated for different ranges of the IQ scale is that there simply aren't adequately validated tests for such a thing. I could be wrong. People who are off the scale are just simply off the scale. No need to belabor the point any further. --- Medical geneticist (talk) 21:03, 8 May 2009 (UTC)[reply]
Anyone can score a 0 on an IQ test, and I am sure it has been done before. Just do not answer any of the questions when the test is given. Does that mean the person has an IQ of 0? No, but the question was the lowest recorded IQ. 65.121.141.34 (talk) 16:41, 12 May 2009 (UTC)[reply]

Dudaterin

Hi, my grandpa heard about a medicine on the radio called "dudaterin" but i can't find out anything about it. he heard it on a spanish language radio. whats the scientific name? or wheres the article about it? he says its a prostate med. so any help would be awesome as hes curious about the topic. —Preceding unsigned comment added by 71.142.67.171 (talk) 00:51, 7 May 2009 (UTC)[reply]

It's likely a trade name or variety of Dutasteride or a related compound. Your father should contact his personal physician for advice, we cannot offer such advice here at Wikipedia. --Jayron32.talk.contribs 02:06, 7 May 2009 (UTC)[reply]

Race of the Future

In reference to Race of the Future, given the current and expected growth in the rate of interracial relationships and births; by what year will the majority of Americans be interracial? TheFutureAwaits (talk) 01:46, 7 May 2009 (UTC)[reply]

In the context of your question, how do you define interracial? One drop of blood or more? David D. (Talk) 02:07, 7 May 2009 (UTC)[reply]

WHat do exclaim as interracial we all came from west africa; our DNA can be traced back to this period save a few exceptions Aboriginal>> SO we are all interracial>> We have adapted over time due to our enviroments>> with the growth of technology and advancement in transportation the adaptions are now coming back together more readily so your seeing these adaptions combining and what people are calling interracial>> Chromagnum (talk) 04:57, 7 May 2009 (UTC)[reply]

I'm quite fond of saying that all Americans are African Americans. A Quest For Knowledge (talk) 16:42, 7 May 2009 (UTC)[reply]
East Africa. —Tamfang (talk) 07:06, 10 May 2009 (UTC)[reply]

Living fungus?

Is the Penicillium roqueforti fungus in the Danish Blue cheese in my refrigerator still alive when I buy the cheese? Neither article really addresses that question, and I couldn't find the answer in cheese either. -GTBacchus(talk) 02:19, 7 May 2009 (UTC)[reply]

It sure is. But most natural cheeses contain live cultures, and so doesn't yogurt and sourdough bread and lots of other products. Incidentally, its probably a Good Thing that it is still alive. Like other cheese cultures and yogurt cultures and yeast, the live Penicillium molds in blue cheese actually inhibit the growth of other cultures which may impart a less desirable flavor, or worse, may be somewhat bad for you. --Jayron32.talk.contribs 03:26, 7 May 2009 (UTC)[reply]
Oh, I wasn't thinking of it as a bad thing. I'm just cataloging living creatures I find in and around my house, and wondered if that one made the list. Sounds like it does. Maybe I'll buy some sourdough bread next time! -GTBacchus(talk) 03:37, 7 May 2009 (UTC)[reply]
No problem. I just find that some people are so ignorant (not you, of course) of what food is really about that when they learn that they are eating live molds or live bacteria or stuff like that, they freak. Just to expand on it, the mold may be made somewhat dormant at refrigerator temperatures, but there is nothing in the cheesemaking process that "deactivates" or "kills" the mold. Its left in a cave for a few months, taken out, a piece is hacked off and sold to you. Its not much more complex than that. Its funny. In our modern age of heavily processed food, people think its weird to eat something that's entirely natural or even (eek) alive. Seems bass-ackwards to me, but its a common attitude in our modern society. When Kraft Singles are considered "cheese" I guess that's what you get... --Jayron32.talk.contribs 03:51, 7 May 2009 (UTC)[reply]
Surely there are no live cultures in sourdough bread once it's been baked.... - Nunh-huh 04:30, 7 May 2009 (UTC)[reply]
No, the dough does but the cooked bread does not. However when people hear that the bread has a distinctive flavor because the dough was innoculated with a live culture, they do that whole "ick" thing like I describe above. Again, this ignores the fact that even Wonder Bread is made with live yeast, but again, people are so disconnected from their foods, they are repulsed by that which is natural and good for them. Weirdness abounds... --Jayron32.talk.contribs 04:48, 7 May 2009 (UTC)[reply]
My favorite example of yummy bacteria is Lambic beer. I'm amazed that ambient critters can make something so delicious - Nunh-huh 05:00, 7 May 2009 (UTC)[reply]
Hey, as a fan of eating oysters myself, I can state unequivocally that eating a still living critter can be quite tasty, at nearly any size. And I'll second any list of tasty foods that includes beer of any kind. --Jayron32.talk.contribs 05:06, 7 May 2009 (UTC)[reply]
I see the wrens and tits in my backyard eating live mealworms that I put out for them, and that looks pretty satisfying. Bugs just don't appeal to me. :/ Moreover, I can't imagine that salmon tastes any less good to a sea lion in the ocean or a bear in a river than it does to me in a restaurant. I enjoy miso soup, which is made with Aspergillus oryzae, a fungus which I've been told is still alive in the soup. Mmmm, life. -GTBacchus(talk) 19:15, 7 May 2009 (UTC)[reply]

Fly sounds

(Inspired by the fly question above) How exactly does a bluebottle fly or your standard, basic house fly emit that distinctive buzz (I'm guessing that it may just be the noise made by the wings during flight but I'm not 100% on that)? Also, does the buzz, or the tone/volume thereof have any particular communicative purpose? When I was a kid, I used to think that a louder, lower buzz indicated that the fly was angry or afraid (when I was trying to swat one, or catch one in a jam jar for example) but knowing what I know now, I don't suppose that a fly is capable of much, if anything in the way of emotion. They also make a distinctive buzz when you fly spray them. Again, I used to believe that that particular sound was a scream of sorts, which did actually make me sit and think about the ethics of using fly spray (always seemed like a slow agonizing death, perhaps something like being burned alive or sprayed with acid, as opposed to a quick one from swatting) on more than one occasion... ;) --Kurt Shaped Box (talk) 03:53, 7 May 2009 (UTC)[reply]

I don't think that flies have the central nervous system to be angry or afraid. The sound of most insects is generated by their wings. The flies sound of flight is probably mostly caused by wingspeed; if it is trying to get away from a threat, the sound of its wings may change, but not because of any emotion, just because they are flapping faster. --Jayron32.talk.contribs 03:59, 7 May 2009 (UTC)[reply]
Thanks for that. As a matter of interest, are there any real 'brain bugs' on our homeworld (sorry if this is a stupid question - I know very little about insects)? Up until a few months back, when it was explained to me here, I believed that queen bees/ants/wasps/termites/etc. had some form of basic intellect and emotional capability... --Kurt Shaped Box (talk) 04:37, 7 May 2009 (UTC)[reply]
The largest arthropods are things like Lobsters and Crabs, and they've got brains not much more developed than ants, just larger to be able to operate a larger body. Insects have behaviors that we anthropomorphise. Basically, we assign what is essentially a reflexive behavior a more intentional cause, because we think "If I did that, it would be because I was thinking." This is not to say that insect behavior is not complex, its just not intelligent or emotive in the way we assign such behaviors to "higher" animals like vertebrates. A queen bee takes care of her hive in a certain way not because of love, but because doing so makes her offspring survive better, etc. etc. Again, like with the "angry" fly, there is no such thing as a "loving" queen bee. Computers can have complex behavior too, but they aren't "emotive" or "intelligent". --Jayron32.talk.contribs 04:45, 7 May 2009 (UTC)[reply]
The "brainiest" bugs are probably honeybees -- they have some pretty remarkable visual learning capabilities, and the way they communicate by dancing is just astonishing. Cockroaches (ugh) also have some interesting capabilities, namely the ability to learn routes. No insect or arthropod has anything like emotions, though. Anyway, the brainiest invertebrates by far are not arthropods but molluscs -- squids and octopi. They even have some responses that are a little bit like emotional expression. Looie496 (talk) 05:34, 7 May 2009 (UTC)[reply]
You've heard of the movements to get chimps, gorillas and monkeys classified in law as 'people', right? I seem to remember there being some sort of campaign on behalf of octopi too, based on their claimed intelligence. Granted, this may have just been internet whackamole stuff... --Kurt Shaped Box (talk) 05:41, 7 May 2009 (UTC)[reply]
In the mid 1990s the British Government decided to include Octopus vulgaris among the protected species in an amendment to the Animals (Scientific Procedures) Act 1986, based on their well developed central nervous system and ability to do complex tasks. This classifies them as a vertebrate with regards to the type of experiments one can do on them in the UK and the license you need to do it. By the way, the plural of octopus is not octopi because it is not derived from Latin. The technically correct plural should be octopodes, but the generally accepted term is simply octopuses. End of pedantry ;) Rockpocket 01:41, 9 May 2009 (UTC)[reply]
(EC)Complex behavior is not proof of intelligence. I can train a vacuum cleaner to do some pretty complex stuff, but it doesn't make it intelligent. Intelligence is not the same thing as complexity; bees cannot abstract, cockroaches cannot interpret. --Jayron32.talk.contribs 05:46, 7 May 2009 (UTC)[reply]
(Neither can most people...but that's another matter!) I don't know that bees cannot abstract - they use their dances to convey information about the direction of nectar in a pretty abstract manner. The direction the bee moves while waggling it's bottom lies in the vertical plane of the honeycomb yet is interpreted as being relative to magnetic North - and the amount of waggling somehow indicates distance to the nectar. That's really rather abstract. Of course it's probably instinctual rather than learned - so we could argue about whether they are THINKING abstractly or whether that's merely a hard-wired abstraction...but when you look into the details of what they are doing, it's rather compelling. Can cockroaches interpret? I don't know - have you seen an experiment that conclusively proves that they can't? As a species, we're very fond of finding things that make use "unique" amongst the animals - and one by one, these get demolished. When I was a kid, it was always said that mankind has language and uses tools and that makes us unique. Then we find that many species have languages - and there are chimps, birds and even fish that use tools. The statement is then modified from "use tools" to "make tools" - but we find chimps and bonobo's making tools by stripping leaves from twigs - so that's busted too. It used to be said that only mankind fights wars - but again, chimps and even bees and ants do exactly that. Then we heard that only humans are aware of "self" - but now we have experiments showing that dolphins, elephants and chimps are all self-aware too. I think it's extremely rash to come out with statements like "bees cannot do abstract thinking" and "cockroaches cannot interpret" without doing convincing experiments to prove that. SteveBaker (talk) 12:54, 7 May 2009 (UTC)[reply]
Steve, I would think that the burden of proof would be placed on the statement that insects can be abstract and interpret. You can not prove a negative unless you first prove a positive. 65.121.141.34 (talk) 13:24, 7 May 2009 (UTC)[reply]
I think Steve was claiming that we cannot (currently) know whether insects can abstract/interpret. The burden of proof is on whoever claims that insects can or can't abstract/interpret. --Mark PEA (talk) 20:34, 7 May 2009 (UTC)[reply]
Yep - in the absence of any experimental results in either direction - we have to say that we don't know. There is no burden of proof - merely a burden of not overstating things! I'd be prepared to accept that a rock cannot perform mental abstraction - but a bee does have a brain, albeit a fairly simple one - and it's certainly not impossible that it might have abstract thoughts. SteveBaker (talk) 22:28, 7 May 2009 (UTC)[reply]

The Book of General Ignorance, a few years ago, claimed that no-one was entirely certain how bees and bluebottles made their distinctive buzz. They blocked up their airholes and cut off their wings to no avail - although both do add to the buzz, they don't seem to be the cause. On the other hand, QI contradicted this (before or after, I don't know), saying the airholes (spiracles) were the cause. Vimescarrot (talk) 10:34, 7 May 2009 (UTC)[reply]

This is not to say that insect behavior is not complex, its just not intelligent or emotive in the way we assign such behaviors to "higher" animals like vertebrates. A queen bee takes care of her hive in a certain way not because of love, but because doing so makes her offspring survive better, etc. etc.
I don't see why the fact that a behavior is instinctive precludes it from being due to emotions. Humans defend their children, scream when afraid, and smile because these instincts were beneficial to our ancestors and were selected for, but that doesn't mean humans don't love, become afraid, or feel happy. It's true that insects have very rudimentary brains, but scientists don't have the slightest clue about what causes consciousness. It may very well be that insect brains or even rocks feel emotions more strongly than humans. --99.237.234.104 (talk) 03:32, 8 May 2009 (UTC)[reply]
You are missing one important consideration: mechanism. Instinctive behaviours are mediated by hard-wired neural circuits, and if we can map these circuits we can show the path from the input to the output. Consider pheromones as an input. These promote quite complex behaviours in both vertebrates and invertebrates. We know that a single, molecularly defined pheromone is sufficient (on detection) to elicit a fixed action pattern of behaviours. By appropriate experimentation, we can show that the behaviour is not due to emotional or learned responses, but due to the detection of a precise environmental stimulus. We can now map the receptive neuron that detects the pheromone, and identify the secondary neuron that projects to the region of the brain the mediates the behaviour. So we can also show the mechanism through which the signal is transferred. Together these experiments show what drives instinctive behaviour. Of course, that is not to say that emotional responses could not influence or modulate these hardwired circuits (they most certainly do). Rockpocket 01:58, 9 May 2009 (UTC)[reply]

Can someone quickly fill me in on what is known about the nervous systems of invertebrates? The Drosophila melanogaster#Behavioral genetics and neuroscience section has a little information but not a single reference. --Mark PEA (talk) 13:51, 8 May 2009 (UTC)[reply]

This site offers some basics. Its supposed to be for kids, but it gives a nice layperson's overview. Here is something a little more detailed. Rockpocket 01:48, 9 May 2009 (UTC)[reply]

Double Slit Experiment

I have done a lot of research on this subject, but in no article I have read has it mentioned what apparatus is used to observe what is passing through the slits.

I don't need an explanation why this is irrelevant. I would like a description of what was used to observe the phenomenon. —Preceding unsigned comment added by 70.176.226.100 (talk) 05:08, 7 May 2009 (UTC)[reply]

The Double-slit experiment is mostly seen as a Gedankenexperiment to demonstrate that subatomic particles like electrons and photons are not actually particles or waves, but something which can be forced to behave like particles or waves in specific circumstances. However, there are cases where the actual experiment has been done. In our article on the double slit experiment, the section titled "Importance to physics" has some references which describe actual double-slit experiments which have been carried out, and their results. If you dug out the references, you could find the actual equipment. --Jayron32.talk.contribs 05:18, 7 May 2009 (UTC)[reply]
here is a video in Spanish where a guy does the classic Double Slit experiment using a bunch of stuff lying around the house. You could probably do this yourself with stuff lying around the house right now, or at best, with a quick trip to Home Depot. --Jayron32.talk.contribs 05:20, 7 May 2009 (UTC)[reply]
That's only half of the experiment. He's showing that light is diffracted - and hence behaves in a wave-like fashion. Where is the part that shows that if you count the photons then the diffraction pattern goes away? SteveBaker (talk) 12:39, 7 May 2009 (UTC)[reply]
Well, that's why its a gedankenexperiment. "Counting" individual photons is a difficult process. You only have three options to actually detect a particle:
  1. You can block the path and let it strike something... This functionally removes the slit, so that idea is out...
  2. You can detect the effect of the particle as it passes through the slit. Impossible with a neutrally charged photon; possible with a charged particle like an electron, but the electron can only be detected by another charge source, and that source will interact with the detector, which will change the waveform of the electron, making it no longer coherent with the rest of the electrons, which means it can no longer interfere with other electrons.
  3. You can bounce something physical off of the particle as it passes through; for example you could shoot electrons through a beam of photons. This is like hitting bullets with other bullets, and the interaction changes the nature of the electron, see #2 above.
The result is that there is nothing that can be done to detect a particle which does not also change the particle (or, if you prefer, there is nothing that can be done to detect a wave that does not change the wave). The result is predicted by the math and the logic; which is why it is usually expressed as a gedankenexperiment. It has been done, but many years after it was proposed, and only occasionally as a curiosity. The results turned out exactly as expected. --Jayron32.talk.contribs 23:03, 7 May 2009 (UTC)[reply]
You may also want to read the Copenhagen interpretation, which is a particular perspective on quantum mechanics which holds that, among other things, the quantum mechanical world and the classical mechanical world do not interact in ways that make sense in a purely classical explanitory framework. In other words, since the double slit experiment uses devices which themselves obey classical physics, it cannot be used to fully understand the nature of an essentially quantum mechanical phenomenon like light. The results of the experiment, that light is sometimes a particle and sometimes a wave, makes no sense in the classical world, but according to the Copenhagen interpretation, that's fine because light is really neither, and we shouldn't force quantum mechanical phenomena to make sense in the classical world. There are other aspects of the Copenhagen interpretation as well, such as the idea that for any sufficiently large system, the quantum mechanical properties and the classical properties converge. Thus, while the double-slit experiment should give us the same results whether we are dealing with photons or ping-pong balls, its just that ping pong balls are so large that it is impossible to actually build slits of the appropriate size to actually detect the interference pattern of their waveforms. But the Copenhagen interpretation holds that since the math is sound (i.e. you can calculate the wavelength of a ping-pong ball) that the quantum mechanics can be assumed to hold true even if the experiment were impossible to do on a practical scale. What does this mean for the Double-slit experiment? Its just that it isn't often done as an actual experiment because we don't need to; the math behind the expected results is sound, so we take it as a given, like Schroedinger's cat and the EPR Paradox (spooky action at a distance). Which is not to say that its NEVER been done (see above for a few examples). --Jayron32.talk.contribs 05:36, 7 May 2009 (UTC)[reply]

Thank you very much, this is exactly what I needed. Have a nice morning/day/night/evening/afternoon! —Preceding unsigned comment added by 70.176.226.100 (talk) 06:12, 7 May 2009 (UTC)[reply]

Sorry to resurrect the thread, but I think there's a common misconception here that needs clearing up. The simplest quantum version of the double-slit experiment is where you turn down the light source until there's a noticeable interval between photons. The counting happens at the same detector as in the classical case, it doesn't involve putting anything at the slits. For each emitted photon (that makes it to the detector) there's a flash of light somewhere on the detector, and over time many such flashes build up an interference pattern like this. As far as I know all of the "quantum double slit experiments" that have been done in practice are of this variety, with no detectors at the slits. This version is already very difficult to explain classically, and it gets even harder when you realize that it also works with electrons and protons and even large molecules like buckyballs. There's also a version of the experiment where you put a detector at one or both slits and the interference pattern disappears, but in a way it's actually less surprising because it's easy to come up with a classical explanation—there must be some interaction at the slit which causes the wave to lose its coherence. It turns out, amazingly, that this explanation is wrong (or at least insufficient) and there really is something fundamentally nonclassical going on, but you need more complicated setups like the quantum eraser or the bomb-testing experiment to demonstrate that. More complicated experiments have been done, but I don't think the double slit experiment with detectors at the slits has been done. The more complicated experiments don't require in-flight detectors, so the problem of how to make one doesn't arise. -- BenRG (talk) 19:04, 8 May 2009 (UTC)[reply]

Science and believe

Is science also a form of believe? If we don't have a particle accelerator in our basement, we have to believe what people told us, for example.--Mr.K. (talk) 11:43, 7 May 2009 (UTC)[reply]

The difference between a religious belief and a scientific belief is that I can check my scientific "belief" where a religious belief has to be taken "on faith" (which is another way of having the guys in charge of the religion say "what we say is right and that's the end of the debate"). I don't have a particle accelerator in my basement - but I can look at published results from people that do. Those reports detail how I could (in principle) build my own accelerator and reproduce their results. Indeed other people do build accelerators and attempt to reproduce results. Very few scientific hypotheses are widely accepted as fact until the experimental results have been documented, duplicated my another team and put through the wringer of 'peer review'. It's certainly tough in the case of multi-billion dollar machines like the LHC - but that's the process.
It's true that you ultimately have to take these results "on trust" - but you can at least be sure that the results have been verified - that the procedures have been examined minutely - that every step of the experiment is documented in a form that you can read about in your local library. Nothing whatever is hidden or secret or to be taken "on faith".
But let's take a more reasonable case:
AH84001
AH84001
If the Pope says "God came to me last night and told me that there is life on Mars" - then good Catholics are supposed to believe him and take that on faith as an actual FACT - not admitting even the possibility that he might be wrong about it (See: Papal infallibility). On the other hand - if someone finds a chunk of Mars that arrived on earth as a meteorite (Allan Hills 84001 for example) - looks inside and finds little squiggly things that look like fossilised bacteria - he has to publish that result - which means that what he wrote has to be checked by other experts in the field - they look at his photos - check that his procedures are OK. If they agree then "peer review" is over and the report gets published - the world's press go wild with stories of life on Mars. The President of the USA makes an announcement about it...but it's still not accepted as fact within the scientific community. Other people take chunks of the meteorite and look at it in other ways - doing chemical analysis. People work hard to find other ways in which these little squiggly structures could have come about. More peer-reviewed papers appear - some disagree. Experiments are proposed - NASA sends rovers to Mars to check this out. Eventually, we come to understand that this rock - impressive though it is - does not prove that there is life on Mars. The hypothesis does not become theory - we do not "believe" that this represents proof. At no point along this process did anyone who understands the scientific method say that the existance of life on Mars was a valid scientific fact.
There are countless cases where scientific 'authorities' have claimed some truth - and that truth has been disproven through use of "the scientific method". Read about N-rays, Cold fusion and the Piltdown man. You can't find many (if any) cases where people have looked carefully in the Christian Bible and said: "You know - this whole section about Noah and the Ark - it's obviously not true - we should take that out of the next edition." - it's dogma - the best people can do is weakly plead that some of the stories are maybe just there to teach us valuable lessons...and there are plenty of people who believe in the literal truth of the Noah story simply because it's printed there between the covers of that book. That kind of thing would never fly in scientific circles. We'd want to know where the evidence is. Show me the fossilised skeletons of billions of animals and people wiped out in this mass-extinction event. Show me how every human on earth is descended from one family who lived so recently in the past. Without evidence - that's a dead theory.
So yes, we use the same word "belief" for both religious and scientific belief - but they mean very different things: "faith" and "proof".
SteveBaker (talk) 12:31, 7 May 2009 (UTC)[reply]
Parenthetically, it's probably worth stating that the example given is incorrect insofar as papal infallibility is concerned. - Nunh-huh 19:18, 7 May 2009 (UTC)[reply]
Yep, just to reinforce that (since we all strive for accuracy here!). The sentence on papal infallibility is not true. 80.41.71.69 (talk) 21:58, 7 May 2009 (UTC)[reply]
Yes science is a form of belief. It is a belief that every expectation about what is true must be continually falsifiable. It is counted as a success of the scientific method every time it displaces previous assumptions by better-proven theory.
I don't have a particle accelerator in my basement either. If you ask here they will let you see what they have in their cellar. Cuddlyable3 (talk) 13:07, 7 May 2009 (UTC)[reply]
SteveBaker's answer is a nice "best case scenario." In practice, things that fall at the limits of experiment CAN be difficult to independently test, and things CAN go wrong. When only one machine can do a test (much more common in certain branches of physics than other sciences), then you are radically reducing the possibility of independent evaluation and replication. This makes the problem of individual error, confusion, or even misconduct much higher. (See, e.g., the Victor Ninov affair.) But it should be noted that these are the extreme ends of things. Most scientific knowledge is significantly within these extremes, and, crucially, the "extremes" actually change over time (the very first cyclotron was, for its time, the extreme cutting edge; now it is a lab toy). So yeah, there is some "faith" involved, but not quite the same thing as the Pope. As far as "systems of belief" go, none are as flexible and changeable in the face of new information about the world itself than science. That puts it in quite a different category than organized religion, in my opinion. --98.217.14.211 (talk) 14:45, 7 May 2009 (UTC)[reply]
I didn't mean to imply that the process I described was perfectly followed - any system with fallible humans involved is going to have the occasional charlatan and the odd genuinely honest (but just plain wrong) individual. Read the story of the Martian canals for example. One guy believes he sees them - his reputation is strong so other people believe him - and the myth gets perpetuated until someone goes and bolts a spectroscope on to the end of a decent telescope and busts the myth wide open. But the important thing here is that the way things are SUPPOSED to go is fundamentally designed to eliminate things that you have to "just believe" and to leave a paper trail that you can follow (at least in principle) to allow you to try to reproduce an experiment. Even if we don't always succeed in reaching those lofty goals - at least we try. Religion not only doesn't try to fix up it's mistakes - it actively discourages you from doing that by making it a fundamental matter that you "have faith" - hence ridiculous stuff gets perpetuated over thousands of years, essentially unchallenged. SteveBaker (talk) 16:58, 7 May 2009 (UTC)[reply]
I believe the reason you don't need a particle accelerator to reproduce the result is that you know the people who do have one are in the scientific culture. That culture has its charlatans like everywhere else, but it at least tries to be rigorous, it would never (broadly speaking) accept something irrational as fact, and it has a history over the long term of correcting itself. None of those things are true about "faith", which by definition relies on unproveable things, and leans heavily on evidence-free dogma just to keep its story straight. --Sean 15:02, 7 May 2009 (UTC)[reply]
Science is constantly changing. Some scientific beliefs from 1909 would now be considered irrational, and some scientific beliefs from now would have been considered beyond absurd in 1909. Most of science is very good, you just have to watch out for certain areas that are purported as science that have not been reproducible in a lab. I think a couple of prime examples are listed above. 65.121.141.34 (talk) 16:24, 7 May 2009 (UTC)[reply]
I meant deliberately believing something irrational which requires an appeal to magic (talking snakes, etc.). --Sean 16:59, 7 May 2009 (UTC)[reply]
We should apologise to the OP. Mr.K. asked about the nature of scientific knowledge. He did not ask for OT posts about how materialists reassure themselves about faith based belief systems, but that's what he got. Cuddlyable3 (talk) 19:24, 7 May 2009 (UTC)[reply]
No. Our OP asked very specifically about "belief" - NOT about "knowledge" - your confusion about that is a telling one! SteveBaker (talk) 22:24, 7 May 2009 (UTC)[reply]
I inferred from the brief original question that it was of the "science is just another religion" genre which is popular here. If you have a better idea what the OP was looking for than the other respondents, why not offer an answer rather than attack our good-faith responses? --Sean 21:45, 7 May 2009 (UTC)[reply]
Why the heck would we apologise? Firstly, there is no evidence that your interpretation is correct and everyone else's is not - secondly, even if you're correct, if Mr K wants a specific answer then Mr K is responsible for phrasing it in a way that won't confuse a solid 90% of the people reading it. Thirdly, I don't see anything so horrendous going on here that it would require an apology even if we had gotten it wrong. This is the science desk - if you don't want answers from "materialists" you probably shouldn't ask a bunch of hard-core science enthusiasts! SteveBaker (talk) 22:20, 7 May 2009 (UTC)[reply]
Mr K is responsible for phrasing it in a way that won't confuse a solid 90% of the people reading it - That also applies to not introducing completely wrong statements about papal infallibility, Steve. You were 100% wrong about Catholics being required to believe whatever the pope says God told him about life on Mars. There is a vanishingly small number of things that Catholics are "required" to believe; in the vast, vast majority of cases, they have as much free will to believe or not as you or I do, without necessarily any implications for their membership of the church. -- JackofOz (talk) 22:34, 7 May 2009 (UTC)[reply]
Actually - no - I'm not wrong. Well, perhaps you think our rather carefully referenced articles on the subject are wrong - but as far as I can tell, here are the facts:
From Pope: "Over the centuries, popes' claims of spiritual authority have been ever more clearly expressed since the first centuries, culminating in the proclamation of the dogma of papal infallibility for those rare occasions the pope speaks ex cathedra (literally "from the chair (of Peter)") when issuing a solemn definition of faith or morals.".
From Papal infallibility: "Papal infallibility is the dogma in Catholic theology that, by action of the Holy Spirit, the Pope is preserved from even the possibility of error when he solemnly declares or promulgates to the Church a dogmatic teaching on faith or morals as being contained in divine revelation, or at least being intimately connected to divine revelation."
From Dogma: "Dogma is the established belief or doctrine held by a religion, ideology or any kind of organization: it is authoritative and not to be disputed, doubted or diverged from."
From Roman Catholic theology: "The most notable differences..." (from other christian faiths) "...include Catholic beliefs in...the Pope as the "Vicar of Christ on Earth", papal infallibility..."
From Vicar of Christ: "This title of the pope implies his supreme and universal primacy, both of honour and of jurisdiction, over the Catholic Church."
(If you doubt these things - follow the references from the articles - many of them lead you straight to the Vatican's own web site).
Now I'll admit that it's unlikely that the pope would claim that life on mars was to become a central part of one's faith in god...but if he did (and some of those popes get awfully old and senile) - and if you wish to claim to be a Catholic - then you too must admit that. Obviously, you can say that you don't - but in so doing you are most certainly denying your faith (see definition of Roman Catholic theology, above).
The Catholic dogma (see previous definition) is that if the Pope (see previous definition) digs his heels in and says "God told me this was true and you guys have to take it on faith" - then as far as the Catholic church is concerned - it is the literal and undeniable truth and about a billion people are expected to turn on a dime and believe it. You can't even consider the possibility ("not to be disputed, doubted or diverged from...") that it might not be true (see previous definition of papal infallibility). None of that "well, I'll have to think about that one!"...nope - if you are a practicing Catholic - you turn around and you believe it with all of your heart...dem's 'de rules!
Fortunately, Popes are not stupid enough to invoke this crazily awesome power over 1/6th of the world's population - but it most certainly does happen - and could easily happen again tomorrow. Obviously as a layperson - you can choose to ignore things that the church tells you to believe (Wow - what a concept!) - but by doing so, you are denying not one but two of the central tenants of the thing you claim to believe. If you don't believe in papal infallibility - then you failed two of the four key distinctive features of Catholicism (because that "Vicar of Christ" thing means more or less the same thing). If you deny the literal infallibility of the pope, all you have left is a literal belief in "purgatory" (Wow - you really believe that?) and a literal belief that Mary was conceived immaculately just as is claimed for Jesus (Not such a big deal - once you've accepted one scientific impossibility - you might as well assume it runs in the family!) Both of those are relatively small distinctions compared to other branches of christianity. I'm sorry - but you are not a Catholic by any of their own definitions if you don't believe in papal infallibility.
Now - do we still want to try to claim that 'belief' in a scientific theory is anything remotely like this crazy religion?
SteveBaker (talk) 00:12, 8 May 2009 (UTC)[reply]
Steve, just stop this. Yes, logic dictates that people who don't believe in two central tenets of Catholicism are not Catholics. But logic also dictates that a magic man didn't create the world in seven days and that there are no talking snakes. You seem to be a rational person, so I'm surprised that you think people who believe the Bible--which contradicts even itself--would care about the logical contradiction you mentioned. --99.237.234.104 (talk) 02:40, 8 May 2009 (UTC)[reply]
Steve, there's so much utter rubbish in what you said above that I'm not going to even try to counter it point by point. You're a scientist whose made it abundantly clear he doesn't believe in God, religion, the Bible or churches. That's fine. Are you entitled to read about things you're not an expert on, and form a view? Of course you are. But reading a few things about these topics does not make you an instant expert on what churches actually teach. And it certainly doesn't give you licence to interpret them from your own very limited perspective (which is OK privately) and then spread complete falsehoods in public arenas like this, on the basis of "What I, Steve Baker, think this means is the truth". Please stick to your own area of expertise, and let those who know about other things talk about them. -- JackofOz (talk) 03:44, 8 May 2009 (UTC)[reply]
Beautiful...simply beautiful. Right there you answered the OP's question more eloquently than I ever could. As a non-Catholic, I'm apparently not allowed to read about this religion and form opinions about it because that's a matter of faith. Publicly available data on the rules of this religion (it seems) count for nothing and you have to be an insider to be allowed to understand them. Science, on the other hand encourages everyone to read, learn and understand what's going on - to ask deep questions and have them answered. JackofOz does not allow the possibility that I may form my own opinion and discuss it - and when I attempt to do so, he just brushes my carefully addressed points aside. Nice one! Thank you! SteveBaker (talk) 13:52, 8 May 2009 (UTC)[reply]
JackOfOz didn't complain that you're a non-Catholic, he complained that you don't have much knowledge of Catholicism, which is an entirely different thing. -- BenRG (talk) 15:45, 8 May 2009 (UTC)[reply]
I certainly didn't complain about Steve's Catholicity or lack thereof. As a non-Catholic, I could hardly do that. Even as a Catholic, one could hardly do that. If you read what I wrote, Steve, you'll see I explicitly said you are entitled to read whatever you like, and form whatever opinions you like. You don't need me to give you permission to do that. But opinions do not equal fact. The point I was making was that you were coming over all high and mighty about Mr K allegedly making inaccurate and misleading statements, but in the process of complaining, you were committing even more grievous errors yourself. That's usually called hypocrisy. There comes a time in everyone's life when they have to say "I was wrong. I'm sorry". Your time is now, Steve. I know you haven't had much practice, but I have unbounded faith that you can do it. -- JackofOz (talk) 21:59, 8 May 2009 (UTC)[reply]
I have to say that the impression I had of the Roman Catholic beliefs, after reading the relevant Wikipedia articles, was very close to Steve's interpretation. If you beg to differ, please do counter Steve's post point by point; I'm genuinely curious, and since the OP's question was about religion, such a counterargument wouldn't be out of place here. The articles on Roman Catholicism should also be corrected if they do not give non-catholics a fair perspective on the religion, and I'd love it if you could improve them. --99.237.234.104 (talk) 04:05, 8 May 2009 (UTC)[reply]
The problem (or indeed, beauty, depending on your perspective) of a belief based system like a religion is that it no-one but the believer him or herself can explain exactly who or what they believe in. A religious belief may be ephemeral, incorporeal, idiosyncratic and highly personal, since it has formed in a unique way for each individual. That holds true whether you consider it to have formed by connection with a deity, be an emotional or psychological support system or whether you consider it to be a successful meme exploited for social control of the masses.
As a scientist myself, its often tempting to counter individual's beliefs by citing the illogical tenets of the religious group they are a member of. But thats a bit like defining the complex sum of an individual's political, moral and ideological values by the political party they vote for: its an oversimplification that fails to take into account the complexity of individuals. I think its very important to distinguish between individual beliefs and religious dogma; they can be very different.
On the subject of the original question. I heard Bruce Alberts give a seminar on the future of science education today. He addressed this very issue. He told how he currently sits on an United States National Academy of Sciences committee who are looking at how to improve science teaching in the US. They did some focus groups with college educated adults in the US and found that the majority consider religious interpretations and scientific interpretations of natural phenomena to be equally valid alternatives for understanding our world. This is extremely worrying for science educators, of course, because only scientific interpretations use, among other techniques, falsifiability and testing of the null hypothesis, to challenge belief, and reach firm conclusions without predetermined bias. It is this rigorous scientific method that separates a scientific finding from a religious belief. Alberts didn't see this as a battle between science and religion, though, merely a failure on the part of science educators (and legislators) to put a framework in place through which science can be taught correctly. Rockpocket 05:26, 8 May 2009 (UTC)[reply]
The only point I'll counter is this one: The dogma of papal infallibility has been used precisely twice - in 1854 to proclaim the Immaculate Conception, and in 1950, to proclaim the Assumption of Mary. Of all the millions of things that popes could theoretically proclaim papal infallibility about, they've chosen exactly two things, the last time being 59 years ago. Have they used it to proclaim that God exists? No. Have they used it to proclaim that Jesus Christ was a real, historical person? No. Have they used it to proclaim that he rose from the dead on Easter Sunday? No. Have they used it to proclaim that the words arttributed to Jesus in the Bible were actually spoken by him? No. I could go on ad infinitum about reasonable subjects of papal infallibility. But the one that Steve chose to use was something as ludicrous as God telling a pope in a dream that there's life on Mars, and the pope then demanding his flock believe it as the literal and incontrovertible truth - as if this were a remotely likely scenario. Not even he believes that. -- JackofOz (talk) 06:34, 8 May 2009 (UTC)[reply]
Certainly it's not often used - but when it is, as a Catholic, you have two choices - accept it (however ridiculous it might be) or admit that you are not in fact a Catholic at all. There is no debate - no "freedom of speech" - the only freedom you have is to leave the game - you cannot deny either of those two proclamations - you cannot debate them - you aren't even allowed to doubt them.
If you are a scientist there are two fundamental differences:
  1. You're expected to question everything - there is no central dogma (in the sense of the definition in dogma) - you can refuse to accept global warming against all of the evidence - and still be a scientist.
  2. Evidence. No pronouncement of either immaculate conception or the assumption of Mary would be possible in science without peer reviewed evidence and independent duplication of findings. Theories have to be falsifiable and they must make testable predictions or nobody will pay attention to them. Hence you don't have proclamations coming from nowhere.
SteveBaker (talk) 13:52, 8 May 2009 (UTC)[reply]
What SteveBaker is consistently ignoring - because it doesn't fit the straw-man scenario he wants to set up - is that infallibility can apply only to matters of faith and morals. If it's a scientifically evaluable hypothesis (e.g., life on Mars), it's a pretty good bet it's not a matter of faith and morals. The doctrine of infallibility simply can't be invoked in the way he wants to invoke it. It's unfortunate; he didn't have to use a caricature of the doctrine to make his point, but he chose to, and chooses to keep doing so, despite people warning him that doing so makes him look bad. - Nunh-huh 16:44, 8 May 2009 (UTC)[reply]
(Oh - and by the way, the English translation of the text of the 1854 proclaimation is: ""We declare, pronounce and define that the doctrine which holds that the Blessed Virgin Mary, at the first instant of her conception, by a singular privilege and grace of the Omnipotent God, in virtue of the merits of Jesus Christ, the Saviour of mankind, was preserved immaculate from all stain of original sin, has been revealed by God, and therefore should firmly and constantly be believed by all the faithful." - the 1950 dogma says "By the authority of our Lord Jesus Christ, of the Blessed Apostles Peter and Paul, and by our own authority, we pronounce, declare, and define it to be a divinely revealed dogma: that the Immaculate Mother of God, the ever Virgin Mary, having completed the course of her earthly life, was assumed body and soul into heavenly glory" - which means that both God and Jesus have indeed been proclaimed to exist since neither dogma would make any sense whatever if they didn't...although I'll grant that "as a real historical person" is a kinda slippery term here. SteveBaker (talk) 14:01, 8 May 2009 (UTC))[reply]
There you go again, asserting things to be the case without any authority except your own logic. I have nothing against logic, but it's not always relevant when it comes to matters of faith. God and Jesus have indeed been proclaimed to exist - indeed, they're central to any concept of faith in what the RC Church teaches - but no pope has ever invoked his alleged papal infallibility about these subjects. That's a fact; what you're arguing is not. -- JackofOz (talk) 21:59, 8 May 2009 (UTC)[reply]
Furthermore, Steve's statement as a Catholic, you have two choices - accept it [papal infallibility] (however ridiculous it might be) or admit that you are not in fact a Catholic at all. is simply not accurate. (Most) Catholic priests don't demand you adhere to every single Catholic doctrine or else you are banished from the church. It simply doesn't work that way. Millions of Catholics use artificial birth control methods, for example, but they are still Catholic and are still welcomed to church every Sunday. In practice religion is about self identification, not logic, and this would be borne out by the many, many Catholics who would decline to accept either of Steve's absolute choices. Rockpocket 01:12, 9 May 2009 (UTC)[reply]
I'd go further than that: My understanding is that the Catholic Church's position is "once a Catholic, always a Catholic". Once you're confirmed, that's it; there's no way out.
If you come to disbelieve the things the Church says you have to believe, then you might be sinning, and if you make enough of a fuss about it and are prominent enough, you could even be excommunicated. But you can't become an ex-Catholic, from their perspective. --Trovatore (talk) 01:22, 9 May 2009 (UTC)[reply]
I think that's pretty close to the mark, which explains why the RC Church's tally of its claimed adherents is way larger than the tally of people who actually believe and practise their teachings. A more reasonable definition of "Roman Catholic" would be a person who (a) is accepted by the RC Church as a member and (b) considers themself to be a member. Which is why a lot of people (such as me) who were raised as Catholics but have parted company no longer ever describe themselves as such, and what the Church has to say about it, as far as they are concerned, is now irrelevant. -- JackofOz (talk) 01:51, 9 May 2009 (UTC)[reply]
I probably should add that this discussion is way off the pace on the relationship between science and religion. A decade or so ago I wrote a short summary for a student here, particularly chapter 3 & 4 but even that was pretty simplistic. The issue of the nature of both scientific and religious belief is a topic which a lot of people think they are experts in but in reality statements on how each work in practice need to be tested against serious study of the nature of truth in both arenas in far more detail than a typical working scientists understanding of falsifiability, or prejudice based on people meeting poor examples of scientists or religious believers. --BozMo talk 06:48, 8 May 2009 (UTC)[reply]

Short clarification: my question was indeed about belief in things that you cannot prove by yourself, not about scientific knowledge. Mainly, it was about why the belief in, say, results of particle accelerators are different from, say, the believes in the bible, Torah or Koran. It was also not specific about Christian faith, although many people online tend to think that Christianity is the archetypical religion. --Mr.K. (talk) 11:31, 8 May 2009 (UTC)[reply]

To answer that directly, I think it's a matter of trust. Do you trust someone, or a group of people, to tell you what is true? To make that choice, you look at where that information has come from, how it was obtained, does it agree with other "truths", as you may define them, or contradict them? --Rixxin (talk) 11:51, 8 May 2009 (UTC)[reply]
It's not entirely about trust. I don't have to trust some other scientist's claims because a part of the process of publication is that he explains in detail how the experiment (or whatever) was performed. Where these conclusions come from. That has to be done in sufficient detail that I could (in principle) repeat the experiment myself. Hence I don't have to trust the guy. When the 'Cold Fusion' fiasco was breaking news - the papers that were published explained in detail how the apparatus was constructed and how measurements were taken. Other scientists who did not "trust" those results built similar equipment and tried the experiments for themselves - thereby discovering the problems with it. That's how cold fusion was eventually debunked. No "trust" was required - which is just as well because it finally turned out that the original team were not entirely trustworthy. SteveBaker (talk) 14:06, 8 May 2009 (UTC)[reply]
I'm not just talking about individuals though, Steve. In your example, the eventual outcome was that the team involved were found to be untrustworthy. That occurred via other people following the scientific method. So when I read a copy of New Scientist, and see the latest amazing discovery, I know I could go recreate that experiment in my basement, but I'm not gonna' - because I trust (note, "trust", not "know") that the system has filtered out the possibility of bad science.
Don't get me started on what I feel when I pick up a copy of the Bible!
--Rixxin (talk) 16:08, 8 May 2009 (UTC)[reply]
But your question was answered above, Mr.K. Scientific beliefs are more valid than religious beliefs because scientists are more rigorous in deriving their conclusions and their methods are subject to much scrutiny. Zain Ebrahim (talk) 12:07, 8 May 2009 (UTC)[reply]

Short clarification II: I know that my question was answered above (by Steve Baker in his first post, thanks Steve). My short clarification was related to the discussion after the answer to my question. --Mr.K. (talk) 12:10, 8 May 2009 (UTC)[reply]

Meta comment: I sometimes wish we would act more like a library reference desk, wherein we provide the questioner with a brief outline of the answer, and then guide them to appropriate authoritative sources where they can learn more. This is especially recommended when dealing with questions like this one that, despite some valiant attempts above, cannot be adequately answered by providing our individual views in a paragraph or two.
References: So, here are a couple of books/essays that you may find useful:
And here is a contrarian view:
I am sure other editors heres will have their own recommendations. Abecedare (talk) 22:45, 8 May 2009 (UT

scientists want to know what is god , what dose helook like , how mush space is he occupaying , how old is he

if you do belife in god then you should take it without a prof , because how can you understand the exsistance of god while

we still make theories about our exsistance , i belive that god is a form we can't understand that exsist out side time limit.

you just belive in it , with out use math expression to prof him .

the question is who is the right religion , muslim , christian , ... , which one to follow .....??????????--Mjaafreh2008 (talk) 11:13, 9 May 2009 (UTC)[reply]

Many scientists decide to invoke Occam's razor, and say that there is no god. This is discussed in the religion subsection: Occam's razor#Religion. --Mark PEA (talk) 23:57, 9 May 2009 (UTC)[reply]

Saw sheet of tiny plant 1/8th inch high

I was walking along a track in a forest in the southern UK recently, in a low-lying area where the ground was soft but not wet-looking. Spreading over a rut in some bare earth, I saw a tiny very flat plant forming a flat green sheet over the soil. It was maybe 1/8th inch high, and consisted of thousands of tiny leaves. The leaves had no veins, and I did not notice any stems either. Has anyone got any idea of what it was please? 78.145.24.191 (talk) 14:07, 7 May 2009 (UTC)[reply]

Hard to say without a photo. Some type of moss perhaps? —Preceding unsigned comment added by 65.121.141.34 (talk) 16:18, 7 May 2009 (UTC)[reply]
Mind-your-own-business?--86.25.194.130 (talk) 17:04, 7 May 2009 (UTC)[reply]
Haha, I thought you were being rude! :) --Sean 14:02, 8 May 2009 (UTC)[reply]

From the photos, mind your own business is taller than 1/8th of an inch, and it has stems. It was not a moss or lichen either. The distinctive feature of it is that it was a low as its possible to get, and the rounded leaves were tiny too. 89.242.98.204 (talk) 20:25, 7 May 2009 (UTC)[reply]

Well, if it was a moss, then it did not have the feathery leaves I associate with moss. The very tiny leaves were shaped like deflated balloons, in plan like the end of a spoon. 78.147.3.176 (talk) 08:51, 8 May 2009 (UTC)[reply]

Could it have been a slime mold?

where's the article for NH2COOH?

I can't seem to find out whether this molecule is a stable compound or not. Entering this formula into google brings up irrelevant results. I know glycine is the smallest biological amino acid ... but what about the smallest amino acid in general? Wouldn't that involve just one carbon? Or is there some reason why this molecule doesn't appear to be that popular? John Riemann Soong (talk) 16:32, 7 May 2009 (UTC)[reply]

By the usual definition of an amino acid, glycine IS the smallest.
    COO-
  + |
H3N-C-H
    |
    R
Where "R" is the side chain that determines the identity of the amino acid, with an "H" at this site being glycine. I don't know that there IS such a thing as H2N-COOH and if so it would fail to qualify as an amino acid by the common definition. --- Medical geneticist (talk) 16:46, 7 May 2009 (UTC)[reply]
I'm sorry we weren't able to help - but Wikipedia has only 2.8 million articles - but I bet there are more than 2.8 million possible chemical compounds out there - so it's inevitable that large numbers of the more obscure ones won't have their own pages. If this one isn't even stable (I have no idea, I'm not a chemist) - then it's really unlikely that it would have an article - and if there are zero Google hits then it's really not notable enough to have one. (Notability is a fundamental Wikipedia requirement for the subject of any article). One question I might ask though - is this the standard way to write it's formula? If not - it would be no surprise that you didn't find any hits. SteveBaker (talk) 16:48, 7 May 2009 (UTC)[reply]

One answer I might give is No, this is the standard way to write its formula Cuddlyable3 (talk) 19:09, 7 May 2009 (UTC)[reply]

Some estimates place the number of possible "small molecule structures" (i.e., not including polymers, where especially things like proteins and DNA/RNA can have arbitrary length and permutations) at 1030–10200. So if even only a teeny tiny fraction actually exist, "yo, that's a lot of compounds!" DMacks (talk) 21:54, 7 May 2009 (UTC)[reply]
That's an interesting question. Usually, an amino acid is considered to be H2N-CHR-COOH, but its technical definition is any molecule with amine and carboxyl functional groups. So yes, technically what you're asking about may be an amino acid, but it's not usually thought as one. It's called carbamic acid. 129.49.7.150 (talk) 16:52, 7 May 2009 (UTC)[reply]
WP:WHAAOE whether Steve likes it or not! --Tango (talk) 16:55, 7 May 2009 (UTC)[reply]
It never ceases to amaze me -- sometimes it takes just the right search terms, but WHAAOE indeed. --- Medical geneticist (talk) 20:58, 7 May 2009 (UTC)[reply]
(ec) It's carbamic acid. Lots of esters are known (and carbamate is a good protecting group for amines), but the free acid decarboxylates readily, so it's mainly an intermediate structure that isn't isolated. It's also probably a transient intermediate in the Hofmann rearrangement. DMacks (talk) 16:56, 7 May 2009 (UTC)[reply]

What causes fishy odour?

What fundamentally causes odours that the human nose perceives as "fishy". People often describe fish as having a fishy smell, as well as bacterial vaginosis and trimethylaminuria. Is there a fish-smelling molecule these have in common that we have certain receptors for?--76.10.155.30 (talk) 18:03, 7 May 2009 (UTC)[reply]

Is it maybe Trimethylamine N-oxide in all 3 cases?--76.10.155.30 (talk) 18:05, 7 May 2009 (UTC)[reply]

Lots of molecules with amine groups have a fishy smell (including ammonia, methylamine and dimethylamine, for example), it is therefore likely that olfactory receptors that specifically detect amine groups mediate the characteristic fishy odor. A new class of olfactory receptor was recently discovered by Linda Buck's lab, the TAARs. They propose that, in the mouse, TAAR2 though TAAR9 detects various volatile amines. Rockpocket 04:36, 8 May 2009 (UTC)[reply]

Relation between two angles in doppler effect

These are, in fact, questions came up while looking at an article for Relativistic Doppler effect. There is a formula

where is the angle of the light for the observer, is the angle of the light for the source, v is the relative velocity of the source to the observer, and c is the speed of light (If they are mistaken, please correct). I don't know how this formula is derived. Assuming a triangle of c, v (at an angleof ) and one more oblique side, for v and the oblique side (if that is ) does not yield what the formula states. How is it derived? And one more related to that. That is used to determine the Relativistic Doppler effect as

where is the frequency of the light for the observer, is the frequency of the light for the source, and is the Lorenz factor. What does mean? For more detail, see Relativistic Doppler effect#Motion in an arbitrary direction —Preceding unsigned comment added by Like sushi (talkcontribs) 18:47, 7 May 2009 (UTC)[reply]

The derivation is given in one of the external links to the article. SpinningSpark 21:09, 7 May 2009 (UTC)[reply]
Thank you for showing the source one of the external links. But I can not follow the jump in formulas there. How can
be derived from
by just "differenciating both sides with respect to and dividing through by , and noting that ?
seems unclear to me.
And there is not the exact formula
anywhere.
By the way, there seems to be a calculation error for
where the far right side should be
—Preceding unsigned comment added by Like sushi (talkcontribs) 06:42, 8 May 2009 (UTC)[reply]

Arbitrary editing break

Differentiating the starting relationship;
with respect to ta yields;
Making the substitution yields;
Dividing through by the factor and a small re-arrangement;
and substituting in the velocities yields;
as required. SpinningSpark 09:15, 8 May 2009 (UTC)[reply]
Thank you again for your answer. I can understand from the third formula,but I do not understand differentiation for the first formula to the second. I looked at the page derivative which was in a disambiguation page for "differentiation", but still not.Like sushi (talk) 12:17, 8 May 2009 (UTC)[reply]
The rhs uses the vector dot product identity and differentiating wrt t using the chain rule results in;
Observing that A and dA/dt have the same direction, this can be put back into the dot product form as;
Making suitable substitutions for A and t yields the rhs of the second line above.
The lhs can be differentiated using the product rule or again using the chain rule.
If you have questions on the basic maths rather than the science, you might be better off asking at the maths desk. SpinningSpark 13:10, 8 May 2009 (UTC)[reply]
Thanks for advice. I understand what you wrote for me in the last post. The content of the internal link product rule served very much for me to understand. Isn't it a good article? Anyway, I will ask math reference desk for another unclear point which is more of mathmatical one,
Yet, there are some points to ask here in science reference desk.
The source says
Is it not
? And the derivation of
is not yet found. I could reach
so far. Like sushi (talk) 15:05, 8 May 2009 (UTC)[reply]
It's a shame that our article relativistic aberration does not give the derivation. Perhaps when you understand it, you will add it for us. Try this paper which seem to have accesible mathematics (no tensors and such). He defines the angles differently, they are at 90° to the Wikipedia formula and he has made the angle as observed at the transmitter the subject of the equation instead of the receiver. But it is just simple algebra and trig to transform it, you should be able to cope. SpinningSpark 18:50, 8 May 2009 (UTC)[reply]
I read the paper.
The equation (2)
(How can I write long overline above AB etc.?)
is understandable, altough why the elmentary sources A and B, which start to emit the spherical wavelets at the same time in the reference frame where the emitter is at rest, start to emit it at an interval of presented in formula (1) (where B is to the direction of emitter's movement from A and starts to emit later) is not detailed in the paper. But substituting (1) into (2) does not yield (3).
Like sushi (talk) 06:40, 9 May 2009 (UTC)[reply]
I have no trouble getting from that, you have made a mistake in the algebra. The bars are done with the "overline" function, for example \overline{\mathbf{AB}} results in . You might be interested in Help:Displaying a formula. SpinningSpark 13:39, 9 May 2009 (UTC)[reply]
Thank you. I have made a mistake. I get now.
And if we take it in wikipedia's way we get
.
That is the same with what we get from
if and .
Like sushi (talk) 04:04, 11 May 2009 (UTC)[reply]
And I tried to continue calculation.
Letting
(5)
stand (Does only the distance of elementary sources in the direction of v matters?), I could manage to follow till
(9)
But it yields not
(10)
but
(A)
In detail,
using the relation of Quadratic equation and quadratic formula,
unfolding "-4ac" part,
canceling and taking out,
canceling 2 and, replacing with (by using and part of basic relationships), and multiplying both the numerator and the denominator by ,
(A)
But this is obviously wrong because if , for , we get infinity because is 0.
Like sushi (talk) 14:09, 11 May 2009 (UTC)[reply]
Well, Imade some mistakes again, I get
(10)
al right now.
Sorry to make you troubled.
Like sushi (talk) 07:49, 13 May 2009 (UTC)[reply]
Well, I've got it!
Now I can derive it from the equation .
Like sushi (talk) 07:49, 13 May 2009 (UTC)[reply]

Mercury and Venus

How could people say Venus is overload with brightness? Venus is only 1.3 (0.72 AU) times closer to sun than us, it will only be 2 times brighter than sunlight. Mercury is only 2.5 times closer to sun, that's 6 times brighter than sunlight on earth. How could not having an atmosphere unable to protect extra sunlights. how could Venus be far too bright when it's far enough from the sun and it have a atmosphere?--69.229.4.179 (talk) 22:35, 7 May 2009 (UTC)[reply]

When you asked this exact same question last week, all you did was pick a fight with anyone who tried to answer it. You ask it again, and your first four words indicate you wish to pick a fight yet again. When you start a question with the phrase "How could people say..." it indicates you wish an arguement, not an answer, and it makes it so people do not want to answer your question seriously. Plus, it is entirely unclear as to what you mean by brightness? There are three ways to interpret such a vague term: a) apparent brightness as viewed from Earth; b) absolute brightness in terms of the amount of light given off by the planet, and c) Average daytime brightness on the planets surface. All three are very different ideas, and unless you can tell us exactly what you are talking about, it is impossible to answer your question. --Jayron32.talk.contribs 22:48, 7 May 2009 (UTC)[reply]
(ec):I am not sure I understand your difficulty, but I suggest you read our articles on Venus and Mercury. In particular Venus#Atmosphere and climate explains why the two planets are so different. Your question might be why is the temperature of these two planets not strictly in accordance with the inverse square law. If that is so, the section in the Venus article I pointed you to will explain that. The short answer is the greenhouse effect. SpinningSpark 22:56, 7 May 2009 (UTC)[reply]
  • Surface temeperature is not my question sorry, the one I meant by is if human orbit at those planets looking down's sunlight brightness. I'm afraid I confuse people by planet color questions. The prompt is most clear I can get.--69.229.4.179 (talk) 23:13, 7 May 2009 (UTC)[reply]
Mercury has a much lower albedo than Venus so even though it is closer to the sun it has a smaller absolute magnitude. An obsever orbiting each planet would perceive Venus as being the brightest. SpinningSpark 23:29, 7 May 2009 (UTC)[reply]

EU / US mpg difference

According to Honda's Dutch website, a new civic gets 5.5 l / 100 km mileage on the highway, with the same 1.8 l engine the US model has. According to the US Honda website, the best the same manual transmission civic will get is 34 mpg, which according to google is 14.5 km / l or 6.9 l / 100 km.

A 6-cylinder volvo V70, again according to the specifications for both websites, this is highway mileage and I do take care to look up the right fuel, transmission, etc, the US site has 9.4 l / 100 km, and the Dutch site has 8.1 l / 100 km. The same difference for another model, the huge XC90: Dutch site has 8.1 l / 100 km, US site has 11.8 l / 100 km.

Any car designers around here on the science desk? What causes this consistent difference? I've heard many stories from different people on this - EU cities would be narrower and bumpier, etc, and the cars would use diesel. But these are the same models, by the same manufacturers, using highway mileage. I assume it's some kind of regulatory thing?

128.200.40.72 (talk) 23:47, 7 May 2009 (UTC)[reply]

My first guess is that there is a regulatory difference, and that the "test drive" which is used to measure the mileage in the United States is replicated on a different test-drive course for Europe (presumably with different road types and driving styles). Whether this is the result of lax regulation, or an honest and sincere effort to match the driving conditions of the different continents, is subjective. Nimur (talk) 01:31, 8 May 2009 (UTC)[reply]
Possibly they are not tested at the same speeds. 55mph is a common speed limit in the US and this may be the test speed. SpinningSpark 01:39, 8 May 2009 (UTC)[reply]
Although our article fuel economy in automobiles seems to be suggesting that 65 mph gives better fuel economy than 55 mph in many cases! SpinningSpark 01:44, 8 May 2009 (UTC)[reply]
Actually, only the '97 Celica is a little bit better at 65mph than 55mph. A few more are marginally better at 65mph than at 45mph. I'd imagine there's an overdrive gear kicking in or a lock-up torque converter engaging. (It's the US, so I'm fairly sure they were testing using automatic transmissions). Franamax (talk) 08:39, 9 May 2009 (UTC)[reply]
The US has recently changed its test procedures, so you may be seeing the new "more realistic" numbers for the US. And as noted above, test procedures are different in different regulatory areas - they are meant for comparison between vehicles within the area. Another important consideration is the different settings for the engine management system between the US and Europe. The EMS is set up to achieve emission standards for things like NOx and unburned hydrocarbons. I'm speculating, but maybe you wouldn't get as much acceleration from a Euro version. Franamax (talk) 08:51, 9 May 2009 (UTC)[reply]

Ringnecked parakeet

What does its mean when she does that wierd thing with her eyes, when she rapidly shrinks and expands the pupils of her eyes? They go from pinpricks to wide as saucers every few seconds while she's doing that. —Preceding unsigned comment added by 81.79.87.167 (talk) 23:51, 7 May 2009 (UTC)[reply]

Means she's watching Blade Runner? :) Now seriously, it could be stress, or could be changing light levels. Red light produces strongest response AFAIK. --Dr Dima (talk) 00:36, 8 May 2009 (UTC)[reply]
The behavior is known as "eye pinning", and its function does not seem to be all that well understood, see this answer at about.com. Looie496 (talk) 01:19, 8 May 2009 (UTC)[reply]
Male Budgerigars often pin their eyes when performing their mating displays, FWIW. I suppose that the hens find it attractive. --Kurt Shaped Box (talk) 00:22, 9 May 2009 (UTC)[reply]


May 8

Skin color in Hindu art

I've looked on Wikipedia but haven't found an answer to my question. In various types of Hindu and/ or Indian visual arts, various divine figures and some non-divine figures (as far as I can tell, not having much knowledge of Hinduism), are often depicted with skin tones of bluish tint, or other colors which appear to be non-naturally occuring. Is there any scientific explanation for why such skin tones are used? What is the symbolic/religious meaning? Thank you. --71.111.205.22 (talk) 01:04, 8 May 2009 (UTC)[reply]

Well from a scientific point of view there's not much i can say about this. But from a religious point of view, many hindu gods, as you, said, are depicted blue. Vishnu and many of his avatars are portrayed blue, which is what you might have been. This might have something to do with the fact that the sea, highly regarded as a symbol of power by the hindus, is blue. But that's not saying much... some questions really don't have a comprehensive answer...Rkr1991 (talk) 08:24, 8 May 2009 (UTC)[reply]

Krishna is often portrayed in blue. Hinduism uses a lot of symbolism to depict its deities and blue, amongst other things, denotes transcendentalism. Remember, the blue depiction of god is in our material universe and on our planet, the sky is blue. The gods are in the spiritual abode and the entire material universe is a cloud in the sky of the spiritual abode. So, do you get the symbolism now? Sandman30s (talk) 08:41, 8 May 2009 (UTC)[reply]
That helps a lot. Thanks for all your assistance! --71.111.205.22 (talk) 15:14, 8 May 2009 (UTC)[reply]

Among the Hindu Trimurti (see File:Brahma Vishnu Mahesh.jpg):

  • Vishnu and his avatars Rama, Krishna etc. are typically depicted blue-skinned (often with yellow dhoti); in fact the word Krishna literally means "black; dark; or dark blue"
  • Shiva is depicted white-skinned (with a blue tongue)
  • Brahma is depicted either red-skinned and/or wearing a red dhoti.

Other deities have different associated colors and symbols; see this very incomplete table for some more information. It is possible to interpret all these associations symbolically but one should be aware that the symbolic meaning is likely to vary with time, place and philosophical leanings of different sects. Abecedare (talk) 18:27, 8 May 2009 (UTC)[reply]

Shiva's blue throat is a result of the poison he holds there. Jay (talk) 11:38, 11 May 2009 (UTC)[reply]
FWIW Osiris, the ancient Egyptian god of the underworld, was commonly depicted as a green (the color of rebirth) or black (alluding to the fertility of the Nile floodplain) complexioned pharaoh in mummiform. Cuddlyable3 (talk) 07:24, 13 May 2009 (UTC)[reply]

Gravitational potential of an 'eccentric' object

I'm trying to find the gravitational potential of the 3-dimensional object described by a sphere of constant density and radius 1 centered at the origin (0,0,0) - 'A', but hollow in the sphere of radius 1/2 centred at (1/2,0,0) - 'B' - and hollow in the sphere of radius 1/4 centred at (-1/4,0,0) - 'C' - for some point x outside of A- effectively a full sphere of density plus 2 spheres of density as described previously. Can we just effectively 'add' the potentials for the respective spheres so that, if M is the mass of the large sphere 'A', we have , where b and c are the respective sphere centers? Is this at all valid?

If so, is there any point in C where a particle could theoretically remain at rest? I think not due to the asymmetry along the x-axis but I'm not sure how to show it...

Thanks very much! Mathmos6 (talk) 04:27, 8 May 2009 (UTC)[reply]

You can just add the potentials since Newtonian gravity is linear, but the answer you wrote down isn't correct because GM/r is the potential of a point particle, not a sphere of constant density. (Also it has the wrong sign.) Your potential is fine (aside from the sign error) if you're only interested in the |x| > 1 region. There is an equilibrium point somewhere in C because every potential function, no matter how misshapen, has to have a minimum somewhere. The minimum in this case is a ring around the x axis. There's also a saddle point (unstable equilibrium) on the x axis. -- BenRG (talk) 11:42, 8 May 2009 (UTC)[reply]
Sorry, I just noticed I didn't read your question very carefully. I think the saddle point is in C (the hole of radius 1/4). It shouldn't be hard to prove that, but you'll need to write down an inside-C, inside-A, outside-B potential. -- BenRG (talk) 12:23, 8 May 2009 (UTC)[reply]
Oh, also, the potential is a scalar, so you want , not . Funny I didn't notice that before. -- BenRG (talk) 12:31, 8 May 2009 (UTC)[reply]

Fair enough, I was only interested in the region outside the largest sphere yes, although I can't say I'd know how to calculate the potential inside the big mess in A - also, I see your point about the scalar, but don't we need to take the fact that the potential isn't radially symmetrical into account somewhere though? Or do the 'x-b'/'x-c's do that anyway? Thanks a lot! Mathmos6 (talk) 03:51, 10 May 2009 (UTC)[reply]

The |x-b| and |x-c| takes care of that. There are four regions (inside B, inside C, inside A but outside B and C, and outside A) and you'll have a different formula for each one. Or you could write down a single formula with max/min or some such, but you'd end up having to split into four cases to prove anything interesting anyway. If you can write down the potential inside and outside a single spherical object then you can write down the potential in all four regions, since it's just a sum of the inside/outside potentials of A, B, and C (with the B and C potentials negated). -- BenRG (talk) 22:42, 10 May 2009 (UTC)[reply]

How to ensure 100% RDA with supplements?

Dietary supplements usually contain 100% RDA of some micronutrients, but some ridiculously high or low value of many others, if they contain them at all. Given that it is prohibitively complicated to find out how much of each micronutrient we actually get in our food each day, and given that it probably won't hurt to get a little bit more than 100%, wouldn't it be nice if there were a supplement on the market that had simply 100% of every micronutrient, just to be safe? Why is there no such "100% supplement" available? Or is there? Mary Moor (talk) 06:02, 8 May 2009 (UTC)[reply]

You don't say where you are, but in the UK there are such supplements available: Centrum makes one [1] --TammyMoet (talk) 08:37, 8 May 2009 (UTC)[reply]
Yep, I've got a bottle of '100% everything' in my own cupboard. I got mine from Boots, I think - though I've seen similar in most pharmacies and health food shops that I've been into... --Kurt Shaped Box (talk) 10:31, 8 May 2009 (UTC)[reply]
Thank you; maybe I have to look some more. I'm in the US, in the state of Washington. Mary Moor (talk) 15:46, 8 May 2009 (UTC)[reply]
A word of advice. Don't pay extra for products that claim to contain 500% (or more) of your RDA. You don't need *that* many micronutrients and vitamins. I don't think that your body can even do anything useful with the excess.--Kurt Shaped Box (talk) 00:21, 9 May 2009 (UTC)[reply]
Not only do you not need that level, high levels of some micronutrients can make you quite ill. Franamax (talk) 09:11, 9 May 2009 (UTC)[reply]
Unless you've got a very unhealthy diet, vitamin pills will not do you any good. Overdoses of vitamins in fact do you harm - more is not better. I used vitamin pills for many years until I started reading scientific research about them, which showed that overdoses reduced longevity, increased cancer rates, kidney damage, etc. Now I never use them (with the exception of Vitamin D in the winter as I am in northern latitudes). Better to put your time and money into eating a healthy varied diet - plenty of vegetables and fruit etc. Its very likely that there are many as yet undiscovered micro-nutrients that have long term effects, and these are only found in healthy foods, not pills. If you want to get particular vitamins or minerals then its best to find out which foods are rich in them and eat those. For example B12 - sardines. Selenium - brazil nuts. And many others. Sardines are also rich in calcium and Omega3, which illustrates how natural foods give you extra nutrients for free. 78.146.190.197 (talk) 13:00, 16 May 2009 (UTC)[reply]

neutrons, protons and electrons

During my one year of chemistry class at school, I had other things on my mind. But now I try to learn on my own...
My very elementary question is: are all neutrons, protons and electrons the same?? That is, can you see for instance, this is a typical oxygen electron and that is a typical sodium electron? Or is there no such difference? Lova Falk (talk) 06:17, 8 May 2009 (UTC)[reply]

Yes, as far as we know, all protons are identical to each other. None of them have nicks, bumps, or customized parts hanging off of them. Same with neutrons, electrons, etc. An electron is an electron - there's no difference between oxygen electrons and sodium electrons. We have an article (of course), Identical particles, but that's rather advanced. Clarityfiend (talk) 06:59, 8 May 2009 (UTC)[reply]
Thank you! This is actually a bit shocking. In the articles on neutrons, protons and electrons, the article Identical particles is not mentioned in "See also". Do you think I should put a link there? Lova Falk (talk) 07:17, 8 May 2009 (UTC)[reply]
Probably not, because this is a generic property of all quantum particles (and even atoms and small molecules). Maybe subatomic particle should link to it. -- BenRG (talk) 21:53, 8 May 2009 (UTC)[reply]
See no-hair theorem for the black hole version of this identity issue. --Sean 14:06, 8 May 2009 (UTC)[reply]
They're not quite the same. For example: The two electrons in a helium atom have opposite spin. It's easy to change these differences, however. — DanielLC 15:45, 8 May 2009 (UTC)[reply]
No, Daniel, you are wrong. Single-electron states may have opposite spin, but the electrons occupying these states are still identical. If they were not identical (i.e., distinguishable), then the helium atom would have had many more quantum states than it actually has. Both atomic physics and statistical mechanics would have looked very differently if at least some of the electrons were distinguishable. --Dr Dima (talk) 18:25, 8 May 2009 (UTC)[reply]
It's more than that. "One electron up and one down" is the same quantum state as "one left and one right" or any other pair of opposite spin directions, so the electrons lose their identities in a more profound way than mere swappability. You don't have two electrons with opposite spins, you just have a compound state with zero spin (a singlet state) that doesn't divide into electrons any more uniquely than a two-liter bottle of soda divides into liters. Still, I think what DanielLC said was reasonable—you can have one electron over here and one over there and none in between, and though they are exchangeable in the wave function they aren't going to swap places with each other by crossing the intervening space. That's a kind of distinguishability, though it's not the technical sense in which the word is used in quantum mechanics. -- BenRG (talk) 21:53, 8 May 2009 (UTC)[reply]

Surface tension

What will be the Condition of spreading of one liquid over another and its expression ?Supriyochowdhury (talk) 06:51, 8 May 2009 (UTC) —Preceding unsigned comment added by Supriyochowdhury (talkcontribs) 06:50, 8 May 2009 (UTC)[reply]

I'm sorry, but I had trouble understanding your question. Could you try again with a little more detail? Thanks. --Sean 14:08, 8 May 2009 (UTC)[reply]
I think Supriyochowdhury is asking for a word for situations like having a layer of oil on top of water, and for the conditions that permit this to happen. I don't know the answers, though. Looie496 (talk) 15:58, 8 May 2009 (UTC)[reply]

== I want to about spreding coefficient $ba (where liquid b spread over liquid a)of a liquid and how can I derived.Supriyochowdhury (talk) 17:54, 8 May 2009 (UTC)[reply]

See spreading coefficient. --Heron (talk) 19:27, 8 May 2009 (UTC)[reply]

British bird identification/song question

There is a bird outside one of the places I reside frequently which makes a "doo-deladoo-delaadoo" (sorry for the crappy transcription) song over and over again, with the pitch going up and then it making a thrush sort of strangled noise, for long periods of time, most of the day. It will occassionally switch a bit, but this is what it does at least 90% of the time. It's been going on for a few months, at least, even into night sometimes. The place is a housing estate, but it backs out onto some wild scrubland/wooded area. First guess would be a blackbird but if so, why does it seemingly have such a restricted singing voice? Thanks for any answers 82.26.198.108 (talk) 07:38, 8 May 2009 (UTC)[reply]

Sounds a bit like a wood pigeon to me, check out the audio file in the article, it certainly has a very repetitive call. Mikenorton (talk) 07:58, 8 May 2009 (UTC)[reply]
Or try [2]. Bazza (talk) 13:29, 8 May 2009 (UTC)[reply]
Common Blackbird#Behaviour mentions the fact that some of them imitate noises and other things they heard in their song. (OR: There used to be one near my aunt's home that did a bit of a Mozart tune followed by a motor saw.)71.236.24.129 (talk) 13:25, 9 May 2009 (UTC)[reply]
It probably is a blackbird. Maybe it's just not very good at music. Nothing else sticks out of the songs I tried on the RSPB site and it's definitely not a wood pigeon. Thanks for the help everyone! 82.26.198.174 (talk) 19:15, 9 May 2009 (UTC)[reply]
Now that I read your description of the song as if sung by a blackbird, then it does sound like a blackbird. They have different calls - this is not their alarm call, but sounds like the call I recall hearing when all is peaceful. 78.146.17.231 (talk) 17:03, 16 May 2009 (UTC)[reply]

Identifying a tree

Can someone identify the species of the large tree growing directly in front of the building in this image? Thanks! LANTZYTALK 08:04, 8 May 2009 (UTC)[reply]

Podocarpus perhaps? Where was this photograph taken?CalamusFortis 15:32, 8 May 2009 (UTC)[reply]

Detective work says at Gujurat University in Ahmedabad, India. Looie496 (talk) 16:05, 8 May 2009 (UTC)[reply]
Podocarpus seems very plausible. Thanks! LANTZYTALK 17:16, 9 May 2009 (UTC)[reply]

Magnatite and hydrochloric acid

All of my searches to date indicate that magnatite dissolves slowly in hydrochloric acid, but no time frame. Does anyone now the time to dissolve, cold and warm —Preceding unsigned comment added by 86.137.11.144 (talk) 08:59, 8 May 2009 (UTC)[reply]

Speed of dissolving will depend on much more than the identity of the substances involved. For example, ground magnetite will dissolve much faster than a solid chunk, and a stirred solution will dissolve much faster than one left to sit idle. It will also depend on the concentration of the HCl, and as you thought, on the temperature. There are just far too many factors to give a definitive answer, variations of any one of these could change the speed of the reaction by orders of magnitude. --Jayron32.talk.contribs 00:05, 9 May 2009 (UTC)[reply]
From testing in lab: 3M HCL will dissolve magnetite powder almost instantly at RT, useful for cleaning. But 1M HCl has to be left at least a few hours, and more dilution solutions can take weeks, to the extent that it is no problem for washing to remove basic residues.YobMod 10:17, 14 May 2009 (UTC)[reply]

Random number

Can humans generate perfectly random numbers between any two arbitrary limits? - DSachan (talk) 09:43, 8 May 2009 (UTC)[reply]

Our page Random_number_generator_attack#Human_generation_of_random_quantities may help. 194.221.133.226 (talk) 10:19, 8 May 2009 (UTC)[reply]

It all comes down to what you define "random number" to be. --98.217.14.211 (talk) 10:23, 8 May 2009 (UTC)[reply]
(ec):No humans can't. It is well known that when humans are asked to pick numbers at random, they are strongly biased to particualr numbers, a fact often made use of by stage magicians. Computer algorithms for generating pseudo-random numbers are not truly random, but appear to be so for all practical purposes because the algorithm is seeded by a continuously changing number such as the time and date. One of the best hardware methods for producing random numbers is to amplify thermal noise. However, even this is not truly random since the signal must necessarily be band-limited which slightly biases against large differences in successive numbers over short intervals. There is also the question of what is meant by random. If the requirement is to build a machine that can output integers within a certain range with equal probability then a machine can be built to approximate to this. If the requirement is to output any real number randomly between two limits then no finite machine is capable of doing this. The reason is that there are infinitely many numbers between any two points. Any real (finite) machine will have a finite resolution and cannot possibly represent all of them, only a finite number of them. In fact, a digital machine is only able to represent rational numbers and these have a zero probability of occuring in a true random sequence of numbers even if the machine was capable of producing any of the infinite rational number between the stated limits. Why? because there are infinitely more irrational numbers in any interval than there are rational numbers, even though the number of both is infinite. SpinningSpark 10:30, 8 May 2009 (UTC)[reply]
When discussing "true random", an example I give that tends get the point across is that a true random number generator programmed to output true random numbers between 0 and 9 should have the possibility of outputting 1 1 1 1 1 1 1 1 1 1 1 ... The point is that it is truly random. Even if every number previously has been 1, the chance that the following number will be 1 is just as high as every other number. So, a generator that ensures the distribution of numbers is equal is not truly random. -- kainaw 13:24, 8 May 2009 (UTC)[reply]
That's true, but actual (computer) pseudorandom number generators don't try to even things out—they're just as likely to produce long sequences of ones as real random number generators are.
Computers aren't limited to rational numbers. They can work with numbers of the form where a and b are rational, for example. More interestingly they can represent the whole collection of computable reals. Computable real arithmetic systems work by successive approximation, and there's no reason you couldn't add genuine uncomputable random reals to such a system if you have a random oracle available in hardware. /dev/random is supposed to give you genuine quantum randomness, so it should work. -- BenRG (talk) 14:20, 8 May 2009 (UTC)[reply]
True, but the point stands - computable numbers are still only countable (the same size as the rationals), so you can still be almost certain that a number chosen at random from a bounded interval with a uniform distribution will not be computable. I don't see how you can add uncomputable numbers to a computer system, a computer can only work to finite precision. You might be able to fiddle around and extend the set of computable numbers a bit, but you won't stop it being countable. --Tango (talk) 14:27, 8 May 2009 (UTC)[reply]
Well, there's no real number that you can prove won't occur (exactly) in an exact-real-arithmetic system running on a machine with /dev/random. It may be philosophically dubious, but I don't think randomness gets any less dubious than that. Sure, a Turing machine with a given oracle can only be in countably many states and will only reach finitely many of those, but by the same token there are only countably many theorems about real numbers in ZFC and only finitely many of those will ever be written down. Only finitely many digits of pi will ever be known to the human race, every formal system has a countable model, etc. Of course, mathematicians never actually generate random numbers, they just reason about what would happen if they did, and computers can do that too. -- BenRG (talk) 20:33, 8 May 2009 (UTC)[reply]
Only because I wouldn't be able to describe such a number, but that doesn't mean much - almost all real numbers are undefinable. (There are describable numbers that aren't definable real numbers, but I believe the set of all describable numbers is still countable, by even the broadest definitions - I could be wrong, though.) --Tango (talk) 21:41, 8 May 2009 (UTC)[reply]
Getting off-topic here a little bit, but I kind of think this is important to repeat: There is no single, mathematically precise, notion of what it means for a real number to be definable. You have to say definable how. This has been the problem from the beginning at the definable real number article, and I have never been able to come up with a truly satisfactory way of making that article both correct and compliant with WP standards. I'm not sure it can be done. The option of campaigning for its deletion has certainly crossed my mind, but that won't make the issues go away; they'll just spread out through other articles, so I don't really think that's a good idea either. --Trovatore (talk) 03:52, 9 May 2009 (UTC)[reply]
Indeed, but I think any reasonable definition gives you only countably many numbers, so it doesn't really matter in this case. --Tango (talk) 13:21, 9 May 2009 (UTC)[reply]
It also depends on what you mean by "generate", as a human certainly can build a Geiger counter which s/he can then use to generate perfectly random numbers. A human can also shuffle a deck of cards pretty well, which can also generate good quality random numbers. --Sean 14:11, 8 May 2009 (UTC)[reply]
Geiger counters are not perfectly random in the mathematical sense and humans probably cannot shuffle as well as you think they can. SpinningSpark 19:42, 8 May 2009 (UTC)[reply]

When two waves intercept at a right angle

Do they interfere with each other? If yes, what types of waves interfere with each other?--Mr.K. (talk) 11:21, 8 May 2009 (UTC)[reply]

Yes, you will get additive interference just as any other case which combines waves. If you know the wavefunction for each wave, you can do a point-by-point combination of the phase for each wave, to determine the resulting total wave amplitude. The interference pattern will depend on whether you have two plane waves, wave-packets, or something else; in the case of two plane waves at right-angles, your interference pattern will be sort of "checkerboard-like" with constructive and destructive interference. Nimur (talk) 12:24, 8 May 2009 (UTC)[reply]
It depends what you mean by "interfere". There will be interference in the sense of transient superposition, as described by Nimur, but not necessarily interference in the sense of a permanent distortion. Gandalf61 (talk) 12:29, 8 May 2009 (UTC)[reply]
OK, if I put two lasers each one intercepting the other at 90 degrees. Will they affect each other in any way? (even if for practical purposes, it's irrelevant).--Mr.K. (talk) 17:11, 8 May 2009 (UTC)[reply]
Did you see our article on interference? They will affect one another, in the region of intersection; but if you were to measure/observe one of the beams outside the region of intersection, you wouldn't be able to tell that it had "crossed" with another beam. If you do the mathematics of adding the sine-waves and solve the wave equation, you will find this result is physically consistent. Nimur (talk) 19:42, 8 May 2009 (UTC)[reply]
In quantum electrodynamics there is an interaction between the beams, mediated by virtual electrons (the simplest Feynman diagram is this one). It's a small effect but it's large enough to be detectable. In principle there's a very tiny gravitational attraction too. Roughly speaking, though, they just pass through each other. -- BenRG (talk) 20:15, 8 May 2009 (UTC)[reply]
If they are in a non linear material you may get non linear optics effects with modulation harmonics or mixing occurring. Graeme Bartlett (talk) 09:02, 17 May 2009 (UTC)[reply]

youtube

How does this work

http://www.youtube.com/watch?v=e3kyNGVK-hI

Thanks--Mudupie (talk) 11:21, 8 May 2009 (UTC)[reply]

I've only watched the first minute or so, but it looks genuine to me. I imagine if one were to beat box into a flute (with sufficient skill) that is what it would sound like. --Tango (talk) 11:30, 8 May 2009 (UTC)[reply]
I agree, it's probably genuine. As for how it works, well the flute works by acoustic resonance, and the beat-boxing is sort of augmented by the percussive effect of the microphone amplifier clipping. Nimur (talk) 12:27, 8 May 2009 (UTC)[reply]

mWS in a mesurement of pressure drop

What´s means "mWS" in a mesumement of pressure drop or in a indication of pressure drop of a equipment? —Preceding unsigned comment added by Gigobqto (talkcontribs) 15:31, 8 May 2009 (UTC)[reply]

It is metres of water (approx 104 Pa, see Conversion of units#Pressure or mechanical stress) but I do not know what the "S" stands for. In the unit mW(g), the g stands for guage pressure, meaning the pressure is metres of water relative to atmospheric pressure, so I would surmise that mWS is measured relative to something else. Perhaps vacuum or negative guage pressure? SpinningSpark 16:19, 8 May 2009 (UTC)[reply]
Ah found it on German Wikipedia, it's metres Wassersäule which if my useless technical German is not letting me down means column of water. It seems to be used exclusively for pressure differences, such as the pressure drop along a supply pipe, and is distinguished from both absolute pressure and guage pressure. SpinningSpark 17:33, 8 May 2009 (UTC)[reply]

buffer solution

Hi I have been asked to write a symbol balanced equation for equilibrium of a buffer solution of 0.6 mol of propanoic acid and 0.8 mol of sodium propanoate. Would the answer be: CH3CH2COOH (reversable arrows) CH3CH2COO- + H+? I'm guessing its not CH3CH2COO-,Na+ (reversable arrows) CH3CH2COO-+Na+ because the next question is asking for a balanced symbol equation for the dissociation of sodium propanoate. Please help! —Preceding unsigned comment added by 92.18.81.46 (talk) 15:39, 8 May 2009 (UTC)[reply]

Looks good to me! Not that you mentioned it, but you may find Buffer solution and Henderson–Hasselbalch equation to be relevent reading. --Jayron32.talk.contribs 20:37, 8 May 2009 (UTC)[reply]
So. Balanced symbol equation. What's that then? Even google thinks we might not know. --Tagishsimon (talk) 00:10, 9 May 2009 (UTC)[reply]

Speed of magnetic and electric fields

What are the speeds of the magnetic field and electric field? Can we treat their speeds like that of an electromagnetic wave? --Email4mobile (talk) 17:27, 8 May 2009 (UTC)[reply]

Generally speaking electric and magnetic fields are not going anywhere at all unless the source of the field is itself moving. What does move is a disturbance in the field and such disturbances are called electromagnetic waves. The speed of these disturbances is c, the speed of light. Also note that there are not really separate electric and magnetic fields, but only the electromagnetic field, and the two are united through the Lorentz transformation. SpinningSpark 17:56, 8 May 2009 (UTC)[reply]

Thanks S Spinningspark for this explanation, but how can I describe the attraction and repple forces without accepting that these fields must move like radiation. For example If I've a giant magnet and another giant magnet was passing by very fast causing the nearest distance between them to be let's say 300,000 km. Will the maximum attraction|repple force take place at that position or after the magnet has passed by another distance proportional to its velocity? —Preceding unsigned comment added by Email4mobile (talkcontribs) 18:27, 8 May 2009 (UTC)[reply]

First of all, it is wrong to think of the magnetic field being solidly attached to the magnet and moving with it through space. Rather, the magnet is causing causing a disturbance in the field as it moves through it. That disturbance is propagated through the field and finally arrives at the second magnet. As for where the moving magnet is when the maximum effect is felt at the stationary magnet: ask yourself this, if instead of a moving magnet we have a moving star, where is the star when the maximum light from it is seen at the fixed position? SpinningSpark 19:10, 8 May 2009 (UTC)[reply]
It works surprisingly well to think of the field as being solidly attached to the magnet. When you push on a solid object the rest of it doesn't move instantly, there's a ripple effect through the object at the speed of sound. The magnetic field acts in a similar way, with the radiation being the ripple. When you stop pushing, the object eventually returns to its original shape, and the same happens to the field. I think possibly you (the original poster) are wondering how much the magnet's field lags behind the magnet when the magnet is moving. The (somewhat surprising) answer is that it doesn't lag at all, as long as the speed is constant. The maximum force between the magnets will be at the point of closest approach (which is the same from the rest frame of either magnet). The star is different—the maximum intensity of light will be somewhat before the time of closest approach. -- BenRG (talk) 21:19, 8 May 2009 (UTC)[reply]

Thank you BenRG and Spinningspark. Indeed the reason, I raised that question was that I was wondering why a conductor has to cut the magnetic lines in order for the e.m.f to be produced. If we think about massive bodies and the fields we will find that massive bodies have a relative motion (example rotating Earth and its movement around the Sun and the Galaxy, meanwhile the fields must have an absolute motion?. If so then why should the motion or disturbance be created in the conductor for example manually to get this emf?--Email4mobile (talk) 23:24, 8 May 2009 (UTC)[reply]

Carbon Dioxide and Dry Ice

Is carbon dioxide inert and is Carbon Dioxide in its solid form, Dry Ice inert? THX —Preceding unsigned comment added by Jzeilhofer (talkcontribs) 19:03, 8 May 2009 (UTC)[reply]

"Inert" is always relative:) Carbon dioxide is a common material used to extinguish fires (the fire extinguishers with a black plastic cone-shaped nozzle, for example), and they produce both the gas and solid forms. But burning magnesium reacts (quite spectacularly sometimes) with it. It dissolves in water (carbonated beverages) and dissolves in and/or binds to many biological components (carried by blood). And plants consume and chemically react CO2 (photosynthesis). Solid vs liquid is still the same chemical, just colder and "more of it" present in the same measured volume. So dry ice reacts more slowly (many reactions go slower at lower temperature) but for reactions that do occur. DMacks (talk) 19:10, 8 May 2009 (UTC)[reply]
Definitely not inert by a simple chemistry perspective, in which Oxygen is reactive and Nitrogen is inert (even if many reactions of N2 are known, they are relatively few). User:DMacks mentions Mg, but CO2 should never be used on any metal fires, due to its reactivity. It also reacts with water, forming carbonic acid, and is involved in many organic chemistry reactions (carboxylations). It will also act as a ligand in metal complexes, and can be reduced. In most lab conditions, it sublimes quickly, so while the solid form is easier to store, in terms of (non-cryonic) chemistry it is equally reactive. YobMod 11:45, 14 May 2009 (UTC)[reply]

Aluminium in tap water, tea, vegetables, and foods?

How much of the aluminium/aluminum in tea is due to the aluminium in the tap water from which it is made? And are there any vegetables which have significantly more aluminium in them than other vegetables? What about other foods? 89.240.209.79 (talk) 21:21, 8 May 2009 (UTC)[reply]

I'm not sure of tea but this site (http://www.eatwell.gov.uk/healthissues/factsbehindissues/aluminium/) has some info - it seems to suggest that it is from the water and soil. On the basis of water-based i'd guess that more water-y veg (e.g. Cucumber) would have more aluminium than 'drier' veg. ny156uk (talk) 21:30, 8 May 2009 (UTC)[reply]

May 9

Cockatiel Blues

What do you guys think of this? Is this Cocktatiel really performing a spontaneous blues improvisation? --Kurt Shaped Box (talk) 00:29, 9 May 2009 (UTC)[reply]

No doubt the bird has heard that tune played 546 times and learned to imitate it. Along the same lines, have you seen Snowball (Cockatoo)? Looie496 (talk) 02:17, 9 May 2009 (UTC)[reply]
Yep, I'm certainly aware of Snowball and others like him. I actually started that article... ;) It seems that parrots really do have some appreciation of human music... --Kurt Shaped Box (talk) 16:20, 10 May 2009 (UTC)[reply]

Thought Experiment: Perpetual Energy

Of all the systems in a car, I believe the brakes are the most amazing. They will reliaby work just the same with no gas and a dead battery. With a minor press on a pedal, 2000 lbs of automobile will come careening to a dead stop from 100 mph in just a few dozen feet. I had no idea my foot was that strong!

I understand that liquids can't be compressed and that is the "magic" of hydraulics. My question is: what would happen if that same effortless press on a pedal was used to power up turbines instead of stopping a giant machine barrelling at ungodly speeds? Surely the energy produced would be able to overtake the effortless pedal press with more to spare! Sappysap (talk) 01:55, 9 May 2009 (UTC)[reply]

You seem to be confused about either the first or the second law of thermodynamics. The first says that energy is conserved. The kinetic energy of the car doesn't just disappear, neither does it go into your foot. The brakes convert the kinetic energy into heat (for example by pressing a disk against the wheel and letting friction slow it down). See File:Ceramic brakes.jpg for a striking visual manifestation of this heat.
The second law says that entropy always increases, or in less precise and more practical terms, energy can't be converted back from heat. This is why it's impossible to make a brake go in reverse and spin up a turbine rather than slowing something down. Braking a car is an inherently irreversible process, because the entropy increases so much. —Keenan Pepper 02:55, 9 May 2009 (UTC)[reply]
And just so we're clear, hydraulic brakes aren't fundamentally different from any other brake system. Ultimately, they also convert kinetic energy into heat by means of friction. The hydraulics simply transfer the kinetic energy before it is turned into heat. —Keenan Pepper 03:00, 9 May 2009 (UTC)[reply]
(Edit conflict) You seem to have confused two separate sets of energy. Your "effortless press" uses a small amount of energy generated in the cells of your leg muscles. This merely moves the brake pedal, which through the medium of the hydraulics (perhaps supplemented by servo-motors powered by your car's engine or battery) moves parts of the mechanisms of the Disk brake on each braked wheel, pressing the brake pads against the brake disks. The energy required is small, and is independent of whether your car is moving or not.
Your moving car, as you are aware, possesses a large quantity of kinetic energy which has been converted from the stored chemical energy of its fuel by its engine. By your lightly pressing the brake pads against the wheels, this kinetic energy is converted by friction into heat: no additional energy is generated by your operation of the brakes. This is not the same as somehow exerting a new large force opposite to the car's motion in order to stop it, which would indeed require an amount of new energy comparable to the car's kinetic energy (though somewhat less as road friction, internal mechanical friction and air drag are already helping to oppose the car's motion). Ask yourself, since energy can neither be created nor destroyed, only converted from one form to another, where would this new energy have come from?
This form of braking is called Dynamic braking, and the dissipated heat energy (which started as chemical energy in the fuel you paid for) is lost/wasted. In some recent vehicles, some of the energy converted by the brakes in order to slow the car is not dissipated as heat, but converted into a storable form, usually mechanical (for example by spinning up a flywheel) or electrical (in the existing or a supplementary battery), for re-use. This type of brake is called a Regenerative brake, but it too is not somehow creating additional energy, merely saving some of the energy that would otherwise have been lost.
My explanation has been broadly conceptual, but I expect another responder will set out the relevant physical equations in mathematical form, if that will help. 87.81.230.195 (talk) 03:12, 9 May 2009 (UTC)[reply]
The reason you can produce so much force in the brake pads with a fairly light touch of your foot is because your foot moves downwards several inches - but the pads only have to move inwards by a tiny fraction of an inch. That provides the 'mechanical advantage' you need to exert that kind of force...kinda like leverage...but with a liquid. Of course, many cars also have power-assisted brakes - and in that case, power (sometimes from the vacuum system of the engine, sometimes via a hydraulic pump attached driven from the serpentine belt - but increasingly, electrical) is applied to help your foot do the work.
As others have pointed out, the energy from the cars motion gets converted into heat and is dissipated by the disk (or drum) in the braking system. After a lot of heavy braking, the disks can get so hot they they'll actually glow - and if they get too hot, the hydraulic brake fluid might boil - this is called "brake fade" and it's exceedingly dangerous because the gasses produced by the boiling brake fluid is easily compressible (unlike the liquid which is impossible to compress) - so if the brakes get too hot, when you push down on the pedal, all that happens is that the gasses compress, the brake pads stop moving and quite suddenly you have no brakes!
That's why on long hills, you should slow down using the engine (down-shifting the gearbox) rather than using the brakes for large amounts of time. If you do have brake fade, then in an emergency, you can gently apply the parking brake - which is typically connected using a cable system that won't suffer from the heating problems. But you have to be pretty careful because you can easily lock up the wheels and put the car into a skid. SteveBaker (talk) 03:30, 9 May 2009 (UTC)[reply]

will ... what you are'nt understanding is that work is constant , so that brakes just applied the work you've done in avery

effective way on the wheels , its just like hydrolic machines , belozers , were the engine do more than 3000 rpm will its juut

make the vhicle to left the bocket up less than a meter . its just transfere like 40000 rpm to left the bocket up ward like

a meter. thats it .

you can explain it by imagining the multi speed bicycle , in some speeds you have to cycle you legs two times to revolve the

wheel once , which is easy ... in other cases you need to cycle one time to revolve the wheel 5 times which is too hard ,its

need mush bower than your legs can produce .--Mjaafreh2008 (talk) 10:52, 9 May 2009 (UTC)[reply]

MSUD in Adults??

Can an adult have MSUD. Can it be contracted in teen or adult years. What else might be causing this Maple Syrup smell in my sons room and bathroom?? —Preceding unsigned comment added by 68.195.147.98 (talk) 03:03, 9 May 2009 (UTC)[reply]

Sorry, the Reference Desk doesn't give out medical advice. Please consult a doctor. Tempshill (talk) 03:12, 9 May 2009 (UTC)[reply]
The second half of the question is not a request for medical advice, though. There are many things that could be causing maple syrup smell in the room - maybe a stash of hidden pancakes? Nimur (talk) 04:32, 9 May 2009 (UTC)[reply]
I'm with Nimur on this one. The substance that is responsible for maple syrup odor - both in pancakes and in MSUD - is sotolone, and, naturally, we have an article about it. --Dr Dima (talk) 07:48, 9 May 2009 (UTC)[reply]

Earthquakes

Is it possible to find the centre of an earthquake? —Preceding unsigned comment added by 174.6.144.211 (talk) 03:14, 9 May 2009 (UTC)[reply]

Yes - they can measure the arrival time of the seismic waves at remote seismology stations and by comparing those times, figure out where the earthquake was happening. Those shock waves move pretty fast but with accurate clocks, it can be done to a reasonable precision. SteveBaker (talk) 03:17, 9 May 2009 (UTC)[reply]
Well, to the extent that there is a well-defined "center". Earthquakes, especially big ones, don't happen at a single point. There's a section of the fault that ruptures. So it's more like it happens along a line (in the non-mathematician's use of the word line — for mathematicians that would be a one-dimensional manifold, possibly with boundary :-). --Trovatore (talk) 03:41, 9 May 2009 (UTC)[reply]
Actually, earthquakes happen (mostly) along a surface, (a two-dimensional manifold, possibly with boundaries.) And the motions vary with time, so we need to ass a third dimension. A classic example would be an earthquake that progresses along a fault. -Arch dude (talk) 07:34, 9 May 2009 (UTC)[reply]
Also see Epicenter. 71.236.24.129 (talk) 08:59, 9 May 2009 (UTC)[reply]
The epicentre, as mentioned above which is where the rupture begins. It then propagates within the fault surface until it runs out of stored elastic strain energy. In many cases the epicentre is distinctly offcentre for the rupture e.g. both the 2004 Indian Ocean earthquake and the 2008 Sichuan earthquake initiated right at one end of the fault segment that moved. We know this because of the distribution of aftershocks, which define pretty exactly the extent of the rupture. In terms of the felt intensity the area with greatest damage is often near the epicentre and for historic earthquakes, that happened before instrumental recordings, this is how the epicentre is estimated. In some earthquakes. however, such as the 2002 Denali earthquake the maximum intensity was felt at the other end of the fault surface that ruptured from the epicentre. That earthquake also illustrates another possible complexity, it began on a thrust fault jumped to the strike-slip Denali Fault before jumping again onto the Totschunda fault, another strike-slip structure. Finally, most earthquakes hypocentres (the actual point of initiation) occur within a small range of depths, normally 10-15 km, known as the seismogenic layer. This is the strongest part of the crust, as increased confining pressure makes fracturing progressively more difficult until the temperature rises sufficiently for ductile processes to become important. Most of the rupture propagation, therefore, occurs either laterally or upwards, so if you looked at the area that slipped, the epicentre is often nowhere near the centre of the rupture at all. Mikenorton (talk) 12:29, 9 May 2009 (UTC)[reply]

Are convergent, divergent, collisional, and transform boundaries fault lines? —Preceding unsigned comment added by 174.6.144.211 (talk) 04:20, 10 May 2009 (UTC)[reply]

All types of plate boundaries have faults associated with them, but only for some of them is there a single fault surface along the boundary itself. Subduction zones pretty much act as simple fault surfaces, as do transforms in oceanic crust. Some continental transforms such as the Alpine Fault in New Zealand, form single fault surfaces along the boundary but others, such as the San Andreas Fault form part of a deformed zone within which there are many active faults. Divergent boundaries in continental crust form rifts with many active normal faults, such as the East African Rift. Collisional zones such as the Himalayas have huge zones of deformation, sometimes 100s of km across, within which there are many seismically active fault zones. Mikenorton (talk) 11:51, 10 May 2009 (UTC)[reply]

I know that it is possible to calculate the depth of a hypocenter by using p waves, but is it possible to calculate the location of a hypocenter? —Preceding unsigned comment added by 174.6.144.211 (talk) 23:31, 10 May 2009 (UTC)[reply]

Electric circuit for highpower sawtooth wave output.

I need a sawtooth wave generator and i've searched the internet for it; but all the circuit i found have very low current output.The output i need is shown in fig.

File:Output Sawtooth.JPG

Where can i find my desired circuit diagram? —Preceding unsigned comment added by Shamiul (talkcontribs) 04:06, 9 May 2009 (UTC)[reply]

You're going to need a pretty hefty amplifier. Have you considered a PA system connected to a signal generator? You could take the output off the speaker-lines and connect it to whatever your load is. My back-of-the-envelope calculations are suggesting that this is going to need an average power output of around 600 watts, which is awfully big for an audio system. Are you really sure you need 25 amps at 40 volts? Nimur (talk) 04:30, 9 May 2009 (UTC)[reply]

Thank you for your answer, but i don't mean amplifier. The main circuit should deliver the output required that can be directly delivered to the load. I want to know the procedure that you used to calculate the power output, in case if i had made mistakes. —Preceding unsigned comment added by 123.49.45.67 (talk) 04:57, 9 May 2009 (UTC)[reply]

Nimur appears to have arrived at the power requirement by a standard RMS integration of the voltage waveform multiplied by 25 amps. However, in his audio amp solution the 5 volt offset would have to be added back after the amp because audio amps do not usually pass dc, the required power rating of the amp is therefore a little less. You do not say why this is not an acceptable solution, and without details of your application it is difficult to give you precise help. Possibly bandwidth is a concern, 10kHz would certainly go through a high quality audio amp but the harmonics are likely to get mangled, badly distorting the waveform from linear. There are certainly plenty of audio amps out there in the right power range at affordable prices. In any case, any solution is liable to be a low power waveform generator stage plus a power amplifier stage as Nimur says, although the power stage might be just a power transistor working off a 40 volt rail if your waveform generator has an open collector output. I think you will struggle to find a ready made circuit diagram to do this - high voltage ramp circuits were common in the days of crt televisions, but I can't think offhand of many high current applications. So to give you some specific suggestions: for the waveform generator there is the 555 timer which has been around since the stone age and the internet is littered with circuit diagrams and application notes. Here is a page describing some alternative waveform generator ICs. If you need great precision of the waveform then you might want to consider a synthesised arbitrary waveform generator which are available as modules/pcbs from some manufactures. For the output transistor, there are many available with the required current and voltage rating. SpinningSpark 10:37, 9 May 2009 (UTC)[reply]
Yes, that's how I got it... I think the original questioner should really provide context. When you say you don't want an amplifier, I'm a little worried - because any real circuit which will generate such high voltages and currents IS an amplifier. If you're a novice electronics enthusiast, you should not be playing with kilowatt-scale, high-amperage systems, because they can kill you. (Let's be clear here - 40 volts and 25 amps is deadly). What are you trying to do? With some context we can give you a better and more realistic answer. Nimur (talk) 15:05, 9 May 2009 (UTC)[reply]
By the way, when you asked the same question last year your image was deleted for the same reason this one's going to be if you don't put a licence tag on it soon. SpinningSpark 20:35, 9 May 2009 (UTC)[reply]
As a retro alternative circuit, how about a 5 volt DC generator in series with a DC generator, the field current of which is regulated by a motor driven rheostat such that it sweeps through the required output voltage? A 600 watt generator is not all that large or expensive. Edison (talk) 01:40, 10 May 2009 (UTC)[reply]
That wouldn't be very energy-efficient! Half the power would be burned over the rheostat! You really shouldn't be encouraging resistive-loss as an effective method for generating a desired voltage. This is especially bad if you're really going to pump the output signal at 25 amps through it! That sucker's going to need a heat sink! Nimur (talk) 16:15, 10 May 2009 (UTC)[reply]
Is your first name Heath? SpinningSpark 02:15, 10 May 2009 (UTC)[reply]

Downshifting vs. braking

Inspired by a SteveBaker answer above: Is it really better to downshift than brake when going down a hill? Won't this cause excess wear on the transmission or gearbox or some other part of the car that, unlike brakes, was not designed to be worn so much? Tempshill (talk) 04:10, 9 May 2009 (UTC)[reply]

Hmm? I don't see where there's any extra stress on the drivetrain- it's the same as what's going on most of the time while you're driving. Brakes slow you down by friction- the pads wear out as you use them. And they're emphatically not meant to be used constantly for long periods of time, because they'll heat up and fail (see brake fade). The drivetrain is meant to be turning pretty much all the time. Friday (talk) 04:15, 9 May 2009 (UTC)[reply]
When I lived in Colorado and Montana I did a lot of mountain driving. I could always tell who the flat-landers were by the excessive amount of brake lights they displayed on the down grade. --jwalling (talk) 04:41, 9 May 2009 (UTC)[reply]
In Colorado and Montana you'd also likely have encountered quite a few people driving cars with automatic transmissions. They are a lot more common here than in Europe. 71.236.24.129 (talk) 08:51, 9 May 2009 (UTC)[reply]
Not exactly the same, but maybe you're interested in jake brake. Shadowjams (talk) 09:02, 9 May 2009 (UTC)[reply]

i think gears were'nt designed to slow the vhicle , because at high speeds its not right to shift back from the fourth to the

first , and in some cases it could break the sustem . i think its better to use brakes instead except when its dangrous to do

so at high speeds or at mountain roads were such action could cause the car to go off the road .--Mjaafreh2008 (talk) 09:49, 9 May 2009 (UTC)[reply]

I live in a mountainous country, where we are taught to always downshift when driving downhill. The article Måbødalen disaster may be of interest. It was in part caused by the bus driver's lack of experience in mountain driving. --NorwegianBlue talk 11:40, 9 May 2009 (UTC)[reply]
But you aren't trying to slow the vehicle, you're trying to prevent the vehicle speeding up. That's a very different thing. If you want to slow down, use the brakes, if you want to maintain a constant speed, put the car in the right gear for that speed (which will be a lower gear when going downhill than when on the level). --Tango (talk) 13:27, 9 May 2009 (UTC)[reply]
Let's be PERFECTLY clear about this. For routine slowing down - brief dabs on the brakes separated by minutes of time, typical in-town driving - then you most certainly should use the brakes. That's what they are there for. However, if you are going to have your foot on and off the brake - or lightly pressing the brake for (let's say) a minute or more - then you're driving extremely dangerously and sooner or later you'll boil your brake fluid and you could very easily DIE as a result of your failure to understand the principles of controlling your vehicle. This most often happens on long downhill sections (but it can happen if you're driving agressively on the freeway at high speeds). In these circumstances, you MUST use the engine to slow yourself down. On a stick-shift, you need to shift down progressively (ignoring the horrific racing noises from the engine!) until your speed is under control - where you want it to be. The engine and transmission most certainly are designed to be able to do this - you aren't damaging it any more than when you stamp on the gas pedal to accelerate away from a stop. On an automatic, taking your foot right off of the gas pedal may be enough - but if you're still accelerating then you need to use the brakes one time to get your speed down - and then put the transmission into '2' or even '1' for the remainder of the hill. People who worry that they are ruining their transmission or engine need to understand that they are ruining their brakes by NOT doing this - heating their brake disks/drums until they are literally glowing red hot is going to cause them to warp and to wear the brake pads prematurely. If you do experience brake fade, the seals on brake slave cylinder can be ruined and you can get all sorts of crud in your brake lines. I can't overstress the importance of this. It's one of the commonest mistakes people make when they aren't used to driving in hilly terrain. Failure to understand how brake fade comes about kills a good number of people every year - you should have paid more attention during driver's ed. classes! SteveBaker (talk) 14:13, 9 May 2009 (UTC)[reply]
Well, this is a bit exaggerated. It's pretty hard to lose your brakes in a modern, ordinary-sized passenger car, unless they are in bad shape to start with. The danger is only serious for heavy vehicles like trucks or buses, or if you do something really stupid like keeping your foot on the brake while pressing the accelerator. It's definitely better to use engine-braking as much as possible, but you don't have to be paranoid about it. I've numerous times descended steep switchbacky mountain roads where I had to brake the whole way down for thousands of feet -- being in low gear helped of course but only to a limited degree. Looie496 (talk) 17:21, 9 May 2009 (UTC)[reply]
[citation needed]? Please quote sources, and re-read Steve's response and the article I linked to. --NorwegianBlue talk 19:11, 9 May 2009 (UTC)[reply]
If you were in first and still going too fast then, of course, you have to use the brake, you have no choice, but you were still risking your brakes overheating. --Tango (talk) 20:19, 9 May 2009 (UTC)[reply]
You sure have a choice. Try shifting to reverse. --NorwegianBlue talk 20:36, 9 May 2009 (UTC)[reply]
Ok, you don't have a good choice! --Tango (talk) 21:50, 9 May 2009 (UTC) [reply]
I think we are confusing two different issues: The use of either brakes or the transmission to slow down the car versus the use of either brakes or the transmission to maintain speed on a hill. The brakes should always be used to slow the car down, but the transmission should be used to maintain the proper speed when going down hill. That's because brakes are not designed to be applied for long periods of time, but in short bursts. --Jayron32.talk.contribs 04:36, 10 May 2009 (UTC)[reply]
Why do you advise against taking advantage of the engine's ability of slowing down the vehicle, say, when leaving a motorway? Used with proper technique, you can drive very smoothly that way. Combined, of course, with using the brakes. --NorwegianBlue talk 10:44, 10 May 2009 (UTC)[reply]
Generally - because brake pads are cheaper to replace than clutches, you should use the brake for short decelerations. But if you aren't concerned about wear and tear, then either approach works and is safe for brief periods. I confess that I use engine braking more than I probably ought. The big issue is only with prolonged use of the brakes - which is just plain dangerous. SteveBaker (talk) 14:20, 10 May 2009 (UTC)[reply]

Climate change vs. Global warming

Are there simple guidelines to determine which term should be used in a discussion. In my mind, Global warming is a major cause for the effects of Climate change on regional scales. It's my impression that 'climate change' is used often when 'global warming' would be more accurate. --jwalling (talk) 04:27, 9 May 2009 (UTC)[reply]

Global warming is one form of climate change (i.e a subset). It's also possible to have global cooling and other forms of climate change. We are currently experiencing massive human induced climate change in the form of global warming which is expect to get worse but obviously when you refer to global warming it doesn't have to refer to this particular instance nor does climate change have to mean global warming. Nil Einne (talk) 04:37, 9 May 2009 (UTC)[reply]
The ambiguity of 'climate change' (warming or cooling) supports my view that 'global warming' is more accurate when discussing the current global climate. 'Climate change' is appropriate for an unspecified region or uncertain period of time, but if you do specify a region and a time period which is showing warming, such as the Arctic for the next 10 years, then 'Climate warming' is more accurate. -- jwalling (talk) 05:35, 9 May 2009 (UTC)[reply]
Well firstly I'm presuming you've already specified on the planet earth and over the past 100 or so years and the next 100 or so years. Otherwise it's still confusing (are we referring to global warming on Venus? 1 billion years ago? 1 billion years from now). But even then arguably anthropogenic global warming (a favourite of denialists) or human-induced global warming is more accurate (or more correctly more precise since global warming is by definition a form of climate change so it's very rare you can be wrong if you specify climate change instead of global warming, it's just that you are not being precise as to what form of climate change you are talking about) since it is what we are observing (not say global warming due to sunspots). If you want to go further, perhaps global warming due primarily to an increase in global greenhouse gas levels as a result of human activities. Even further global warming due primarily to an increase in global greenhouse gas levels as a result of human activities including extensive use of fossil fuels, large scale changes in land use...... I think to some extent both are precise enough. There is only one form of massive global climate change we are currently observing and expect as far as I'm aware and that is global warming. So if you want to talk about climate change from a global sense in the next 100 years then of course that will primarily be about global warming not global cooling or anything else. It also depends on what you're talking about. For example the scientific opinion on climate change is about all climate change we expect in coming years and as I've mentioned this is global warming. Similarly the Intergovernmental Panel on Climate Change is supposed to study all climate change and of course global warming is of primary interest to them. In other words, IMHO it's more a matter of semantics and trying to argue which one is more accurate or perhaps IMHO since as I've already stated more accurate is a bit of a misnomer, which term is better applied to a given situation is mostly a pointless waste of time outside of times where it matters e.g. the naming of wikipedia articles, it's better to concentrate on more important things liking convincing those who are fooled by the denialists that there isn't one big conspiracy and climate change in the form of global warming is happening and is going to cause major problems and yes it is caused primarily by human activity. Nil Einne (talk) 08:46, 9 May 2009 (UTC)[reply]
The problem with the terminology is that the increase in greenhouse gasses is causing global warming - but also other kinds of climate change (increased hurricane strength and/or frequency, rising sea levels, changes in drought patterns, etc). So the term "Climate Change" better suits the whole range of problems that are being caused. However, some of these effects are secondary effects of global warming - without global warming, most (if not all) of the other effects wouldn't be happening. However, there is at least ONE other effect that we can describe as "Climate Change" which is totally unrelated to global warming or the greenhouse effect - and that is the destruction of the ozone layer. That was not caused by greenhouse gasses like CO2 and Methane - but instead by Chloroflourocarbons released into the atmosphere. Destruction of the ozone layer causes increases in ultraviolet radiation - which is harmful in all sorts of ways. Worrying about the ozone layer has become less trendy than concern over the greenhouse effect - but it's also a major problem. I think it's reasonable to say then that the term "Climate change" is more encompassing than "Global warming" which is in turn more encompassing than "The Greenhouse effect". So, although I confess to tending to use the terms interchangeably - if we are being super-careful:
  • If you are talking about CO2, Methane and other gasses that we're dumping into the upper atmosphere - then you need to talk about "The Greenhouse Effect".
  • If you are talking about organohalons, chloroflourocarbons, nitric oxide and related gasses causing destruction of ozone - then "Ozone layer depletion" is the correct term.
  • If you are talking about gasses dumped into the lower atmosphere that cause plant destruction and soil pollution as they are washed out of the atmosphere by rain - then you need to talk about "Acid Rain".
  • Any or all of the previous three terms could be described as "Atmospheric pollution".
  • If you are talking just about the rise in globally and seasonally averaged air temperatures - then you should say "Global Warming" - because there are other causes (and mitigating factors) for that beyond the greenhouse effect - things such as the decrease in the planets' albedo due to melting ice caps and glaciers and the increase in albedo due to the contrails from high flying jet aircraft.
  • If you are talking about all of the consequences of all of the things we're doing to the planet's atmosphere and hydrosphere - then "Climate Change" is a more appropriate term - because there are other causes of that - such as changes in water usage patterns and evaporation rates, destruction of the Ozone layer, the effect of large cities creating local hot-spots, etc.
  • If your concerns range beyond that, then "Oh Shit!" may be the term of last resort!
SteveBaker (talk) 13:45, 9 May 2009 (UTC)[reply]

marine diesel engine

working of air distributar Bose09 (talk) 07:10, 9 May 2009 (UTC)[reply]

Diesel engine has some information under medium-speed engines and low-speed engines but it is a bit scanty I'm afraid. SpinningSpark 11:08, 9 May 2009 (UTC)[reply]

Kc calculation

"The equilibrium constant, Kc, for the reaction to form ethyl ethanoate from ethanol and ethanoic acid, C2H5OH + CH3CO2H=CH3CO2C2H5 + H2O, at 60 degree Celsius is 4.00. When 1.00 mol each of ethanol and ethanoic acid are allowed to reach equilibrium at 60 degree Celsius, what is the number of moles of ethyl ethanoate formed?" This is the question I have to solve. I know you guys dont do homework questions and I dont want the answer to the question either. I just want to know how I am supposed to solve this question, i.e the working. Thanks. —Preceding unsigned comment added by 116.71.59.203 (talk) 08:38, 9 May 2009 (UTC)[reply]

Use the first formula given here. yandman 11:50, 9 May 2009 (UTC)[reply]

But the question doesn't give the amount of water produced. I cant use that formula. —Preceding unsigned comment added by 116.71.59.203 (talk) 11:57, 9 May 2009 (UTC)[reply]

You don't need to be given that...you need to figure it out (at least algebraically) from the info that is given. You're not told "how much ethanol is present at equilibrium" but "how much ethanol is starting before equilibrium has occurred". The amount of reactants are decreased by the identical (considering a stoichiometry of a balanced reaction) that the amount of products increases. I am sure you did examples like this in class or in your textbook. DMacks (talk) 17:37, 9 May 2009 (UTC)[reply]

So by "figuring it out", Id say that the amount of water is also 1 mole? I haven't really understood this part. —Preceding unsigned comment added by 116.71.33.115 (talk) 05:10, 10 May 2009 (UTC)[reply]

Maybe and maybe not. You start with 1 mole of starting materials and 0 mole of products (per question as written). As you can see in the reaction, every bit of water that is formed comes from ethanol, so if the full 1 mole of starting material is consumed, you get 1 mole of product, and that would leave leave you with 0 mole of starting material. The whole idea of equlibrium and Kc is that the reaction doesn't run all the way (which would consume all of the starting material) but only runs partway. "Some" starting material gets converted to product, "the rest" remains as starting material. And Kc defines exactly the ratio of those two quantities. You know the total (perhaps SM+P?) and the ratio (formula for Kc and its given value), now solve for the variables. DMacks (talk) 08:07, 10 May 2009 (UTC)[reply]

teeth

I have a very weak teeth , i visit alot of dentist by they were'nt alot of help

now after all that advance in evry field , i find it wierd that they had'nt find

asolution for cavity , its brrety simple .

some kind of amouth wash that will produce a thin film at the surface of the tooth

so it wont be in contact with food left , so no cavity will form .

realy i dont think its that hard , we built a space ship , rockets , submarines , airplanes

i think its aprrety easy task .

i said that because i visit the dentist so much , and its frightning each ti.

--Mjaafreh2008 (talk) 09:37, 9 May 2009 (UTC)[reply]

Okay. Do you have a question? -- Captain Disdain (talk) 10:26, 9 May 2009 (UTC)[reply]


Answers removed, after discussion on the talk page.

Mjaafreh2008, as Captain Disdain pointed out, you aren't really asking a question. Please rephrase your question, keeping in mind that we cannot give medical advice. --NorwegianBlue talk 16:55, 11 May 2009 (UTC)[reply]

Class of drug

Is Clopidogrel an Anticoagulant? If not what's the difference? (No medical advice.)71.236.24.129 (talk) 11:02, 9 May 2009 (UTC)[reply]

In the ATC classification system, clopidogrel has the ATC number B01A C04, and is classified as an antithrombotic agent. This ATC group has its own Wikipedia page: ATC code B01. If you check it out, you'll find that a traditional anticoagulant such as Warfarin is in the same group. The difference is that clopidogrel inibits platelet aggregation, whereas anticoagulants inhibit the proteins necessary for coagulation. --NorwegianBlue talk 11:25, 9 May 2009 (UTC)[reply]
(After EC) The usual definition of an anticoagulant is a substance that stops the blood from clotting. Anticoagulants like heparin or warfarin directly affect the clotting factors and prevent the entire process from getting started. Clopidogrel is an antiplatelet drug, which prevents platelet activation, the process by which platelets form a kind of plug to stop bleeding. The drug therefore prevents aggregation of platelets but does not interfere with the rest of the clotting cascade. It may seem a subtle distinction since the end result of both classes of drugs is to interfere with the formation of blood clots, but there are differences in mechanism and what the various drugs are used for, so it's a meaningful separation. --- Medical geneticist (talk) 11:38, 9 May 2009 (UTC)[reply]
A difference that also may be of interest is in their indications: clopidogrel is typically used to prevent arterial thrombosis after medical procedures such as the insertion of a coronary stent, anticoagulants are typically used to prevent venous thrombosis. --NorwegianBlue talk 12:00, 9 May 2009 (UTC)[reply]
Thanks. It's a bit clearer now. 71.236.24.129 (talk) 14:01, 9 May 2009 (UTC)[reply]

How a factory works

Is there any book/package/work/whatever that will teach you exactly how a factory works? Or if you want to create a factory you must pick up the pieces from here and there? --Mr.K. (talk) 11:53, 9 May 2009 (UTC)[reply]

Please clarify: are you asking about a factory or a Factory pattern? --NorwegianBlue talk 12:04, 9 May 2009 (UTC)[reply]
If it's the latter: A piece of software that "constructs class instances" (all three of those words having very specific meanings) according to some set of parameters - then you REALLY need to ask this question on the computing section of the ref desk.
If you're talking about an actual physical factory...then it's still not got much to do with science and you might get a better answer on the miscellaneous desk where there are more people to respond to the question.
SteveBaker (talk) 13:18, 9 May 2009 (UTC)[reply]
Every factory is different. Some are highly mechanized, and some are just big empty rooms where people work, sewing or bending sheet metal or some other process. If you have a question about a specific type of factory, we can answer that better. Maybe you would find the machining, assembly line, and manufacturing articles helpful. Nimur (talk) 13:39, 9 May 2009 (UTC)[reply]
Most factories are based on a Workflow analysis. So if you are thinking of a brick and mortar version that would be a good starting point. Apart from that we'd really have to know what type of factory you're interested in. There are significant differences between building a meat processing plant, a saw-mill or a coffee machine manufacturing plant (OR). Even restrictions on site selection (where you could/should build it) would be quite different. 71.236.24.129 (talk) 13:58, 9 May 2009 (UTC)[reply]

Yes, every factory is different. I just wanted to know if any of you have ever seen a book/work/or whatever that explained exactly how you could build your own factory of whatever. I don´t want you to describe how a factory works, I just need the bibliographic reference. --Mr.K. (talk) 17:49, 10 May 2009 (UTC)[reply]

When you say 'build', do you mean like construction of the building? Or like design of the inner mechanical workings? Or like making all the machine parts? Or building the workforce and management structure? Or what? Are you thinking start with a plot of greenfield land and end up with a working factory?
A warehouse with workers stood at tables could be a factory, and all you'd need to build that would be the construction skills to make a one-room building (and the workers and tables and supplies and know-how and...) Something to make pharmaceuticals would be very different, which in turn would be completely different from a factory for smallscale cake production which would, in turn, require quite different skills to building a facility to process volatile petrochemicals.
So, are you just looking for a book that describes how to build a simple building? Like, bricklaying skills and suchlike? O_o 80.41.104.79 (talk) 21:12, 10 May 2009 (UTC)[reply]

Net charge of earth

Is the net charge of the earth is +ve ? Why?? —Preceding unsigned comment added by 117.98.99.182 (talk) 13:24, 9 May 2009 (UTC)[reply]

I don't think so. I'm pretty sure that the Earth as a whole entity has neutral charge. There are locally charged regions (a lot of times, charge builds up in the atmosphere); and while we have a radiation belt full of charged particles, there are both electrons and positive ions to balance out. Take a look at this Discussion on the Earth's Net Electric Charge. In this published paper, a scientist from the Navy Research Lab states "As a consequence of this discussion, any statement made up to now on a net electric charge of the earth remains in doubt. It may be that the net electric charge of the earth (as globe or as planet) is zero or near zero - always or in the average - but we do not know a way to prove or disprove it." This paper is a pretty good introduction - it defines the problem well, mentions several experimental and theoretical approaches, and describes the various difficulties in all these methods. Nimur (talk) 14:04, 9 May 2009 (UTC)[reply]
Hi Nimur, I saw that paper but cannot read all of it. Does it debunk the common statement that the surface of the earth has a high negative charge (balanced by positive charge in the upper atmosphere/ionosphere)?. Or is it merely saying that past attempts to enumerate this charge precisely are misguided? SpinningSpark 14:25, 9 May 2009 (UTC)[reply]
The author refines the question by asking whether the question is "net charge on the earth, volumetrically", "net charge on the earth's surface, integrated over the entire surface," and "net charge on the surface in a local sense". Without doubt there is a net charge on the earth's surface in a local and transient sense, and this is generally believed to be nulled out by atmospheric electric charge, sometimes re-equalizing with lightning. The author discusses the "spherical capacitor" model, with a charged surface (lithosphere) and a charged ionosphere boundary, but investigates whether that can really apply to a surface that is 70% oceanic. Further, there are discussions about net current, vs. net charge, and whether one can exist without the other. There is discussion about Earth's space environment (being a conductive plasma, any net charge on Earth would be able to flow off into space and equalize to zero). In each of these scenarios, the author provides the essential basic-physics approximation, and attempts to derive some consequences, before presenting conflicting experimental evidence to suggest that the basic-approximations do not hold well. His concluding remarks are that there are no practical methods to measure a net-charge on Earth, because of the various confounding factors; and that the theoretical explanations give various answers ranging from "large positive charge", "neutral," "large negative charge", "dynamic and fluctuating with zero-mean," yet none of these theories are practical to verify experimentally. Nimur (talk) 14:43, 9 May 2009 (UTC)[reply]
In specific answer to your question, SpinningSpark, he does not "debunk" the common statement about charge-splitting between lithosphere and troposphere/ionosphere. (As I mentioned above, this explanation is widely understood as the basis for a lot of tropospheric weather, and has been clearly observed in a variety of experimental setups). However, he does make two counterpoints regarding the extension of this model to the entire globe. First, characterizing the magnitude is much more complicated than it might seem (my experience in this area strongly supports such a stance). Second, the capacitor-model of charge-splitting may not be applicable over the entire earth's surface, (certainly not "instantaneously", and probably not even in the time-average-sense). Nimur (talk) 14:46, 9 May 2009 (UTC)[reply]
A 'Science Advisor' on physicsforums.com says that Earth is "neutral to about one part in 10^26", but I can't see where he gets that figure from. As mentioned above, the Earth is bathed in a plasma that would conduct away any excess charge. Perhaps the next question should be "What is the net charge of the Solar System?" --Heron (talk) 17:55, 9 May 2009 (UTC)[reply]

Polynomial in several variables

Somehow I stumbled on the polynomial . Now I wonder what that is, that's to say, does this polynomial occur naturally in some context of math or other science? 93.132.150.3 (talk) 13:50, 9 May 2009 (UTC)[reply]

It doesn't look particularly useful to me, and I don't think it's a polynomial it's represented in a way that doesn't look very polynomial-ish. It appears to be some kind of norm function (though I don't think it satisfies some of the properties). It looks like it takes two vectors of length n, and combines them to give a single scalar value. Did you notice, though, that if any value of x is equal to any value of y, that the result is zero? Maybe that's a feature, or maybe it's a bug... Nimur (talk) 14:00, 9 May 2009 (UTC)[reply]
You don't just stumble on something like that as if you found it lying in the street, tell us what page it is on or otherwise give context. SpinningSpark 14:36, 9 May 2009 (UTC)[reply]
(After ec) I first mistook for some kind of volume, which it is clearly not, at least not the common kind. In the context I found it, the variables are free symbolic variables, so I didn't think of it evaluating to zero ... but yes, it's a feature, not a bug. I deliberately did not mention any details on the domain of the variables so I wouldn't miss any suggestions for other kind of variables. 93.132.150.3 (talk) 14:38, 9 May 2009 (UTC)[reply]
Take a look at Cauchy determinant SpinningSpark 14:48, 9 May 2009 (UTC)[reply]
Great, that's the polynomial! The way I came on it was that I had a look at signature of permutations where the second proof uses a similar polynomial and I was wondering if the somehow arbitrary restriction to i<j could be avoided. Unfortunately, if you use the above polynomial for that you get the square of the signature, which makes it not very useful for that purpose. 93.132.150.3 (talk) 15:07, 9 May 2009 (UTC)[reply]

cloud formation

I have searched google and wikipedia and could not find the answer to this question. What attracts the water vapor/ice particles together to form clouds? Ionic or static charges? The wikipedia page for cloud formation simply says "When surrounded by billions of other droplets or crystals they become visible as clouds".

Is the attractive force between these billions of particles simply a natural phenomenon and no one knows why, or is there a commonly accepted answer?

thanks for your help,

HarveyHarveyalton (talk) 15:08, 9 May 2009 (UTC)[reply]

For clouds to form, two things must happen, the air must be below the dew point and there must be seeds (eg dust) around which water vapour can nucleate. This can happen with rising warm air, as the air rises in the atmosphere the pressure falls which causes the dew point to fall. If the pressure falls far enough, the air becomes saturated (ie relative humidity is 100%) and water droplets start to form (if there are cloud condensation nuclei present). Other processes can cause air to rise and trigger cloud formation. Amongst these are the collision of warm and cold regions of air in frontal systems and orographic lift. SpinningSpark 15:24, 9 May 2009 (UTC)[reply]
Just a few corrections: the air need not be below the dew point, but only equal to it, for a cloud to form. This is because the dew point, by definition, is the temperature at which the air is saturated with water vapor. Also, it is not the pressure fall which causes rising air to become saturated, but the temperature change due to adiabatic cooling. But these are minor quibbles.
I think the OP misunderstands the concept of a cloud: The cloud particles are not physically bound to each other, they merely reside in the same area. Cloud droplets appear together (as a cloud) because air temperature can not change rapidly in a short area without equalizing itself quickly. Therefore, you won't have just small areas here and there where the temperature is at the dew point, but rather a (relatively) large area on the order of a few hundred or thousand feet. And since the atmosphere has no shortage of cloud condensation nuclei, almost any area at the dew point will also have cloud droplets, and thus a cloud.-RunningOnBrains 16:14, 9 May 2009 (UTC)[reply]

Pur water filter.

We have one of those Pur water filter gadgets that bolts onto the kitchen tap. It has an exchangeable charcoal filter cannister and a little battery-operated gizmo with a red/green LED.

The LED lights green when you run water through it - and the light turns red when the cannister needs changing.

Does anyone know what the sensor is actually measuring? I kinda suspect it's just measuring flow rate and integrating that to figure out the amount of water that's run through the filter...but I'd like to know for sure.

SteveBaker (talk) 16:07, 9 May 2009 (UTC)[reply]

I used to work in a laboratory that used de-ionised water in industrial quantities. We checked for when the filter chemicals needed changing by measuring the resistivity of the water. This can only be done when the water is actually running - as in your device apparently. So the facts fit but its still only a guess. SpinningSpark 16:27, 9 May 2009 (UTC)[reply]
If you have one of these the promo video on that page (click the cinemachine) says the LED indication is based on amount of water used. SpinningSpark 16:36, 9 May 2009 (UTC)[reply]
The one I'm thinking of is this one (actually, the one on the left of that picture). SteveBaker (talk) 17:38, 9 May 2009 (UTC)[reply]
I'mm not sure resistivity is a usable measure for simple charcoal-filtration. I would expect that lots of ions aren't filterable that way, so the detector would have to be calibrated for the expected presence and amount (fluoridation, hardness, etc.) for your particular supply. Ain't gonna get 10MΏ without reverse-osmosis or at least some well-designed ion-exchange resins. DMacks (talk) 18:55, 9 May 2009 (UTC)[reply]
I agree, there is a big difference between filtered drinking water and laboratory quality de-ionised water. I expect it is simply measuring throughput and knows the life expectancy of the filter cartridge. --Tango (talk) 20:13, 9 May 2009 (UTC)[reply]
That's my guess too - but I have a deeper concern. What cheap flow rate sensor might they be using? These gizmo's are pretty cheap. Is it possible that they just measure the amount of time for which water is present? (I'm kinda skeptical that these things are doing a good job). SteveBaker (talk) 22:39, 9 May 2009 (UTC)[reply]
I'm curious - why do have one of these in the first place? Do you live somewhere with water that doesn't taste very nice? As far as I know, that's the only genuine use to these things. Have you done double blind taste test to check if it is really making a difference? --Tango (talk) 00:31, 10 May 2009 (UTC)[reply]
The water quality here in Round Rock, Texas is generally pretty good - but there are certain times of year when the turbidity of the water goes off the charts and despite assurances that it's safe, it looks disgusting - and the filter takes care of that. I have not done double-blind taste tests - I'm more concerned about the actual, published data from the water utility. But overall, I'd rather we had this than have my wife attempt to destroy the planet by buying bottle water - which would be the sure and certain result of not using it. SteveBaker (talk) 14:03, 10 May 2009 (UTC)[reply]
Steve, I can do the standardized test for drinking water turbidity. Can you collect some water on a day when it is very turbid and send it to me? It would be interesting to see if it exceeds what you water utility is reporting (or it exceeds the established limits for these things. ike9898 (talk) 15:58, 12 May 2009 (UTC)[reply]
My Miele vacuum cleaner shows when its dirt bag is full by a mechanical indicator of the air pressure difference across the bag. I suspect the charcoal water filter is monitored the same way, by the water pressure difference. If so, a red light could change to green when the tap is turned partly off. Cuddlyable3 (talk) 00:13, 10 May 2009 (UTC)[reply]
Oohh, Miele keeps my clothes clean and my whites white! Also the best vacuum cleaner in the world. I'm always suspicious of automatic indicators for replaceable items though - they can be equally easy to calibrate to meet corporate revenue targets. Steve, I'd suggest a call to the customer support desk (and persistence) or appropriate use of a screwdriver. I'd agree though with the comments above that it uses pressure differential, which really has nothing to do with whether or not the charcoal is active anymore - it just tells you when the filter is clogged. If they work on the water mains and send down a blob of black goo, you might still be getting good water, just not very much of it Franamax (talk) 00:56, 10 May 2009 (UTC)[reply]
Since they sell a similar gizmo with an all mechanical indicator I'd be rather surprised if they didn't just use the battery operated LED for the indicator. The rest of the setup is probably the same as for the mechanical thing. I don't want to take ours apart. So, I can't tell you what exactly the mechanism is. My guess would be nothing more fancy than an impeller like thing moved by water flowing through the unit. (No pressure increase like in our article.) It likely turns some wheel and lever setup till it hits the battery contacts. Here's an experiment you can do when your filter is full. Just remove it and put it back in. I guess your LED will return to green. (Our mechanical indicator gets reset when I do this.) 71.236.24.129 (talk) 04:23, 10 May 2009 (UTC)[reply]
This patent describes a monitoring system for such a filter (not necessarily your filter though). It consists of a switch activated by the flow of water and a timing circuit that counts how long water has been flowing across the total lifetime of the filter. Replacement is indicated when the timer reaches a preset value. The switch is a simple on-off, and so doesn't measure actual flow but rather assumes some typical flow rate. Dragons flight (talk) 04:25, 10 May 2009 (UTC)[reply]
Possibly also of interest this patent describes a system for measuring actual flow rate based on pressure difference (and not just on-off). Dragons flight (talk) 04:36, 10 May 2009 (UTC)[reply]

Light bulb

What does it mean to have a 60W, 120V light bulb? I would assume it means that the light bulb transfers energy at a rate of 60W and that the potential lost across the light bulb is 120V. The reason I ask this is because I had the following homework question: Two 60W, 120V light bulbs are connected in series with each other and are in parallel with a 1600W, 120V air conditioning unit. The battery has an emf of 120V. What's the current intensity?

To me, there seems to be a contradiction. Both light bulbs have to lose 120V in total, so they can't be both 120V light bulbs, right? Normally I would have said that the total power of the system = 1720W, which equals E*I. Solving for I, we get 14.3A. The correct solution, however, is 13.59A, which I was able to get by the applying the formula P=V^2/R to each light bulb and the AC unit. But working backwards from this, we get that the potential across the light bulbs is 60V (to be expected, but it's still not 120V) and the power is 15W. Can someone help? Thanks. —Preceding unsigned comment added by 65.92.6.148 (talk) 18:43, 9 May 2009 (UTC)[reply]

You're on the right track and see an interesting problem! The voltage-rating of the bulb is just a maximum allowable, not the definite drop. A passive component such as a resistor would have a voltage-drop of "whatever potential is applied across it" by definition:) But if you put "too much" potential across it, you'll cause more current to flow than it can handle or more power dissipation (i.e., heat) to it can withstand or something like that. DMacks (talk) 18:48, 9 May 2009 (UTC)[reply]
So if I understand you correctly, 60W is also the maximum allowable power, right? But since neither of those (voltage or power) describe this situation, how can we use them to answer the problem? —Preceding unsigned comment added by 65.92.6.148 (talk) 19:15, 9 May 2009 (UTC)[reply]
For your problem think of a bulb (and even the AC!) as just a resistor, and the watt and voltage ratings as an indirect way of defining its resistance. Does that help ? Abecedare (talk) 19:27, 9 May 2009 (UTC)[reply]
(ec)Because knowing the maximum power and voltage allows you to calculate the resistance. Once you have the resistance you can calculate the voltage and power in a new situation. But beware, this only works in homework questions, in real life the resistance changes quite dramatically at different voltages because the temperature of the filament is different. SpinningSpark 19:30, 9 May 2009 (UTC)[reply]
The OP sees a contradiction where there is only this error: A light bulb that is called a "120V light bulb" never has to lose 120V. It just "needs" that voltage to shine at its full strength which is 60W. When two such bulbs are connected in series the applied 120V is split between them so each bulb gets 60V. They shine dimly but are still "120V light bulbs", that's just their maximum rating.
Clearly the homework question was designed to be a simplistic exercise using Ohm's Law for DC circuits and ignores some practical realities Cuddlyable3 (talk) 00:01, 10 May 2009 (UTC)[reply]
An air conditioning unit draws a lot of power. I never heard of one that works from a battery. Even the cooling unit on a refrigerated truck works from a small gasoline en;gine. For a homework question maybe we can accept the imaginary situation, But I think it would be better to stay with real-life situations unless there is a definite reason for going into the reaalm of imagination. Otherwise, the imaginary aspect may confuse the student. —Preceding unsigned comment added by 174.130.249.69 (talk) 21:34, 9 May 2009 (UTC)[reply]
Real life problems require many years of experience and training to solve and developing the solution can take months or even years. Homework on the other hand needs to be solved in 30 minutes max with no more than a calculator. Of necessity the questions are artificially contrived and it is pointless and unnecessary to complain about it. SpinningSpark 00:00, 10 May 2009 (UTC)[reply]
Indeed. Any air-conditioner I know of uses an AC motor, so I'd rather expect the current intensity to be the maximum discharge rate of the battery, and if the battery had 120V output, my answer would be "fire extinguisher". However, that's not the spirit of the question and it doesn't even ask whether the ACU is turned on. :) Franamax (talk) 01:07, 10 May 2009 (UTC)[reply]
The question shows (apparently) that the teacher is unaware that light bulbs are far from Ohmic resistances. Their resistance varies dramatically with the voltage across them. The light bulb filament's resistance increases dramatically as voltage increases. A hot tungsten filament has perhaps 13 times the resistance of a room temperature tungsten filament, with more recent estimates being 15 times the resistance. Answer the question in a simple minded way and you will likely get the teacher's answer. Or use the curve from the second ref and get an accurate answer. There is absolutely no reason that light bulbs and air conditioner motors could not be operated from a 120 volt battery. I have worked with 120 volt batteries and with light bulbs operated from them. I have worked with powerful motors operated from 120 volt DC batteries. Edison (talk) 01:29, 10 May 2009 (UTC)[reply]
The small-text indent was supposed to show a light-hearted comment, sorry if that didn't come across. The teacher is obviously making an assumption about steady state operation and expecting straight series-parallel math. Do you have a source for how a lamp filament operating at 60V has appreciably less temperature (and thus lowher resistance) than one at 120V? Not that it matters to the OP, but since you brought it up, I'd be happy to learn. Oh yeah, what was the exact model of air conitioner you ran off a 120V DC battery? Franamax (talk) 02:13, 10 May 2009 (UTC)[reply]
Note the portions of my answer in blue. If you click on them, they magically take you to books. As for DC air conditioners, ones operating on 24 volts DC are readily available at up to 3000 watts input. No reason the motors could not have been wound for 120 volts DC input. (I would exercise caution in connecting 5 of them in series across 120 volts DC (locked rotor and all that)). Certainly an invertor or motor-generator could be used to run an AC air conditioner from 120 volts DC, which to the battery would look like a 120 volt DC air conditioner. Edison (talk) 02:57, 10 May 2009 (UTC)[reply]
How appropriate, to have Edison explaining about light bulbs! Looie496 (talk) 03:15, 10 May 2009 (UTC)[reply]

Balanced symbol equation

A chemistry question, above, mentioned in passing a balanced symbol equation, also calling it a symbol balanced equation. What would one of those be? Even google thinks we might not know. Anyone fancy putting an article together? --Tagishsimon (talk) 19:24, 9 May 2009 (UTC)[reply]

The "reversible/equilibrium reaction" symbol is the ⇌ character (or the wikipedia template {{eqm}}). DMacks (talk) 19:35, 9 May 2009 (UTC)[reply]

I'm not sure that we need a new article. Assuming that "balanced symbol equation" simply means a chemical equation that is balanced, then the concept is already covered by Chemical_equation#Balancing_chemical_equations. --Heron (talk) 20:37, 9 May 2009 (UTC)[reply]
Reversible reaction uses "<math> \rightleftharpoons </math>" to get . hydnjo (talk) 21:15, 9 May 2009 (UTC)[reply]
Unfortunately, unicode &#8652; doesn't have an HTML entity name that I can find. DMacks (talk) 21:39, 9 May 2009 (UTC)[reply]
And yet another image. hydnjo (talk) 22:25, 9 May 2009 (UTC)[reply]

May 10

Microwave oven question

So I'm watching the Food Network and this lady is trying to soften up a big block of chocolate in a microwave oven so she can make chocolate curls and she goes overboard and melts the inside of the block, which happens to me all the time when I try to soften butter - those extra four seconds make all the difference.

My question here is about pre-loading. I believe you're always supposed to put some load into the oven, i.e. if you run the oven empty, you can ruin the magnetron. So if you put a cup of water in with something else, does the presence of the extra EM-absorber influence the other thing?

Put another way - if she'd put a cup of water in the oven at the same time, would the block of chocolate turn out more uniformly heated or would it have melted internally the same way and just taken longer to get ruined? And would it make any difference if it was a static or rotating-tray oven? If I can figure out those questions and world peace, I'll be a happy man! :) Franamax (talk) 01:30, 10 May 2009 (UTC)[reply]

Putting a cup of water would have been a bad idea with chocolate. The steam from the heating water would have caused the chocolate to "seize". Basically, if small amounts of water get into chocolate as its melting, all the fat clumps together in rather annoying ways. Chocolate should be kept scrupulously dry during melting. The water may slow down the melting of the chocolate, but is also is likely to mess up the process itself... --Jayron32.talk.contribs 04:29, 10 May 2009 (UTC)[reply]
For melting Chocolate you should not set your microwave to full power. Most (all?) microwaves regulate the "power" setting by switching the unit on and off at certain intervals. That gives the heat time to dissipate during the "off" cycle. Water takes a lot of energy to heat in a microwave. It's unlikely to produce any significant amount of steam at the settings needed to melt chocolate. (20% to 30% should do for most microwaves.) What you are going to get at full speed is some components of the chocolate starting to boil and separating out of the mixture. (Even if you don't try to boil water at the same time.) It's like trying to melt it in a hot frying pan instead of in a bowl suspended in a pot of hot water. A cup of water in the microwave would have created an uneven heat distribution in the oven. That would not really change with a rotating tray, because the relative position of water mug and chocolate would not be affected. You'd just get less of an effect on the side not directly facing the cup. So just lower your setting or heat the chocolate in short intervals and you should be fine. 71.236.24.129 (talk) 04:52, 10 May 2009 (UTC)[reply]


the real answer to the question is don't use the microwave to melt chocoloate. It's lazy and unprofessional. And gets bad results, as you've discovered. Melt it over water you put on the stove. —Preceding unsigned comment added by 94.27.208.52 (talk) 10:27, 10 May 2009 (UTC)[reply]

There is one reason to melt chocolate in the microwave. SteveBaker (talk) 13:55, 10 May 2009 (UTC)[reply]

has there ever been a double-blind study of any of the claims of the bible?

Have there ever been a controlled, double-blind study of any of the claims of the bible?

I'm thinking things like breaking a group of sinners into control and test groups and having only one group repent and seeing the results, etc. Obviously we can't tell which subjects would go to Heaven, but are there any claims in the bible that we can tell, and which have been tested in a controlled double-blind experiment? 94.27.208.52 (talk) 10:19, 10 May 2009 (UTC)[reply]

Not exactly what you ask for, but the article Efficacy of prayer may be of interest. --NorwegianBlue talk 10:32, 10 May 2009 (UTC)[reply]

There are no claims of the efficency of prayer in the Bible. There are no claims in the Bible that could be verified in the manner you suggest. Generally, the Bible doesn't contain many claims except of the kind "this guy said such-and-such", or historical accounts of the kind of "this tribe slaughtered another tribe, and then the Assyrians enslaved everybody". People unfamiliar with the Bible, or religion, generally tend to confuse claims made by specific churches or clergies with "the Bible". It is very simple to dispel such confusion, all you need to do is actually read the book. Reading the Bible doesn't mean you are automatcally a pious Christian, it simply means you want to check for yourself what these people keep talking about. --dab (𒁳) 10:49, 10 May 2009 (UTC)[reply]

It might be worth to add that some of the historical claims of the bible have been verified, but likewise that many are incompatible with our current knowledge of history. And many again are neither independently verified nor refuted. History does not use double-blind studies, but other methods, of course. --Stephan Schulz (talk) 11:21, 10 May 2009 (UTC)[reply]
I happen to agree with dab that actually reading the bible is the quickest way to convince one of its nonsense. However, I wonder if he has been taking his own advice, I think the bible is littered with references to prayers being answered. A quick random trawl through just one book (1 Kings) came up with numerous examples, for instance;
  • Yet have thou respect unto the prayer of thy servant, and to his supplication, O LORD my God, to hearken unto the cry and to the prayer, which thy servant prayeth before thee to day (8:28)
  • Then hear thou in heaven their prayer and their supplication, and maintain their cause. (8:45)
  • And the LORD said unto him, I have heard thy prayer and thy supplication, that thou hast made before me: I have hallowed this house, which thou hast built, to put my name there for ever; and mine eyes and mine heart shall be there perpetually. (9:3)
By the way, is double-blind referring to Saul and Onan? SpinningSpark 12:00, 10 May 2009 (UTC)[reply]

Unless you're taking the fundamentalist approach to interpreting the bible, the bible makes *no* claims. The bible is a written expression of people's faith, much like a poem or a song can be an expression of a person's love. If you're reading the poem looking for verifiable claims, you've missed the point. Wikiant (talk) 13:20, 10 May 2009 (UTC)[reply]

Of course, if everyone who was "into religion" took that view - the world would be a very different place. Sadly, I'd guess that at least 99% of religious people are either:
  1. People who DO take the fundamentalist view that every word in that crazy book is true...OR...
  2. People who haven't actually read the book from cover to cover and who are taking their view of the subject from what they are told by the people in group (1).
The book itself is mostly self-contradictory gibberish. SteveBaker (talk) 13:47, 10 May 2009 (UTC)[reply]
The trouble with this thought experiment (and one of the major reasons that science says that religion is unfalsifiable) is that if such an experiment were to disprove some hypothesis that's stated in the Bible, the religious folks out there would undoubtedly say something like "But God wants to test our faith and therefore made your experiment come out that way." - there is no way on earth they'd say "Huh! Wow! Well, I guess you atheists were right all along - I won't bother repenting anymore."
So there is no experiment you can do that would falsify what's stated in the bible because it says that god is literally omnipotent - and all rational thought on the subject ends right there. When some hypothesis is unfalsifiable, it's beyond the realms of experimentation and generally regarded as (at best) irrelevant and (typically) something that Occam's Razor says we should simply ignore.
Moreover, the experiment you suggest (where one group of sinners repents and the others do not) cannot be perfomed blind - yet alone double-blind. Again; if the repentant group were not to gain any benefits, the religious crowd would merely claim (with some justification perhaps) that they weren't truly repentant - or that the "unrepentant" group were secretly repentant after all. Of course if the experiment did show some benefit for the repentant group - I would claim (with a good deal of justification) that the results were merely consequences of the placebo effect. Without a true double-blind study (which seems impossible in this case), none of this would tell you anything useful at all.
I suppose the best you could do would be to raise a bunch of children from birth, half randomly assigned to "conventional" religious teachings and with the other half being taught some entirely fake religion. As they grow older, find the sinners amongst them (and one group has the traditional 10 commandments to sin against while the other group has things like "thou shalt not run with scissors" and "thou shalt not exceed the speed limit, even in a 55mph zone") - sort both groups into those who repent and those who do not and follow their progress. If repenters of both religions do better - then it's placebo effect - if only repenters of the "real" religion do better then maybe we're on to something here. However, the religious people would doubtless come up with a whole bunch of weak excuses to explain any results that disproved their position - so it's still unfalsifiable. Of course it wouldn't be enough to have just the children not knowing whether their given religion is real or faked - the whole society in which they grew up would have to be similarly double-blinded...so this isn't in any way a practical experiment.
SteveBaker (talk) 13:47, 10 May 2009 (UTC)[reply]
Statements such as for I the LORD thy God am a jealous God, visiting the iniquity of the fathers upon the children unto the third and fourth generation of them that hate me seem to me to be very much making a claim. Unless as you say, you are reading it like a poem or a song can be an expression of a person's love (although there is precious little love in that quote). But in that case the Bible cannot be held to be the "word of God" since it is now, according to your view, "an expression of people's faith" (not God's word) and I cannot see how it can be held to be one's moral authority with that outlook. You have a nice book of poems but not an authoritive source of belief. SpinningSpark 14:09, 10 May 2009 (UTC)[reply]
I wouldn't say a "nice" book of poems...most of it is exceedingly nasty when compared to modern standards. SteveBaker (talk) 15:01, 10 May 2009 (UTC)[reply]


In reply to Spinningspark, I did not claim the Bible contained any "nonsense". It is a compendium of Iron Age to Roman Era texts, no more nor less "nonsensical" than any other text of the same category or genre.

I did also not dispute that the Bible relates incidents of God answering prayers. It does not contain any claim that God answers prayer with any frequency or predictability. He answers them whenever he bloody pleases. Hence the verses you quoted, which portray God's reaction to prayer as a remarkable exception, not the statistical rule. You will also note that then hear thou in heaven their prayer and their supplication is the subjunctive mood, i.e. the expression of the wish that he may hear, not the statement that he does hear. Obviously, the Bible implies that God is in control. But it does not make any prediction as to when or how he isgoing toexertthat control next time you pray, it is only presenting a collection ofanecdotes of how he (allegedly) did react in the past. I would be interested in your proposal for a double-blind study for that. This would be similar to offering a prediction of what my next edit is going to be based on my edit-history, but with the requirement that this prediction is made where I can see it. I would then be completely at liberty to either confirm your prediction or to prove it wrong. In reply to SteveBaker, if you think the Bible is "self-contradictory gibberish", I trust you have never tried to read a Buddhist sutra or a Hindu Purana. If you're going to read a 2000 year old text, you'll need to be prepared to make some philological effort. You cannot read ancient texts like you do the NYT. The problem with the Bible seems to be that many people assume that they can, while nobody would dream of approaching a Pali sutra or an Ugaritic hymn like that. --dab (𒁳) 12:00, 11 May 2009 (UTC)[reply]

Yeah - I've tried to read some of those other religious texts and I have to agree that they are vastly worse than the Bible. I use the bible as an example because it's the only book of that genre that I've ever managed to read all the way through...and even then, it was a struggle! Talk about turgid! Once you get away from the well known stories, there is an awful lot of pointless waffle about who begat who and junk like that. Also, the Bible isn't a 2000 year old text - it's been hacked about by countless generations of people who were not acting to preserve the meaning of the text - many parts of it are less than 1000 years old. However, it's very clear that there is no effort made to preserve consistency or direction - you are told to do some thing on pain of eternal damnation on one page - then 10 pages later we're told the exact opposite. If this book (in it's present state) were regarded as a mere historical curiosity - then that would be very different from a book that well over a billion people claim to be the core of their belief system. I'm pretty sure though that less than one in a thousand of them has actually read the book from cover to cover (which is REALLY surprising - I mean - if you're going to define your entire life according to some book - you'd really want to read it at some point!)...right now, nobody I know (including one priest, two rabid fundamentalists and one missionary) has ever actually sat down and read it from start to finish. SteveBaker (talk) 13:17, 11 May 2009 (UTC)[reply]

tangent on atheism

And herein is the problem. If you want the bible to stand as "an authoritative set of beliefs," then you must live with the circular argument: The bible is authority because the bible says that it is the word of God and the word of God is authority. Thus, you are left with a choice. Either (1) The bible is, fundamentally, a circular argument and so is relevant only to those who, a priori, believe it to be relevant, or (2) The bible is an expression of the God-human relationship and so is relevant to everyone. Option (1) makes the bible, in effect, completely meaningless. Wikiant (talk) 17:34, 10 May 2009 (UTC)[reply]
I fail to see either the logic of that statement or its relevance to the question we are supposed to be answering. If the Bible is an expression of the God-human relationship (option (2)), it is only relevant to me if it conforms, a priori, to my beliefs in what the God-human relationship consists of. That sounds awfully similar to option (1) in my book. Supposing I accept option (2), it is not relevant to me (and I would guess a lot of other people) because I have no intention of having anything to do with a God-human relationship that has God sending a bear to rip apart children who have merely called his prophet "baldy" or who strikes a man blind for refusing to fuck his dead brother's wife. (edit) Thus your claim that option (2) is relevant to everyone is false by at least one. SpinningSpark 19:15, 10 May 2009 (UTC)[reply]
You're suggesting that, under option (2), the bible is not relevant to atheists. But, wrt assumptions, atheists are simply the flip side of fundamentalists. The former assume that everything in the bible is true (ergo double-blind studies cannot satisfy them). The latter assume that everything in the bible is false (ergo, again, double-blind studies cannot satisfy them). WRT the original question, applying double-blind studies to biblical claims is like applying double-blind studies to poetry -- one misses the point from the very start. Wikiant (talk) 22:42, 10 May 2009 (UTC)[reply]
Atheists don't believe everything in the Bible is false. We believe those parts that are supported by evidence, disbelieve those parts that contradict evidence and are generally indifferent to the rest (most of us use Occam's razor when we need to decide something on those issues, which doesn't happen very often). --Tango (talk) 23:41, 10 May 2009 (UTC)[reply]
(@Wikiant) You assume too much, I made no statement about whether or not I was an atheist, and my reasoning on (2) does not apply only to atheists; it is perfectly possible to believe in God and at the same time discard the Bible. You are making straw man arguments. It is certainly not true that a double-blind experiment could not possibly satisfy a scientifically minded atheist. Regarding us missing the point, the question asked was not what is the point of the Bible (not a suitable question for the Science Desk anyway), but whether it can be tested, which is most definitely answerable. Arguing that the Bible is poetry and so is untestable is really just trying to avoid the issue. Even if that is accepted, the Bible is still being used as a moral reference and it is valid to ask if its moral claims make sense. SpinningSpark 06:59, 11 May 2009 (UTC)[reply]
You aren't describing an atheist, but an agnostic. The atheist believes that God does not exist. If you can be swayed by a double-blind experiment, then you aren't an atheist. Wikiant (talk) 11:21, 11 May 2009 (UTC)[reply]
No, that's nonsense. I believe that when I tumble down the stairs I will fall, so I'm a gravitist. But that does not mean that I would keep that position if I managed to miss the floor and waft away. Of course positions can and do change in response to evidence. --Stephan Schulz (talk) 11:39, 11 May 2009 (UTC)[reply]
See Weak and strong atheism. What you describe is strong atheism, which I consider just another religion (generally an unorganised one, but still characterised by the same rejection of the scientific method). Most atheists are weak atheists (a category which includes (at least some) agnostics, but is larger). I consider myself a weak atheist and not an agnostic. I am certain that, given the evidence I have available to me, I can confidently reject (at any commonly used confidence level) the hypothesis that there is a god. I am, however, open to the possibility that new evidence will turn up to support that hypothesis and I will need to change my beliefs accordingly. I think most people that describe themselves are atheists hold similar views. --Tango (talk) 11:54, 11 May 2009 (UTC)[reply]
Who is "you"? Anyways, that's still not quite right. Strong atheism is a believe that there is no god, weak atheism is the lack of a believe in god, agnosticism is the belief that either we don't know, or that we cannot know, depending on whom you listen too. But even my belief in gravity or no god can be changed by proper evidence. There is a difference between belief and dogma. --Stephan Schulz (talk) 12:16, 11 May 2009 (UTC)[reply]
"You" is Wikiant, the person that wrote the message above mine and one indentation less. "Lack of belief in god" would describe agnostics, weak atheists and strong atheists - by definition, it describes anyone that isn't a theist. The difference between those categories is the level of certainly that there isn't a god. A strong atheist is absolutely certain that there isn't a god and nothing will change their mind. A weak atheists think it is possible, but very unlikely that there is a god. Agnostics aren't sure either way (they may err to one side or the other, but will be somewhere near the 50/50 mark). (Precise definitions vary depending on who you ask, by some definitions the categories overlap (see agnostic atheism, for example [by my definitions, that would probably fall under weak atheism]).) --Tango (talk) 13:08, 11 May 2009 (UTC)[reply]
Well, we have to be a little careful here. My personal position is that the "God hypothesis" is unfalsifiable. That means that I know for an absolute fact that proof of the non-existance of God (as defined by almost all religions) is impossible. That's not quite the same thing as total disbelief...but it's very, very close. I know for a fact that there are an infinite number of unfalsifiable hypotheses (Russell's teapot for example) - and that the probability that any particular one of those is true is arbitarily close to zero...but not zero. Technically, that makes me an agnostic. However, that word is generally understood by the religious fraternity as someone who has genuine conflicted doubt about the existance of God...which could not be futher from the truth. I am as certain that there is no god as that there is no tooth fairy and that there is no Santa Claus. I am as certain that there is no god as that you have not just transformed into a pink bunny rabbit while reading this post. That's a very high degree of certainty indeed. However, it's not 100% - nor can it be because we have an unfalsifiable hypothesis here. What matters is that I act and behave in every way as if I'm certain that there is no god. The almost infinitely small amount of doubt is of interest to mathematicians only! 1/infinity is pretty damned close to zero!
So I am the strongest kind of atheist there can possibly be...because logically, nobody can be 100% certain that an unfalsifiable hypothesis is actually false - although they can be arbitarily close to 100% certain. If the word "atheist" has any meaning at all - then a lot of people are atheists. If it has no meaning then there are dramatically different kinds of agnostics. There are the kind who (like me) put the probability of god(s) existing at the 1/infinity level - those who think it's a 50/50 thing - and the totally rabid fundamentalists who have a 1/infinity level of doubt in their chosen god. If you choose to apply those terms then absolutely everyone is an agnostic...which makes the word useless.
Considering atheism to be "just another religion" is the entire topic of this thread. If you still believe that to be true - then you have not read a single word of my first two posts to this thread. There is a huge difference here. To be a strong atheist (or even a weak one, I think), you have to want logic, proof, verifiability and falsifiability in all things. To be in any kind of religion at all - you have to be prepared to believe without question what someone else has told you - with no possibility of there ever being proof. That's a really dramatic difference. Calling atheism a "religion" is profoundly upsetting to me - and a contradiction of the very meaning of the word: a-theism...no theism...no religion. Religious people would dearly love for the atheists to be labelled as "religious but just in another way" - but that's totally untrue.
SteveBaker (talk) 13:17, 11 May 2009 (UTC)[reply]
Steve, you seem to have misunderstood either me, or the concept of strong atheism. I said that strong atheism is just another religion, and I stand by that. Strong atheism is the belief that there is absolutely no chance of a god existing (not a very very small chance, literally zero chance). That isn't based on logic or the scientific method, it's an absolute assertion which is not (and cannot be) supported by empirical evidence - sounds pretty religious to me (I don't think it is a belief actually held by many people, though). What you describe as your own beliefs is weak atheism, which is not religious. As a mathematician, I must object to your abuse of "infinity". Infinity is not just a very large number, it is larger than any number can be. 1/infinity is precisely zero (for the appropriate concept of infinity, it is undefined for other concepts of infinity). If I interpret you correctly (ignoring your bad maths!), you believe the probability of the existence of a god is a finite, positive real number. A very small one, but still finite and positive. (There are no infinitesimals in the real numbers - that's why 0.999...=1.) --Tango (talk) 16:08, 11 May 2009 (UTC)[reply]
By Wikipedia's definition of a strong atheist (ie someone for whom the probability that gods exist is zero), I don't think anyone is a strong atheist. The older version of the article had a more useful distinction:
Strong atheism - belief that there are no gods
Weak atheism - no belief that there are gods
There might be utility in Wikipedia's current definition to contrast with the people for whom the probability that (their) god exists is one. There are people with that stance. Zain Ebrahim (talk) 14:39, 11 May 2009 (UTC)[reply]
As I've said above, defining weak atheism as the lack of belief in a god is useless. Someone that believes there are no gods also lacks a belief in a god (tautologously). Weak atheism is the view that the existence of a god is extremely unlikely (to the point where you can just assume there isn't one), but with the acceptance that one can never be 100% certain. --Tango (talk) 16:08, 11 May 2009 (UTC)[reply]
Sorry, but that is not the standard definition. There is a difference between "I believe there is no teapot in orbit around Mars" and "I don't believe there is a teapot in orbit around Mars", although it requires a decent supply of nits. The one is active disbelief, but the other is just a lack of positive belief. As an example, someone may not have any opinion on the topic, be it for lack of information or lack of interest. --Stephan Schulz (talk) 16:50, 11 May 2009 (UTC)[reply]
I'm pretty sure it is the standard definition. Either way, your definition is certainly wrong. I know there is a difference between those two statements, but they are not mutually exclusive (a belief in the lack of a god is a lack of belief in a god, as I've said 3 times now [the converse obviously doesn't hold]). If you wish to categorise people by their beliefs in this fashion, those beliefs need to be mutually exclusive. --Tango (talk) 16:57, 11 May 2009 (UTC)[reply]
Ah. There is the beef. No, they do not need to be mutually exclusive. Of course a strong atheist also is a weak atheist, just like a communist is left-wing, or an obese person is overweight. --Stephan Schulz (talk) 19:20, 11 May 2009 (UTC)[reply]
I don't think that is the standard definition. It certainly isn't the definition use in our article, which starts: "Strong atheism is a term generally used to describe atheists who accept as true the proposition "gods do not exist". Weak atheism refers to any other type of non-theism." (emphasis mine) --Tango (talk) 19:29, 11 May 2009 (UTC)[reply]
The odd thing about these definitions is that they describe only three qualitatively dissimilar groups: theists (i.e., probability that god exists = 1), strong atheists (i.e., probability that god exists = 0), and weak-atheists-and-agnostics for whom the probability that god exists is positive but strictly less than 1. Within this group, one might draw a continuum from weak atheist to agnostic to (weak?) theist, but the difference among these is quantitative, not qualitative. Wikiant (talk) 20:13, 11 May 2009 (UTC)[reply]
I don't think that to be a "strong atheist" you have to think the existence of God is actually impossible. You just have to believe that it's possible to derive reliable conclusions from God's nonexistence.
So for example a strong atheist who believes "if the Sharks someday win the Stanley Cup, then there is a God" will also believe "the Sharks will never win the Cup".
On the other hand, a weak atheist who believes "if there is a God, then the Sharks will someday win the Stanley Cup" will simply decline to conclude that the Sharks will someday win.
As for me, I believe in God, but I don't actively hold to very many specific propositions about God. So God is there, but our beloved los tiburones are still going to have to find a way to suck it up in March. --Trovatore (talk) 20:44, 11 May 2009 (UTC)[reply]
I guess that depends on what you mean by "reliable". If you mean "certain", then those two definitions are equivalent. If you allow a little uncertainty then I would disagree. I conclude from the lack of a god that there is no harm in me not going to a church every Sunday, a mosque every Friday and a synagogue every Saturday (isn't is nice of the Abrahamic religions [I regret, I don't know when the various non-Abrahamic religions like to meet up] to have their main prayer meetings on different days? It makes everything so much simpler for those that like to hedge their bets!). I consider that conclusion reliable enough to follow it. I don't consider myself a strong atheist and I don't think I count as one under any common definition of the term. --Tango (talk) 22:53, 11 May 2009 (UTC)[reply]
Yes, it's all pretty qualitative. You might be able to add a dividing line at the 50/50 mark quite easily. I describe a weak atheist as someone that believes the chance of there being a god is very very low, but I don't attempt to quantify "very very low", I don't see anything to be gained by arbitrary dividing lines. --Tango (talk) 22:53, 11 May 2009 (UTC)[reply]

Quark strangeness and charm

No, not the Hawkwind album. Strangeness and charm quantum numbers are preserved in strong interactions, as are the (less commonly used) bottomness and topness quantum numbers. The natural extension of these properties to first generation quarks would be "upness" for up quarks and antiquarks and "downness" for down quark and antiquarks. However, "upness" and "downness" do not seem to be identified as separate quantum numbers; instead, they are merged into the isospin quantum number. Is this purely a historical anomaly (isospin was introduced a long time before the discovery of quarks), or is there a more fundamental reason - e.g. are "upness" and "downness" not actually preserved in all strong interactions ? Gandalf61 (talk) 10:50, 10 May 2009 (UTC)[reply]

Upness and downness are as good flavor quantum numbers as strangeness, charm, bottomness and topness and are indeed preserved by the strong interactions. But it is conventional (and convenient) to replace them by two other flavor quantum numbers, namely isospin and baryon number. It is not uncommon to also replace strangeness by flavor hypercharge. The reasons for doing so are twofold.
Firstly, as you guessed, historically baryon number and isospin were identified as important quantum numbers much earlier then the existence of quarks and their flavors became accepted. Secondly, there are practical reasons for using baryon number and flavor isospin. For instance, unlike upness and downness, baryon number is preserved by all known interactions including week interactions.
Also, the energy scale of the strong interactions as given for instance in terms of the pion decay constant is much bigger than the up and down quark masses and roughly the same order of magnitude of the strange quark mass, but much smaller than any of the other quark masses. That means that the symmetry operation that replacess the up quark with a down quark and vice-versa is a good symmetry and the charge associated with that SU(2) symmetry (which happens to be flavor isospin) is a valueable theoretical tool. For instance, there are two different kinds of nucleons (protons and neutrons) and three different kinds pions, which means there are six different ways pions can interact with nucleons. Flavor isospin symmetry tells us that if we can understand and describe one of those interactions, we have automatically understood and described the other five. It pays to use the symmetries of the theory to organize the description of the interactions.
The mass of the strange quark is not entirely negligible but is still small enough that the symmetry operation that replaces the up quark, the down quark and the strange quark among themselves is a fair symmetry and the charges associated with that SU(3) symmetry (which happen to be the flavor isospin we already talked about plus the flavor hypercharge) are still somewhat valueable tools. That's why sometimes it is usefull to replace strangeness with flavor hypercharge. All the other quarks are too heavy and enlarged symmetries that also include those heavier flavors are bad and their charges are essentially useless. That's why charm, bottomness and topness are used instead.
Notice that I have carefully described those quantum numbers as flavor isospin and flavor hypercharge in order to avoid confusion with weak isospin and weak hypecharge which are entirely different quantum numbers. The latter ones are gauged symmetries and are preserved by all known interactions. Beware that folks in the Nuclear Physics crowd use unqualified isospin and hypercharge to refer to the former ones while folks in the High Energy (Particle Physics) crowd use those unqualified terms to refer to the latter, which unfortunately adds a lot to the confusion. Dauto (talk) 19:41, 10 May 2009 (UTC)[reply]
Thank you for that very clear and comprehensive answer. Gandalf61 (talk) 06:15, 11 May 2009 (UTC)[reply]

Submarine hull breach

In numerous films and books, both fictional and factual, it's said that a pressure hull breach in a submersible at great depth would happen so fast the occupants wouldn't know it had happened, and that a pin sized hole would create a beam of water that could cut a person in half. Can you tell me fast the water is actually moving? I appreciate there are a huge number of factors to consider (cabin pressure, water pressure, cabin size, size of breach and probably a host of others) but roughly how fast would the water be moving? In a (hypothetical) much larger vehicle with much greater internal space, would people potentially be able to witness and even avoid such an event (closing a hatch that can resist the pressure, for example)?81.129.229.14 (talk) 12:44, 10 May 2009 (UTC)[reply]


Well, submarine says that "When submerged, the water pressure on submarine's hull can reach 4 MPa (580 psi) for steel submarines and up to 10 MPa (1,500 psi) for titanium submarines like Komsomolets," - so we can be pretty safe in saying that 1500psi is the most pressure we have to consider here. I looked on the website of FlowCorp who make machines that cut anything from cardboard to titanium using high pressure water streams. According to their site, a machine that can cut through a few inches of cardboard requires a stream that's about one hundredth of an inch in diameter with a pressure of 20,000 psi - and to cut through steel, they use 40,000 to 60,000 psi. So it seems unlikely that a 1500 psi stream from a pinhole (which is around a hundredth of an inch in diameter) would cut a person in half if it has less than a tenth of the pressure needed to cut cardboard. Flowcorp's machines produce a Mach 2 flow at 40,000 psi and a Mach 3 flow at 60,000 psi - so at 1500 psi, the speed would probably not be all that fast.
Actually, we can work it out - the Hagen–Poiseuille equation should pretty much apply here:
Where:
is the volumetric flow rate
is the internal radius of the tube (meters)...0.000125 meters
is the pressure difference between the two ends (pascals)...10,000,000 pascals
is the dynamic fluid viscosity (pascal-second (Pa·s)), 1.3 × 10−3
is the total length of the tube (meters) - say 5cm??
So the flow rate is .000014 m3s-1 - which (through a hole with a cross-sectional area of our pinhole) is 285 meters per second - which is about 640mph. That's quite a bit more than I'd suspect from the FlowCorp numbers...but it depends sensitively on the diameter of that "pinhole" - if the hole is half that diameter then the flow rate is 1/16th as much and the speed of the jet is 1/4th as much. That sensitivity on the size of the hole means that this story can be as true or as false as you want it to be!
SteveBaker (talk) 14:56, 10 May 2009 (UTC)[reply]
The water would presumably start to spread out and slow down as soon as it came through the hole, so it would be going slower once it reached the person that was going to be cut in half by it. I'm not sure how much slower, so it may or may not be significant, but I do observe that the video clip on that website shows the high pressure stream only travelling a few millimetres before hitting the target. What we are discussing would, presumably, be orders of magnitude further (unless the person was leaning about the side of the sub at the time, but even that seems unlikely - they are double-hulled, aren't they?). --Tango (talk) 19:01, 10 May 2009 (UTC)[reply]
The Russian submarine Kursk suffered a torpedo explosion that ripped a 2 square metre hole in its hull at a depth of 100m. Nevertheless, 23 members of the crew at the other end of the submarine survived long enough for an officer to take a roll call and write a list of their names. Sadly, they all died before a rescue could be attempted.
In an incident with a happier outcome, a seawater hose failed on the Australian submarine HMAS Dechaineux (SSG 76) when it was at its maximum diving depth. The engine room was flooded, but the crew managed to control the flooding just in time to allow the submarine to surface. Gandalf61 (talk) 15:56, 10 May 2009 (UTC)[reply]
Check out Cecil Adams' answer to a similar question here. --Sean 13:12, 11 May 2009 (UTC)[reply]

The volume of ice at the north and south ice caps and Greenland?

I seem to recall that the rise in sea level is suppossed to be 50 metres if both ice caps, the ice over Greenland, and all other ice melted. But I am sceptical. The world is a big place. The oceans have an area of about 360 million square kilometres. If they rose by 50 metres, then the extra volume would be 18000 million million cubic metres. What is the total above-sea-level volume of ice at the north and south ice caps, Greenland, and other glaciers and ice? 84.13.171.69 (talk) 14:39, 10 May 2009 (UTC)[reply]

Well, your skepticism is easily debunked given some numbers:
Firstly: be careful - not all of the sea level increase predicted from Global warming comes about from melting ice. Remember, as objects heat up, they expand - and that will happen to our oceans if we don't stop the temperatures from increasing. However, the rate at which the warmth from the atmosphere heats the deep oceans is hard to estimate (I got told off about that the last time this question came up!) - so let's ignore that for the moment - bearing in mind though that it's certainly not an insignificant part of this process.
The ice over the antarctic is way more than a kilometer thick in places - remember that a cubic kilometer is a thousand million cubic meters - so 18000 million million cubic meters sounds impressive - but that's "only" 18 million cubic kilometers.
According to antarctica the antarctic ice sheet covers 13.7 million square kilometers and averages 1.6 kilometers thick...which is 22.4 million cubic kilometers - significantly more than the 18 million you need to make your math come out right. Which means that a 50meter sea level rise can easily be accounted for by the melting of antarctica alone...that's forgetting all of the ice and snow on mountains and glaciers in the rest of the world and the effect of the expansion of the volume of water due to the overall warming trend. There are even worse consequences possible here...as the ice melts from the continent of antarctica, the quadrillions of tons of pressure of all of those cubic klicks of ice goes away and the continent will rise upwards as the pressure is relaxed. This will allow the continental shelf in the region to rise up - displacing yet more water to the rest of the planet. To add insult to injury - the increasing water depths pressing down on the continental shelves (and further inland as flooding begins) will press downwards on the other continents making them sink a little - making it seem like the water is rising even faster (although in reality, it would be the land sinking - that's little comfort to someone living on the coast!) Some estimates for the worst case sea level rise go as high as 70 meters...and given these numbers, you can see why! SteveBaker (talk) 15:26, 10 May 2009 (UTC)[reply]
Of course to melt so much ice, it would take about 6x1024 joules to overcome the enthalpy of fusion of ice. Given that the relevant solar constant in Antarctica is well below the 300 watts per square meter average for the planet, it would take a long time to melt. (My back-of-the-envelope calculation suggests that if we could convert 100% of the solar radiation in Antarctica into 100% efficient ice-melting, we could melt that much ice in just 50 years, though!) Of course, this calculation is not a realistic model of climate-change - I haven't accounted for albedo, let alone convection and weather! - but it does put some perspective on the size of the numbers involved. Nimur (talk) 15:54, 10 May 2009 (UTC)[reply]
There are also factors that go in the other direction, though, including geothermal heating and heat transfer between the tropical and polar regions, as well as increased ice flow rates as the ice thins and its temperature rises. Looie496 (talk) 16:36, 10 May 2009 (UTC)[reply]
A couple of things I missed:
Melting ice at the north polar ice cap doesn't directly affect the sea level because it's already floating - which means that it's already displacing the same amount of water it'll create when it melts. That's good news - but along with it comes bad news. As Nimur points out, it'll take a long time to melt all of the antarctic ice...however, it doesn't actually have to melt - if it merely slides into the ocean and floats - that's enough to produce a problem. It has been noted that there are mechanisms that accelerate the destruction of glaciers when the meltwater from the top of the glacier finds it's way through crevasses to the underlying ground and produces a lubricating layer between the ice and the ground beneath. This allows the ice to slide much more rapidly down to the ocean where it can first form "ice shelves" and then break up into icebergs (which displace water even before they melt).
Greenland contains only about 2 million cubic kilometers of ice - less than a tenth the amount of antarctica - but it's melting much more rapidly - so in the short term, it's a bigger problem.
SteveBaker (talk) 17:08, 10 May 2009 (UTC)[reply]
Your first point directly above is not quite correct (as was pointed out to me the last time I answered a similar question - isn't it great that we can learn from new material as we come across it?). Don't forget that the water is salty, but the ice is fresh, so the displacement is not nearly as exact as we might hope. Antarctica is a huge multiple whammy due to the reasons mentioned above, but the arctic will also be a significant source of sea level rise. Matt Deres (talk)
Well - it's true - seawater is about 2.5% denser than distilled water - so there would be a tiny difference due to the arctic ice cap melting - but it's not enough to make much of a difference to global sea level. The arctic ice is only 4 to 5 meters thick on average - so even if 100% contributed to sea level rise, it would be negligable compared to the antarctic glaciers at 1600 meters thick...but you're right, technically that 2.5% density difference makes a tiny difference to the ocean depths. SteveBaker (talk) 12:24, 12 May 2009 (UTC)[reply]
Half right. The salinity makes a huge difference to the current flows and speed of mixing. Which in turn drives the rate of rise. I see we do not have an article on Double diffusive convection yet.--BozMo talk 12:33, 12 May 2009 (UTC)[reply]

Time Travel

Wouldn't time travel be undesirable? Besides the grandfather-paradox and everything else about going back in time (which is really impossible considering you need a 'receiver' which wasn't built years ago), consider this scenario. Some time in the future the world is going to face a calamity, e.g. the sun expanding into a super giant and enveloping the earth. All those people in the future (who are still on this planet) would want to find the fastest way to get out of the situation. Assuming they all choose to go back in time. The population of the planet at the time they choose to go back to would increase immensely. But the calamity in the future is still going to happen. Assuming all those people living there at the time of that calamity also decide to go back in time, and this keeps happening, sooner or later we'd have an infinite number of refugees on the planet. Maybe we should make some rules and regulations on what times in 'history' (which will be our future anyway, until the receiver is built) you are allowed to send billions of people back to, to prevent ridiculous overcrowding.--KageTora (영호 (影虎)) (talk) 15:22, 10 May 2009 (UTC)[reply]

As long as you go back far enough that you won't live to see the calamity a second time, and you don't have any children, there isn't a loop to worry about. As long as you spread people out over a millennia or two, there shouldn't be able real problem. (Assuming such a thing is possible, which it probably isn't. Also, if you can invent a time machine you can probably use similar methods to invent a spacecraft that travels faster than light, in which case you could evacuate to another solar system instead.) Incidentally, the sun will turn into a red giant, not a supergiant. --Tango (talk) 15:27, 10 May 2009 (UTC)[reply]
If you accept the repugnant conclusion, this would likely be a very, very good thing. — DanielLC 15:31, 10 May 2009 (UTC)[reply]


Yeah - there have been countless sci-fi stories about the undesirability of it...and few (if any!) that make it sound in any way desirable. However, time-travel is impossible - so it's not really something we have to be concerned about. However if you'd like food for thought:
  • Let's dump all of our CO2 emissions and other garbage back into the Triassic era.
  • Temporal tourism results in huge crowds of time travellers with fancy holographic cameras gathering just before momentous events in history. It becomes virtually impossible for any historic event to go "as planned".
  • Stock markets collapse, banks fold, casino's can do no business as future travellers do the "deposit a penny in 1800 and collect the compounded interest in 2800", sending back in time copies of almanacs and horse-racing results.
  • Temporal anomalies (if your version of time travel allows them) make life hard to lead.
  • Business collapses as each one attempts to patent all of the products of their competitors before that product was even invented.
  • Businesses realise that they can manufacture goods in one time and send them back or forwards to a time when they were either vastly more expensive or insanely desirable.
  • The movie business collapses as people travel forwards in time until their copyrights have expired and watch new movies (and the sequel that hasn't even been made yet) for free.
Pretty soon, you realise that it all becomes very silly! SteveBaker (talk) 15:35, 10 May 2009 (UTC)[reply]
(EC)If they didn't have any children, they would all die out in, say, a hundred years or so (assuming life-span hasn't increased dramatically), but that would leave a block of about a hundred years which would be uninhabitable, because of the huge population. This would keep happening until all the 100-year blocks would be gone. Also, presumably it'd be the same people coming back every time, if their 'history' (our future) is not changed by all these people coming back (which would actually be the grandfather paradox).--KageTora (영호 (影虎)) (talk) 15:49, 10 May 2009 (UTC)[reply]
True, Steve. Nice answer. I could imagine the Battle Of Agincourt, with all the French and English soldiers suddenly surrounded by millions of people, and they'd be thinking, 'where did all these people come from? Shall we postpone the battle?', which would probably end up just not happening. Nice one.--KageTora (영호 (影虎)) (talk) 15:55, 10 May 2009 (UTC)[reply]
As long as your time machine doesn't allow for travel back to before it was invented (which, to my knowledge, all serious ideas for time travel don't - unless you can find a naturally occurring time machine that has existed for a while), a lot of those problems disappear (or can, at least, be solved). Time limited copyright and patents would obviously be pointless, you would to change those laws so they make more sense. Just like we have the Berne convention and similar agreements to handle copyrights in different countries, we would need agreements between different time periods (I'm not sure what they would say, though... the idea of limited monopolies to encourage creativity without stifling progress is a good one, limiting in terms of time is an excellent way to do it but we would need some other limitation if time travel become commonplace). Economics would change drastically once the time value of money becomes zero. The underlying principle of trade would still function perfectly well. Intertemporal trade would be very similar to international trade - things like Comparative advantage would still apply, just within a very different framework. Google suggests several people have given the matter some thought. Seeing as I'm supposed to be revising, I'll go and read some of their conclusions! --Tango (talk) 16:50, 10 May 2009 (UTC)[reply]
Sadly, not my original idea. There is at least one SciFi book that describes this idea. JFK's limo sets out through the streets of Dallas - but before it can pass the school-book repository, the whole of Dealy Plaza is suddenly crammed full of bizarrely dressed people clutching their tiny, white iTimeMachines. The presidential Limo can't even make it along the street and is forced to turn back and find a different route. As Oswald emerges, confused, from the repository, hundreds of happy tourists surround him demanding autographs and asking him to pose for their holo-snaps. One inexperienced temporal tourist offers Mr Zapruder $100 for his camera. Sadly, when they return to their own time, nobody has any clue what they are talking about and nobody has even heard of Oswald. SteveBaker (talk) 16:54, 10 May 2009 (UTC)[reply]
I generally assume the Novikov self-consistency principle, so such problems don't exist. --Tango (talk) 17:52, 10 May 2009 (UTC)[reply]
Exactly what I am talking about, Steve. Consider the combined populations of both any single point in the future and all possible points in the future. It would be possible even to go back to the same point in time you've visited before multiple times and see many versions of yourself. It's a ridiculous thing. Plus, you could even go back in time, live your life there, and so long as you didn't alter the path of your parents meeting (Cf. Back To The Future), you could be safe in the knowledge that even when you die, you'll be born again, anyway.--KageTora (영호 (影虎)) (talk) 18:00, 10 May 2009 (UTC)[reply]
I don't see any problems with any of that. It's rather different to how we generally consider existence to work, but it is all perfectly consistent with itself and the assumption that time travel is possible. --Tango (talk) 18:22, 10 May 2009 (UTC)[reply]
Seems to me though the Novikov principle might result in incredibly constrained existences for those people who travelled to the past. You wouldn't be able to buy that particular loaf of bread, because it has the folate needed for the neural development of the fetal ancestor who then is smart enough to go to the school where he meets the woman with whom he parents a child whose descendant eventually introduces your grandpa and grandma. You wouldn't be able to chop down the tree whose descendant eventually falls across a creek and allows your mother to escape a lava flow just in time. That would be some strange existence. Franamax (talk) 20:46, 10 May 2009 (UTC)[reply]
Yes, but you probably wouldn't notice anything strange. You probably wouldn't even try and chop down that tree. You just have to get rid of this purely religious notion of "free will" - you are part of the universe, your decisions are part of the universe. If you universe is "conspiring" to prevent paradoxes, it will probably do so simply by you not thinking of causing them. The only time you would observe anything strange would be if you consciously tried to cause a paradox. Accidental paradoxes would just not happen with no particular reason required for them not happening. --Tango (talk) 20:53, 10 May 2009 (UTC)[reply]


Apart from being undesirable it would be a complete disaster, im thinking about the Butterfly effect...:) —Preceding unsigned comment added by 84.64.16.41 (talk) 20:46, 10 May 2009 (UTC)[reply]

It's interesting that a descendant of H.G.Wells, who first put down the idea of a time machine in one of his books, called, funnily enough, The Time Machine, was on Discovery a few weeks ago discussing the grandfather paradox. He extrapolated that if he went back to kill H.G.Wells before he came up with the idea of the book, the universe (which tries its best to eliminate paradoxes - except in the quantum realm) would make the gun not go off. He would then be asked why he tried to kill H.G.Wells during his police interrogation, and he would say he was testing the grandfather paradox, which, in turn, according to his theory, might actually cause H.G.Wells to write the book.--KageTora (영호 (影虎)) (talk) 21:43, 10 May 2009 (UTC)[reply]

It's a fun thought. I'm not sure I'd call it "interesting", though. It is too highly contrived to be interesting. --Tango (talk) 23:36, 10 May 2009 (UTC)[reply]
Time Travel is one of those technologies that everyone wants to have, but nobody wants anyone else to have. See also Invisibility Devices, Mind Reading, Flying Cars, etc.
I think this is an essential quality of a good science fiction technology. APL (talk) 03:54, 11 May 2009 (UTC)[reply]

The original poster's idea has been done I think rather definitively, albeit in the opposite direction, in The Restaurant at the End of the Universe. --Trovatore (talk) 04:04, 11 May 2009 (UTC)[reply]

I wouldn't say any of steve baker's..er... examples are problems which we would face if there is time travel... Rather, since none of these has already happened, we can say these are pieces of evidence. Which, of course, leads us to one of two conclusions : a) As has been pointed by many, time travel is impossible. b) Even if time travel is invented in the future, we, for whatever reason, won't be able to do these things. Maybe because there is some superstrict board which monitors us and forbids us to do anything of that sort... maybe it has already been invented but the scientist is just too scared that people will misuse it in the above mentioned ways that he has just kept it mum... Maybe we didn't understand properly... a different interpretation which satisfies all these problems... Who knows... My point is that don't count your chickens before they hatch.. Anything is possible in Science and Religion... Rkr1991 (talk) 11:43, 11 May 2009 (UTC)[reply]

What does religion have to do with it? --Tango (talk) 11:46, 11 May 2009 (UTC)[reply]

Hey that was just a general comment... like all's fair in love and war... Like the joker says, why so serious ? Rkr1991 (talk) 11:54, 11 May 2009 (UTC)[reply]

"All's fair in love and war" is a well known idiom with a clear metaphorical meaning. "Anything is possible in Science and Religion" isn't. --Tango (talk) 16:12, 11 May 2009 (UTC)[reply]
Isn't the OP's question the exact plot of the Star Trek episode All Our Yesterdays? Astronaut (talk) 18:25, 11 May 2009 (UTC)[reply]
I hadn't seen that episode, but after reading the article, it would appear that it is, basically, yes.--KageTora (영호 (影虎)) (talk) 04:56, 12 May 2009 (UTC)[reply]

Psychokinesis

Hi, My name is Emil Khalilbekov, I'm a photographer from montreal. you can join me at <EMAIL ADDRESS DELETED> would like to make a BIG POINT in Psychokinesis subject.

here is your link http://en.wikipedia.org/wiki/Psychokinesis

I have red the hole thing and i find you have forgotten a BIG BIG DEAL....

You talk about Psychokinesis and telekinesis ect... most of your subjects are american peoples, how and who ever made Psychokinesis happen and possible and some articles and facts of government and scientists opinions....

But you HAVEN'T EAVEN ONES SPOKEN ABOUT " the Chinese techniques in martial art" Or what they compare Psychokinesis to is :::::::"CHI ENERGY":::::::: the power of CHI or QI or KEE or other various names, as The Yin and The Yang .... I mean you have your links for all of this but (((( MY PROBLEM IS THAT YOU DON'T LINK THE TWO SUBJECTS TOGETHER!!!!!!! ))))


Psychokinesis is a word to replace the believe of the The Yin and The Yang..... Moving objects by the Chi Power, Deep Meditation and Chi exercises, You can develop your "Psychokinesis" Powers by doing QI GONG Exercises every day!

This is a big point , if you read your own articales you will realise that both subjects talk about the same effect but both are not linked together....

BIG MISTAKE!!!!!


'I would STRONGLY recommend to make more researches and add an IMPORTANT article that connects the two subjects together !!!!''''Bold text

Chi Power or as some call Psychokinesis IS a NATURAL Effect of the Human being....


If Article added, It would be cool to add my Name as the person who reminded this fact to you. —Preceding unsigned comment added by 70.53.148.72 (talk) 15:35, 10 May 2009 (UTC)[reply]


Sheesh! Where do we start here?
  1. What is your actual question?
  2. Please don't post your message twice - please don't include your email address (I've deleted it).
  3. Psychokinesis has NEVER not once, ever been demonstrated under proper scientific conditions - it's bullshit. If you can do it - or know someone who can - talk to the Committee for Skeptical Inquiry people or the James Randi Educational Foundation and they'll give you a million dollars (literally!) if you can prove it under rigorous conditions. The fact that nobody has yet done that - despite this lucrative offer - speaks volumes.
  4. This is not the place to soapbox your favorite ideas.
  5. This is not the place to discuss changes to Wikipedia articles - use the article's own Talk: page.
  6. Be aware that telekinesis is considered "pseudoscience" and Wikipedia has some increasingly strict rules about how we portray such things. If it's bullshit (and this is), we're going to say so.
  7. Nobody's name is "attached" to an article - take a look at any of them - no accreditation is ever given.
  8. Wikipedia is "the encyclopedia you can edit" - which means that if you find a mistake or see something that needs fixing, you have to do it yourself. However, I'll go out on a limb here and say that every edit you make along the lines you describe will be deleted again within a lot less than 24 hours.
SteveBaker (talk) 15:44, 10 May 2009 (UTC)[reply]
Steve, can you package that response into a suitable template format, for quick re-use? I've been on pseudoscience patrol often enough to need it... Nimur (talk) 16:09, 10 May 2009 (UTC)[reply]
I have it on a hot-key. :-) SteveBaker (talk) 16:37, 10 May 2009 (UTC)[reply]

And (9) Please don't SHOUT. It will only serve to destroy what little credibility you might gain.--86.25.193.89 (talk) 16:52, 10 May 2009 (UTC)[reply]

"UN-altered REPRODUCTION and DISSEMINATION of this IMPORTANT Information is ENCOURAGED, ESPECIALLY to COMPUTER BULLETIN BOARDS." —Tamfang (talk) 20:52, 10 May 2009 (UTC)[reply]

I think he's complaining that the Psychokinesis article doesn't mention the Chinese concept of Qi. Astronaut (talk) 18:09, 11 May 2009 (UTC)[reply]

I tried placing it on a computer bulletin board with telekinesis, not touching the keyboard, sadly without any results. What am I doing wrong? Should I have used a Mac instead of a PC? Edison (talk) 14:02, 12 May 2009 (UTC)[reply]
I think Linux is the only OS that supports telekinesis, and also FreeBSD if I recall correctly --BiT (talk) 15:27, 12 May 2009 (UTC)[reply]

Why do males have nipples?

- Most male mammals have nipples like female mammals. Why do the males need nipples? Is it a kind of rain check, to be used later if the females become unable to suckle for some calamitous reason? ~~ —Preceding unsigned comment added by 59.93.8.216 (talk) 15:41, 10 May 2009 (UTC)[reply]

ava looke at Nipple#Nipples_on_male_mammals a little bit maybe

ncal43 —Preceding unsigned comment added by Ncal43 (talkcontribs) 15:44, 10 May 2009 (UTC)[reply]

The male nipple actually carries a very nice lesson about how evolution works. The only genetic difference between male and female mammals (well, for most mammals) is the Y chromosome, which marks an animal as male. Because there is only one copy of the Y chromosome in a cell, it cannot evolve in the same way as other chromosomes, which have two copies each. The consequence is that the Y chromosome is very small -- nature keeps as few genes as possible on a chromosome that can't evolve in the usual way. This means that the genetic difference between males and females is tiny -- basically just a few genes that control the expression of other genes. Since the genomes are so similar, every structural difference between males and females requires complicated gene-programs, so it pays for nature to keep the differences as small as possible. It's easier to reduce an organ to a small, nonfunctional form, like the male nipple, than to eliminate it completely. Looie496 (talk) 16:52, 10 May 2009 (UTC)[reply]
I would maybe reword that last sentence to say that it is easier for an organism to use the same body-development plan for as long as possible before coming to rely on genes on the sex-linked chromosomes. Looking at the link Ncal43 supplied, it seems that rather than the "organ" being "reduced", it is actually the further development of the mammary gland which is suppressed. And the Y chromosome is actually full of a startling amount of junk. Franamax (talk) 20:09, 10 May 2009 (UTC)[reply]
It only takes a tiny bit of female hormone to make the male breast start producing milk. The normal male breast is indistinguishable from the immature female breast. See Gynecomastia and Male lactation. Edison (talk) 23:00, 10 May 2009 (UTC)[reply]
There is only really one relevant gene on the Y chromosome - SRY. It determines the production of male hormones, everything else follows from that with exactly the same genes as females have. --Tango (talk) 23:32, 10 May 2009 (UTC)[reply]
I should clarify - there are other sex-linked genes, but they don't determine sex. They happen to be on the part of the Y chromosome that doesn't cross-over with the X chromosome, but that is just coincidence. --Tango (talk) 23:34, 10 May 2009 (UTC)[reply]
Further to the above, I would like to question the implicit assumption invariably accompanying this question, that nipples only have one function. Many organs in many organisms have dual or multiple functions, and the nipples are no exception. Apart from their primary role as efficient interfaces between maternal breast and infant mouth (which is not indispensable, as Monotremes lack nipples), they are also, at least in H sapiens, erogenous zones, being erectile and sensitive during sexual arousal. This applies to male as well as female nipples. Any mature and sexually active male unaware of this must have a very unimaginative partner. 87.81.230.195 (talk) 03:53, 12 May 2009 (UTC)[reply]

Physical error in Star Trek?

In the new Star Trek film, Kirk and Sulu are falling in a free fall toward the surface of Vulcan, when they are beamed up in mid-fall to the Enterprise, and they are shown falling on the transported pad with an audible thud, and then merely brush it off. Now, when falling on the ground from the sky, isn't what kills you that the deceleration happens too fast, causing all the kinetic energy in your body being transferred in one go? In this case, it doesn't matter what Kirk and Sulu are falling on, or from what height. If they aren't gently slowed down in advance, they'll die anyway when they suddenly stop falling. In fact, they don't even need to hit anything - merely stopping in mid-air would be fatal if it happened too fast. Have I got this right? JIP | Talk 17:06, 10 May 2009 (UTC)[reply]

It's fiction. Try not to worry about it. Transporters don't really exist so issues of whether they are depicted realistically are quite utterly unanswerable. (When Time Magazine asked "How does the Heisenberg compensator work?", Star Trek technical adviser Michael Okuda responded, "It works very well, thank you.") SteveBaker (talk) 17:12, 10 May 2009 (UTC)[reply]
I was expecting this kind of answer. Merely replying this way is, in fact, equal to saying Kirk could have simply waved a magic wand and caused Nero's evil rogue ship to disappear and make everyone happy, when after all, it's fiction. What I would have wanted, is an answer to whether I have understood the physics thing in falling down correctly. Just because something didn't really happen is no excuse to throw all pretense of realism out of the window. JIP | Talk 17:16, 10 May 2009 (UTC)[reply]
Actually, that's why the transporter needs a Flux_capacitor. It adjusts the speed of incoming bodies to match the inertial frame of the receiving station. In the old series, Scotty is doing this manually with those levers he operates. If the relative speed is high, he may not compensate the movement completely - and rather than risking to have them fly up and then down again, he errs the other way. Alternatively, look at the concept of a Bergenholm in the Lensman series, which operates by temporarily suppressing the inertia of a body, allowing it to accelerate without picking up kinetic energy (and to revert to the old velocity once it's disabled again). --Stephan Schulz (talk) 17:33, 10 May 2009 (UTC)[reply]
Yes, you are essentially correct, if they impacted the transporter at the same speed they were going to impact the ground at, it would have the same effect. We'll have to assume the transporter changed their speed when it changed their position (I see no reason why it couldn't do that - if you can rematerialise matter you should be able to choose its momentum pretty freely) - let's assume the small drop is because whoever was transporting them (I haven't seen it yet) was afraid he wouldn't get the position exactly right (perhaps due to them falling in an unpredictable way) and didn't want them to materialise halfway through the pad. As for what would happen if you just stopped in mid-air, it would depend on what stopped you. The reason rapid deceleration is bad is because different parts of your body decelerate at different rates (the bit that impacts first stops almost immeadiately and the rest of your continues moving, so you get crushed). If whatever stops you stops you in a similar fashion, it will be just as bad, if it stops all of you at the same rate (using artificial gravity, or something), then it would be harmless. --Tango (talk) 17:43, 10 May 2009 (UTC)[reply]
I'm sorry - but you can't expect a scientific answer for something that IS magic. Transporters do not - nor cannot exist...period. What they do or do not do with residual momentum when the source and destination are moving at different speeds is quite utterly unknowable. I could say - "Well, the StarTrek transporter perfectly removes the difference is speed and direction between source and destination - and all you saw in the movie was due to the transporter placing the center of gravity of the humans at an appropriate distance above the deck. The problem was that they were lying flat at the time and thus fell two feet to the ground on arrival"...but it's fiction...it doesn't exist...it doesn't matter. When you watch a StarTrek movie, you turn off your brain and enjoy the ride. SteveBaker (talk) 17:47, 10 May 2009 (UTC)[reply]
Nonsense, it's still entirely possible to apply a modicum of scientific thought to the process. After all, science is a way of approaching problems. So long as we accept that A) These are not real-world facts we're dealing with. and B) If we go too deep we'll eventually hit either an unknowable, or a contradiction. Then why couldn't this question be approached in a completely logical, scientific way?
We have observed transporters moving individuals from planet surfaces up to a ship in orbit. (In fact, that's their primary purpose.) The speed difference between the ship and the planet surface can be massive. (They usually show the Enterprise entering orbit against the planet's rotation, for whatever reason.) Since they can complete these planet-to-ship transports without splattering anyone against the side of the transporter room, we have to assume that the transporter is adjusting the away team's speeds to match the ship's speed. So the physics of this dramatic beam-out in the new movie is nothing special, the transporter must do that for every single transport. (Luckily, obscure transporter physics is not what makes the scene dramatic.)
So why did they crunch against the floor? Easy. They beamed in on their sides instead of the usual boots-down orientation. Even if they were perfectly lined up with the floor of the transporter you'd still fall a little. Imagine the difference between beaming in with both boots flat on the deck, and beaming in with one elbow just barely touching the deck at a weird angle.
We've seen this sort of thing before, where someone beams up in a weird position and then falls when they beam in. Not so much in the original series, but in TNG, Voyager, etc. Another fun thing is when they beam someone up who's running, but they beam in stationary and have to 'catch themselves'. I'm sure we've seen that once or twice. APL (talk) 03:47, 11 May 2009 (UTC)[reply]

Tango's reply answered my original question about physics. As for SteveBaker's comment, you are essentially right, but your original reply came out as a simple handwave, rather than the detailed explanation you now gave. Stephan Schulz provided a good reply while keeping in mind that Star Trek is fictional: "Yes, if they weren't slowed down, they'd die. That's why Scotty used a flux capacitor to slow them down. How does a flux capacitor work? By magic." I wasn't asking about the internal workings of flux capacitors or transporters, but about simple everyday physics that happen outside the transportation. JIP | Talk 18:04, 10 May 2009 (UTC)[reply]

(EC - twice)Transporters could not exist and would be a dangerous thing anyway even if they did. We all know about the 'got transported inside a physical object by accident and died' thing. Well, that would happen in any place that has an atmosphere. You'd be transported into that air, and oxygen atoms are physical objects. You'd end up meshed up with the air.--KageTora (영호 (影虎)) (talk) 18:23, 10 May 2009 (UTC)[reply]
That problem could perhaps be solved by having the transporter transport the air out of the way first, by having the transported object and anything at its destination simply exchange places? How would it do that, then? The same way it would already work: by magic. We aren't interested in how transporters work, we are interested in what effects they have. JIP | Talk 18:27, 10 May 2009 (UTC)[reply]
Well, just as a side note, for a transporter to work, it would have to dismantle all the atoms in the person's body, then reconstruct the body at destination. This would mean that the actual scene where they suddenly appear in mid air would be silly, considering they never did that before in any other scenes. As for the rapid deceleration thing, yes. There is an advert on British TV warning against bad driving where two cars have a head on collision. And it warns (very graphically) that even if you have air-bags you can still be killed because your internal organs keep moving even when you and the car have actually stopped suddenly, and they can all be ruptured.--KageTora (영호 (影虎)) (talk) 18:35, 10 May 2009 (UTC)[reply]
Which, I think answers your question. It would be an instant dismantling of every atom in the body, therefore causing no movement of internal organs caused by any rapid deceleration, and then reconstructing the person at destination.--KageTora (영호 (影虎)) (talk) 18:41, 10 May 2009 (UTC)[reply]
So, when they reappeared above the transporter pad, they would have zero velocity, and then they would just drop down the half a metre or so onto the transporter pad, leaving them essentially unscathed? That makes sense. How the transporter is going to change the velocity to zero without going through deceleration is a question of quantum physics, not mechanics, and therefore outside the scope of this topic. JIP | Talk 18:47, 10 May 2009 (UTC)[reply]
Teleportation is generally described in terms of transferring information - you examine the object at its source, destroy it, transmit the information about it to the destination, and rebuild it. There is no reason why you should have to rebuild it with the same momentum it started with (although to momentum of parts of it relative to the others would need to stay the same - you don't want your heart to stop mid-beat, or anything, who knows what would happen then?), since it isn't the same object. It is a new object that is created to be identical to the first. --Tango (talk) 18:52, 10 May 2009 (UTC)[reply]
"There is no reason why you should have to rebuild it with the same momentum it started with" No reason? Except that simple, point-by-point subtraction of the free-fall momentum is not going to work! Every atom is so confounded by thermal noise; macroscopic displacements of organs are going to throw the tissues out of place; molecular vibrations; statistical trends of gas-flow in the lung and fluid in the blood.... I don't think you could just "subtract" a scalar speed from every atom in a body and have everything "be okay." (Imagine what making an enormous change to the net velocity of every single atom will do to the instantaneous temperature and pressure of the material?) SteveBaker is correct to hand-wave this off as magic. If we had a scientific explanation for these "small details", we would have already figured out transporter technology. We do not have any explanation for these details (probably because it is fundamentally physically impossible; at the very least, we have at least got a technological limitation). Therefore, every time you make a "science-like" statement, as a "flux capacitor that resets the momentum", you are just moving the bar up a notch in terms of where you stopped caring about the magic. You can't explain the flux capacitor scientifically, so it's just technobabble for a "magic-wand". I don't see why you should even bother pretending that this has solved the physical incongruity. Nimur (talk) 19:06, 10 May 2009 (UTC)[reply]
Well in TNG, they replaced the flux capacitors with pattern buffers, so they must have been working on the science. :) Possibly helped by the Stargate SG-1 team. I'd like to know about a few more of the small details though, such as why a phaser vanishes the person, their clothing and anything they're holding - but not one scrap of ground or the building they're holding onto. Also why no-one ever thought of installing seatbelts when they upgraded the Enterprise. The TV/film industry is truly a strange land. At least the old-time SF writers had to try to explain the miracles. Franamax (talk) 19:47, 10 May 2009 (UTC)[reply]
That isn't a problem with removing the momentum, that's a problem with teleportation itself. We have to suspend disbelief at some point. In my opinion, good science fiction requires us to only suspend disbelief for a very small number of things (they can be extremely unbelievable, though) and then everything else follows logically from those assumptions. If you assume that teleportation is possible and just ignore all the problems with it, then it requires no further suspension of disbelief in order to accept that you can remove the momentum from a teleported object. Star Trek is reasonable sci-fi by that definition, but they do tend to resort to technobabble a little more than they ought. X-Files is terrible sci-fi (well, "speculative fiction" is probably a better term, it isn't strictly sci-fi, but the same point applies) - it requires you to suspend disbelief separately for each episode (which is particularly bad considering it is meant to be about solving mysteries and there is no way you can possibly solve the mystery before it is revealed [and trying to do so is part of the fun of watching such shows] since you don't know what unbelievable thing is going to turn out to be true this week). Stargate is good, once you accept the premise of the show there is very little that is unbelievable in each new episode (nothing at all in many episodes). --Tango (talk) 20:24, 10 May 2009 (UTC)[reply]
Yes, also, laser weapons would be viewed as a constant stream of light, not a bolt that flies along much slower than the speed of light, considering they are made of light.--KageTora (영호 (影虎)) (talk) 19:54, 10 May 2009 (UTC)[reply]
Funnily enough, transporters DO exist, at least in some form. I saw something on Discovery about one that has been built, but it can only transport a single atom at this point in time. I have no links for this. Anyway, there was some question about it being the same atom or a replication of the one the machine just destroyed. The scientists who created it, however, said it was the exact same one, but didn't provide any proof. To transport a human, however, would require (obviously) a bigger machine and much more power to store all the information during the 'transportation' process. But this is a problem, because we have no way of knowing whether it is the same person that has been 'transported' or an exact replica of one that has been utterly destroyed, which is, in fact, terrifying if you are the one going through the process. The reason we have no way of knowing is because, being an exact replica, they will say they are the same person.--KageTora (영호 (影虎)) (talk) 19:51, 10 May 2009 (UTC)[reply]
Is this] what you're talking about? They're not claiming to have moved the physical atom though. Franamax (talk) 19:54, 10 May 2009 (UTC)[reply]
... and even if one atom could be "transported" (or re-assembled as an identical copy), there are bandwidth limits that mean it will never be possible to "transport" anything nearly as complex as human by any technology imaginable at present. It's fun to pretend, though. Dbfirs 20:27, 10 May 2009 (UTC)[reply]
Quantum teleportation. It's quantum information (qubits) that gets moved, so it's not really teleportation. It's the same as this message making its way from my computer to yours—there are no electrons or other particles that make the trip. I'm not teleporting electrons even though there happen to be electrons on both ends. Furthermore you can move qubits around by using a quantum channel, you don't need the quantum teleportation protocol. So really it's not a big deal, and I wish these articles wouldn't be so sensationalistic about it. -- BenRG (talk) 22:14, 10 May 2009 (UTC)[reply]
A moderate fall to the floor is a moral lesson by the transporter. Falling at high speed can be dangerous when there doesn't happen to be a transporter with a helpful operator around. The transporter doesn't compensate completely for the speed difference in order to give the fallers a scare so they will try harder not to fall another time. That's my theory anyway. Seems at least as good as technobabble. Some parents use a similar principle when warning/punishing their children for doing dangerous things. "Son, you could have drowned by swimming less than an hour after eating so I'm pulling out the waterboard to show you how it feels". What, your parents didn't do that? PrimeHunter (talk) 20:55, 10 May 2009 (UTC)[reply]
But it would be exceedingly unusual for the velocity of an orbiting transporter room to exactly match the velocity of the place they're beaming from/to. There are at most two locations moving with a given velocity on the surface of a rotating sphere. Only dramatic license can explain the observed behavior of Star Trek transporters. -- BenRG (talk) 22:14, 10 May 2009 (UTC)[reply]
Back to the workings of the flux capacitor, which JIP asked about, I'm thikning that what might have happened (and I'll grant you I haven't seen the movie) is that the atoms were not just snatched fromt he air like, say, a baseball player catching a ball. Instead, couldn't they have been slowed down at a rate allowing them to suffer minimal damage, then "beamed in" when they were slow enough to avoid the harmful effects?
In other words, if velocity was lethal limit, all the atoms were slowed down while continuing to go ythrough the air, with a "transporter lock" on them. They were slowed gradually, then, even moved back up a ways if they had to be. Then, when they were going slow enough, they could then be safely brought to a place where they could land.
This may just be explaining the "magic" a little more, but at least it sounds more plausible than some things. After that, I'll just use the line fromt he parody I wrote in college once:
Scotty: I can't defy the laws o' physics, Cap'n.
Kirk: Sure you can, Scotty, this is Star Trek, we do it all the time. :-)Somebody or his brother (talk) 01:12, 11 May 2009 (UTC)[reply]
That wouldn't fit with how transporter beams are usually described, it would need to be a tractor beam. There is no reason why they couldn't write a story in which a tractor beam and a transporter beam are used together for such a purpose, but I see no reason to assume that's how it worked unless they say so. As I've said above, there is no reason to think momentum has to be preserved during teleportation. (And, as BenRG points out, there is going to be relative velocity between the source and destination anyway whenever the ship is in orbit.) --Tango (talk) 10:16, 11 May 2009 (UTC)[reply]
Actually - the things that bother me most about that movie in particular are the ways that the computer interacts with people. Like when the computer politely informs the captain that the autopilot has been damaged...surely the autopilot is a piece of software that runs in the computer? If it has enough working processors to perform speech recognition and to drive all of those fancy displays - why the heck can't it run the 7 lines of code you need to implement an autopilot?
 while ( the autopilot is engaged )
 {
   if ( you're too far to the left ) steer to the right a bit ; else steer to the left a bit ;
   if ( you're pointing too far up ) steer down a bit ; else steer up a bit ;
   if ( you're going too fast ) slow down a bit ; else speed up a bit ;
   sleep ( 1 millisecond ) ;
 }
You could argue that the computers sensors were damaged - yet they appear to be giving the captain realtime on-screen data of time to collision - you could argue that the computer's control over the engines and steering are broken - but then we'd have to believe that there is a hard-wired connection between the joystick on the captain's chair and the actual engine controls, which seems really unlikely.
If he hadn't kicked the engineering team off of the ship he could have had them write those 7 lines of code (it took me less than 30 seconds) then had the last of them beam off the ship. Anyway - what's wrong with "Computer: Transfer control to the shuttlecraft"? Or just aim the ship in the right direction and say "Computer: Don't touch the engine controls" - then run to the transporter bay and beam yourself to the shuttlecraft?
Then, when Scotty is trying to "get a transporter lock" on the falling crewmembers, he's looking at a display with dots on it showing the position of the two crewmembers and frantically trying to line up a couple of cursors on top of them. Aside from the fact that he only seems to have to do this in two dimensions(?!?) - the fact that computer knows their positions accurately enough to draw the graphics on the display means that it would be TRIVIAL to have it 'snap' the aim cursor to the nearest dot...making it a simple matter for any unskilled transporter officer to snatch people out of the air like that.
Conclusion: The Federation's computer programmers are utterly useless!
Anyway - we should listen to the wise words of Arthur C. Clarke: "Clarke's Third Law: Any sufficiently advanced technology is indistinguishable from magic." - which is what we have here. Magic - with all of it's capabilities and arbitary limitations. SteveBaker (talk) 12:30, 11 May 2009 (UTC)[reply]


There are some minor problems with the code given for the autopilot. In reality it's not that simple. What you just described was the most simple proportional controller, which will oscillate horribly even in the most ideal conditions. For control systems at least a PID controller is required, and if not all of the system's parameters are known, or there can be measurement errors (which they always are), than even that's not enough and you have to combine them with predictors and parameter estimators, or use more advanced and complicated controllers. However, I still agree that if the voice recognition works, the autopilot should also work, except for some required hardware accelerators burning out (you know one of the most important starship design rules: no fuses in the cockpit), but even then there should be backup software solutions available. But I am sure it's a lot more than just 7 lines of code.
Science fiction means, that it can contain devices or events which are not possible by our current understanding and/or technological level. You can invent your own rules, but you have to stick to them. So if you have a flux capacitor and you use it by your own rules and without contradicting yourself, it's fine. You can use any crazy science you want, if the connections in it are logical, the consequences are well described, and all the gadgets are explainable under the rules you use. Look at Stanislaw Lem for example: in the novel Fiasco he uses giant walking robots, cryogenics, some kind of pseudo-FTL space travel, gravity-altering devices and weapons, nanofungus, but it still remains scientific: the miraculous gadgets are not just plot devices, their functionality as well as their shortcomings are presented in detail and, logically, seem plausible. Or, a more modern example, Fine Structure [3] has every crazy thing even thought about by science fiction authors: alternate dimensions, teleportation, FTL, time travel, superpowers, universal constructors, telepathy, etc., but is still amazingly scientific and realistic.
Star Trek is, by the way, not science fiction in the strict sense. Science is always just a background or a plot device. The problem of these kind of "flux capacitors" is, that they usually do not follow even their own rules. If it really worked, would it make sense? - that's the question we should ask. In the Back to the Future, for example: Why would anyone have a newspaper article about a photo of an empty place in the graveyard? How do Marty's parents not freak out when they realize their son looks exactly like the mother's previous lover? How does doing just nearly similar things lead to the nearly same future? How can Marty possibly take the place of a completely different child (as he altered his parent's marriage quite a lot)? - These are the questions that make it unrealistic, and not the "how does the flux capacitor work?". --131.188.3.20 (talk) 14:23, 12 May 2009 (UTC)[reply]
Plot devices aside, there is actually an awful lot of considerations embedded in the auto-pilot's concern "Where should the ship be and how fast should it be moving?". It would start with knowing what course was entered, knowing current position and velocity relative to something (what reference frame does one use on a starship?), and being able to adjust for potential obstacles, etc. I don't see a huge problem with a ship being able to predict time till collision and yet not having enough guidance for a full autopilot to know where it should be going, etc. Your point about simply having the ship maintain constant velocity in a predetermined direction is a good one though, I don't see why that should be a problem. Dragons flight (talk) 16:29, 12 May 2009 (UTC)[reply]

I know I'm coming late to the party, having only just seen the movie, but I have to but in with the obvious answer. <assume star-trek meta-physics>The transporters always have to compensate for a large velocity difference, since the ship is in orbit, thus has a significant velocity in the frame of the "stationary" person being transported from the planet surface (this has been mentioned above). As for their horizontal orientation upon arrival, the same point applies: being in orbit, the ship is almost never in the same "upward" orientation of the person being transported, so the transporter must compensate for this always. So, if you are talking about consistancy with previous and subsequent events (which I can only assume represent the physics of the Star Trek universe), their velocity upon arrival is not an error, but their orientation is.</assume star-trek meta-physics> Ew I feel dirty, let's not talk about hypothetical future physics anymore :-D -RunningOnBrains 16:55, 12 May 2009 (UTC)[reply]

Also, it seemed to me that near the end of the movie when they beam Spock off the little future ship, he's sitting down when he gets transported but arrives standing up. I guess the plot wouldn't have benefited from slapstick "chair pulled out beneath him" humour at that part of the script, but the nebulously defined transporter serves its role as a charmingly flexible plot device yet again. TastyCakes (talk) 17:14, 12 May 2009 (UTC)[reply]
Star Trek transporters (and the equivalent technology in other si-fi) have always been able to straighten people's legs, there are plenty of examples of it. I don't see anything that function would contradict, so we just need to suspend disbelief for it. --Tango (talk) 17:57, 12 May 2009 (UTC)[reply]
My point is that the transporter apparently couldn't change the orientation of Kirk and Sulu when they were falling, but it could change Spock's orientation when his ship blew up. TastyCakes (talk) 18:50, 12 May 2009 (UTC)[reply]
Perhaps the sudden change in gravity would be harmful? As it is, they went from about 1g (presumably they were at terminal velocity, I haven't seen it) to 0g (they would have been in free fall once they appeared above the pad). Perhaps changing from 1g in one direction to 1g in another would be too much (it would be a change of about 1.4g in magnitude). It's not a particularly good argument, but it will do if you would rather that than just calling it creative license. --Tango (talk) 17:57, 12 May 2009 (UTC)[reply]
If transporters worked by creating some kind of wormhole, then momentum, velocity, gravity and all these would have a great effect. However, by the use of Heiselberg compensators, I thing it's very clear they work by deconstructing the target and reconstructing an identical copy. So they could reconstruct the copy when and where and facing whatever direction as they wanted. The only question that remains is why are they not using it for "saving" people and reconstructing them in case of death, or just healing injuries by replacing damaged body parts. Maybe a protocol forbids it? --131.188.3.20 (talk) 23:20, 12 May 2009 (UTC)[reply]

the sun's limb

There's an old joke:

"Your cough sounds better."
"I've been practicing."

Well, I have forty-some years' practice in making myself misunderstood. So this is a paraphrase of my question of April 28 (I didn't see the replies until today).

Given:

  • The sun, being gaseous, has no discrete surface. (I got clear agreement on that point, but that's not what I was looking for.)
  • It sure looks as if it had one.

What is it that looks like a surface? When astronomers speak of the limb of the sun, e.g. to define the four 'contacts' of a transit, what do they mean? —Tamfang (talk) 19:10, 10 May 2009 (UTC)[reply]

It is a gradient, it's just hard to see it very clearly. In some cases, especially eclipses, it shows up very nicely as a gradient. Have you looked at photosphere and chromosphere and solar corona? Depending on your astronomical need, and your area of research interest, any of these might be considered "the surface" of the sun. Specific definitions with specific radii will be suitable for particular cases, but as you already realize, there is not a clean radius at which there is total consensus to call the "edge". Nimur (talk) 19:32, 10 May 2009 (UTC)[reply]
Gradient. The central region has been blocked out to prevent saturation of the imager, because the contrast difference between the photosphere and the corona is pretty huge. Nimur (talk) 19:32, 10 May 2009 (UTC)[reply]
In the eclipse picture, I see a gradient in the outer corona, yes. I don't see a gradient in the part of the picture that is relevant to the question. —Tamfang (talk) 20:37, 10 May 2009 (UTC)[reply]
Would one definition be the point where an object behind the sun is no longer visible at a given (or any) wavelength? Franamax (talk) 19:51, 10 May 2009 (UTC)[reply]
Perhaps, but I don't think that would give you a firm boundary. Objects would just get harder and harder to see (as more and more of the photons from them get blocked). There would be no point where suddenly all the photons are blocked when none of them were before. --Tango (talk) 20:58, 10 May 2009 (UTC)[reply]
There is no precise boundary to the Sun, just a gradual fade to nothingness.
The reason it looks like it has a clear boundary when you look at it with the naked eye (or, more advisedly, through a solar filter!) is because the range over which it varies from dense enough to be easily seen to sparse enough to be impossible to see is quite small (from this far away, at least). If you look at it through a telescope you can see that it does fade out gradually, as seen in this picture. --Tango (talk) 20:04, 10 May 2009 (UTC)[reply]
Sun#Photosphere: The change in opacity is due to the decreasing amount of H- ions, which absorb visible light easily. That's a start, anyway. —Tamfang (talk) 20:37, 10 May 2009 (UTC)[reply]
I'm not sure of this, but it seems to me that clouds could be seen in a similar way. From a distance they can look opaque and sharply defined, yet the closer you get the less distinct their boundaries appear to be. I am not a meteorologist nor an expert on the sun - just wondering if it's an apt analogy. --Scray (talk) 01:48, 11 May 2009 (UTC)[reply]
Sounds like a good analogy to me. There are quite a lot of difference between small droplets of liquid suspended in a gas and a large ball of plasma held together by self-gravitation, but neither has a clear boundary. --Tango (talk) 10:18, 11 May 2009 (UTC)[reply]
As I recall, the apparent sharp "surface" is an optical illusion caused not so much by decreasing opacity as by the changing index of refraction. Supposedly in this situation there is a discontinuity, a well-defined limiting angle from which light rays can appear to come. I've never actually worked through this in detail. --Trovatore (talk) 09:08, 12 May 2009 (UTC)[reply]

Lightning Photographs

hi,

how do people take pictures of lightning? I mean its obviously not someone with super quick reactions, so how is it done? Constantly taking photos during a storm and hoping to get something? Computer detectors that can take a pic instantly?

thanks, --84.64.16.41 (talk) 20:35, 10 May 2009 (UTC)[reply]

Long exposure with a pinhole aperture? —Tamfang (talk) 20:40, 10 May 2009 (UTC)[reply]
Lots of photographs of empty sky will eventually yield a few photographs with good lightning. Especially in the era of digital cameras, this is becoming increasingly common. I know some people who attempt to do automated shutter triggering, but this is difficult. Nimur (talk) 20:55, 10 May 2009 (UTC)[reply]
All the pictures on lightning seem to have been taken at night, which supports the "long exposure" theory. (If you take a long exposure shot during the day it would just be overexposed all over unless you had a special lens with a minute aperture.) --Tango (talk) 21:08, 10 May 2009 (UTC)[reply]
Actually, I'm not sure even a really small aperture would work - it would probably have to be so small as to not pick up the lightning. The key things are the difference in brightness between the sky and the lightning and the length of time the lightning is there relative to the length of the exposure. The latter has to be very short if you don't want to have to take a crazy number of shots before you get a good one (if you have a 1/100 second exposure, you would probably have to take thousands of shots, if you have a 1s exposure you might be able to get away with a few dozen during a big storm - the exposure being 100 times longer essentially makes the lightning 100 times shorter). If the latter is very short, then the former needs to be very large to make up for it. --Tango (talk) 21:13, 10 May 2009 (UTC)[reply]
Yes that's right, all the really spectacular pictures are taken with long exposure, that is, fix the camera down and lock the shutter open. Not much good for anything other than to look pretty as often there is more than one strike in the shot (but makes for a better picutre). It is perfectly possible to capture lightning with a hand-held camera and a "normal" shutter speed, I've seen it done several times, but be prepared for a lot of failures. With high speed cameras it is possible to capture the ascender strike followed by a frame of the descender. This is more scientifically useful, but not so pretty. SpinningSpark 21:27, 10 May 2009 (UTC)[reply]
If you wanted to do that, though, wouldn't you use a high speed video camera? --Tango (talk) 23:30, 10 May 2009 (UTC)[reply]
Lightning strikes up?--KageTora (영호 (影虎)) (talk) 00:27, 11 May 2009 (UTC)[reply]
See Lightning#Leader formation and the return stroke. --Tango (talk) 00:52, 11 May 2009 (UTC)[reply]
You could "just" set up a digital video camera, aim it at the sky in night or daylight, and push "Record", then grab the one or two or three frames that recorded the strike. By digital video camera, I don't mean MiniDV or anything that records in PAL or NTSC, because the frame will look awful; but rather a camcorder that records to an MPEG stream of some sort. Tempshill (talk) 02:33, 11 May 2009 (UTC)[reply]
I've only shot lightning at night, as others have discussed above, but there are lightning triggers that will detect the lightning flash and release the camera's shutter. These can be used in daylight. Here's an example, no endorsement implied. I've also seen plans based on an Arduino. -- Coneslayer (talk) 12:08, 11 May 2009 (UTC)[reply]

May 11

Quanta

Reading A Brief History Of Time and hearing from other sources, it says that even observing a quantum particle changes it. What is all that about? Does it mean the light hitting it from an electron microscope changes it? Or is there some other mechanism involved? Also, how do they know this, if they didn't see what it was like before they observed it (logical to me)?--KageTora (영호 (影虎)) (talk) 00:32, 11 May 2009 (UTC)[reply]

Well, that might not be strictly true. If you set up an electron to have spin 1 on any axis and measure spin along the same axis, there should be no change in the state of the electron. It is of course a bit pointless to measure something that you already know, but it disproves the assertion nevertheless.
On the second question: Simply setup a repeatable experiment where you can make two measurements. Run the experiment 100 times and measure both things, then run it 100 times and measure only the second thing. If the results of the second measurement differ significantly between both batches, the first measurement had an effect. —Preceding unsigned comment added by 84.187.75.89 (talk) 01:50, 11 May 2009 (UTC)[reply]
I don't think that works. Even if you measure something you already know, your measurement still changes it. The spin after you measured it is now in an unknown state. Nimur (talk) 06:01, 11 May 2009 (UTC)[reply]
"Observing a quantum particle changes it" means that (some) measurements don't commute, in other words if you do measurement A and then measurement B you get a different outcome for B than if you'd done B first. Since it's a statistical effect you have to repeat the experiment many times (on identically prepared systems) in order to see it. So I think anon's description was exactly right. It's also true that measuring something you already know doesn't affect the system (at least, quantum mechanics doesn't say it has to). -- BenRG (talk) 12:29, 11 May 2009 (UTC)[reply]
(EC) The book is probabily talking about wave function collapse. Your interpretation of it as being the effect of the light on the particle being observed is a real phenomenon usually refered to as observer effect. These are two different things. The second one happens wheather you take quantum effects into consideration or not. It even happens in other situations that have nothing to do with physics. See for instance observer's paradox. The wave function collapse is a much more misterious phenomenon in which a particle which is in a state of superposition of several pure states suddenly jumps into one of those states when it is observed. To this day there is still considerable debate about what exactly happens when the wave function collapses. But there is no doubt whatsoever that it happens. As to the second part of your question, the particle being observed may have been carefully prepared so that they know beforehand what state it is in, and then when the experiment is performed it is verified that the particle indeed jumped into one of the possible states compatible with its original state and the experiment performed. Dauto (talk) 02:07, 11 May 2009 (UTC)[reply]
There's no doubt that the model with the collapsing wave function works, but a lot of doubt that particles suddenly jump into the measured state just as you measure them. The only thing you can determine experimentally is that the system was prepared to be in one state and later measured to be in another; what happened in between is a matter of interpretation. In path-integral quantum mechanics there isn't anything that looks like wave function collapse, and the spreading out looks symmetric in time—the maximum indeterminacy is midway between preparation and measurement. That matches the experimental reality better. If you have a particle emitter and a particle detector at opposite sides of a room, you can investigate what sort of changes to the room will affect the rate of particle detection (e.g. adding a barrier or a block of glass), and the answer (both in theory and in practice) is that the rate will only be affected if you put things where the path-integral formalism says the particle "is", not in the much larger area that the wave-function-collapse picture says it "is" just before detection.
The other reason to be suspicious of the picture with the spreading out and the collapse is that it looks like classical probability. There's always some uncertainty in the preparation of a system and that uncertainty increases with time (for example, an uncertainty in momentum leads to increasing uncertainty in position). But when you measure the position several times in quick succession you obviously should get roughly the same answer each time, no matter how much the position had previously "spread out" in your model. In order for the model to predict that correctly, you have to "collapse" the system's position in the model after the first measurement, and the mathematical process for doing that looks just like the mathematics of the quantum collapse (a Bayesian update). In the many-worlds formulation of quantum mechanics that's exactly what the collapse is, a Bayesian update with no objective effect on the world. The objective effect of measurement that people attribute to the collapse is actually caused by quantum decoherence.
In short, the situation is far too complicated and ambiguous to conclude that wave function collapse is physically real. Given how much it looks like a Bayesian update and how little it looks like other physical laws, I'd bet pretty strongly that it isn't. -- BenRG (talk) 12:29, 11 May 2009 (UTC)[reply]
Yes, I agree with that point of view. But that's still not a universally accepted interpretation of what happens at the wavefunction collapse. My statement intended to convey the fact that there is still dabate about that specific point. I think the term "Wave Function Collapse" is here to stay even if the decoherence interpretation eventually becomes universally accepted. Dauto (talk) 16:06, 11 May 2009 (UTC)[reply]
(Incidentally - an electron mictroscope uses electrons - not light - to image its target. Not that this changes the answers.) But basically, yes - there is no way to measure something without somehow affecting it. For large scale objects, the consequences are negligible - but for things like electrons, the consequences are significant - just one more weird thing that happens at the quantum level. SteveBaker (talk) 12:14, 11 May 2009 (UTC)[reply]
(Except that the rules do permit some measurements to leave the system unaffected—for example, a remeasurement of a property that you already know, but there are also less trivial examples.) -- BenRG (talk) 12:29, 11 May 2009 (UTC)[reply]
If you really want to enter into the "was the information really there before we measured it" rabbit hole, the most interesting places to start are EPR Paradox, Bell's theorem, and, somewhat differently, my favorite, Wheeler's delayed choice experiment. --140.247.10.164 (talk) 14:20, 11 May 2009 (UTC)[reply]
Clearly there is a high probability that Schrödinger's cat will eventually be mentioned somewhere in this thread. In fact, it was in a superposition of mentioned/not mentioned states - until I just mentioned it, which collapsed its wave function ... Gandalf61 (talk) 14:43, 11 May 2009 (UTC)[reply]

Hypocenters

I know that you can calculate the depth of a hypocenter by p waves, but is it possible to calculate the location of a hypocenter? —Preceding unsigned comment added by 174.6.144.211 (talk) 02:00, 11 May 2009 (UTC)[reply]

Yes. Dauto (talk) 02:23, 11 May 2009 (UTC)[reply]

how? —Preceding unsigned comment added by 174.6.144.211 (talk) 02:54, 11 May 2009 (UTC)[reply]

By comparing the time of arival of seismic waves at different seismographs around the world you can find out which seismographs were closer and which ones were further away and therefore reconstruct where and when the earthquake happened. Dauto (talk) 04:21, 11 May 2009 (UTC)[reply]
See hypocenter and particularly epicenter#Epicentral distance for an explanation. Astronaut (talk) 17:55, 11 May 2009 (UTC)[reply]

Chimera

In the Chimera dab page, somebody took out an entry for the wrong reason. My knowledge of genetics is pretty limited, so it's just possible that there is already an entry there that covers the same topic. The two items are:

  • Chimera (protein), a hybrid protein made by splicing two genes (still there)
  • Chimera (EST), a single cDNA sequence originating from two transcripts

Advice? Clarityfiend (talk) 03:26, 11 May 2009 (UTC)[reply]

I think you misunderstand the purpose of disambiguation pages. They are not intended to provide a comprehensive list of all of the meanings of a word - that's the job of a dictionary - and for that we have Wiktionary. The purpose is (as its name implies) to remove ambiguity when there are multiple articles with the same (or very similar) name. Since there is no article Chimera (EST), there is no ambiguity - so there does not need to be a DAB entry for it. Now, if you are arguing that there SHOULD be such an article - then go write one and add it to the DAB page. But until that happens, it's perfectly OK to remove the entry in the DAB. SteveBaker (talk) 12:06, 11 May 2009 (UTC)[reply]
A red main link is not a valid reason to delete a dab entry (see MOS:DABRL). Granted, the bluelinked article that went with it didn't refer to "chimera" as the guideline requires, but there are sufficient ghits both inside and outside of Wikipedia to make me think the topic is more than just a dictionary definition. If the two dab items refer to the same concept, there's no problem. However, if they are different, a related bluelinked article might be found or chimera might be mentioned in cDNA. Clarityfiend (talk) 21:22, 11 May 2009 (UTC)[reply]
I don't understand where you're coming from with that. MOS says: "A link to a non-existent article (a "red link") should only be included on a disambiguation page when an article (not just disambiguation pages) also includes that red link." - so is there an article out there (not a dab) that links to "Chimera (EST)"? If not, then the MOS clearly says that it shouldn't be in the DAB page. If I do a search on "Chimera (EST)" and click on "all pages that link to "Chimera (EST)"" in the search results - there is a talk page and two entries for the ref-desk. No articles. Hence the DAB entry most certainly should have been deleted per MOS:DABRL. What's to argue about? SteveBaker (talk) 12:10, 12 May 2009 (UTC)[reply]
I don't know much about genetics either, but they are clearly two different things; one is protein and the other is DNA. Also expressed sequence tag is probably a better target for the blue link as this is the subject that gives rise to Chimera (EST). But as Steve says, the red link has no incoming links from other articles and none of the potential blue links discuss it. Having an entry on the dab page only misleads readers into thinking Wikipedia has information when there is none. SpinningSpark 22:16, 12 May 2009 (UTC)[reply]

"God probably doesn't exist"

Buses in London carried for some time ads endorsing atheism. They said - "God probably doesn't exist". Does it make sense to say "probably" in this case? Isn't this a case where we cannot apply any meaningful statistics? PS: this question is not about the existence of God, so don't hijack the topic.--Mr.K. (talk) 15:54, 11 May 2009 (UTC)[reply]

You can't apply frequency probability, you can apply Bayesian probability. --Tango (talk) 15:57, 11 May 2009 (UTC)[reply]
(after edit conflict) I think you can get quite close to a frequentist probability. Culturally, there have been many gods (Zeus, Ra, etc). If you're willing to accept that Zeus doesn't exist (as most people will) then you can simply count the number of gods, and count the number that you're quite sure don't exist. This gives you a low estimate of the probability that a given god doesn't exist. I'm not saying this is valid reasoning, but it is a way to apply frequency probability.
You can also generalize this to all instances of a particular type of claim, which I think is closer to what people actually do and explains the connection with the informal use of the word "probably" here. As you go through life, you encounter various types of claims, and for many you eventually decide whether they're true or not. For instance, you will encounter many spectacular claims for which there is no strong evidence, ie. claims that require faith. And you may be undecided on some and decide to reject others. This gives you a probability distribution over all such claims.
This, I think, is more or less why many people will dismiss things like ESP, horoscopes etc. offhand, without careful scientific investigation. A thorough investigation would cost far too much energy, so they rely on their internal probability model (ie. experience) to reject these things.
It is in this sense that atheists might say that God "probably" doesn't exist. Their experience tells them that claims of that nature often turn out to be untrue (by their personal concept of truth), and they judge subsequent claims by that experience, in this case because there is no way to achieve a scientific level of certainty.
I should also point out that in this case the odd use of the word "probably", is exactly what lends the phrase its humor. Much like Carlsberg tag line: "Probably the best beer in the world". risk (talk) 16:24, 11 May 2009 (UTC)[reply]
I think the example with Zeus and others is not appropiate. They were deities, but they were not omnipotentt beings, creators of the universe, etc. All the so called "gods" from Greek and Roman mythology were just like some superhumans. The current concept of God is that of an omnipotent force or being, who or what designed the laws of physics. I think the question is not about weather he/it exists, but how we interpret it. These advertisments however don't seem to address questions about neither philosophy nor probability, they seem to be just a provocation, just like gay pride parades.--131.188.30.106 (talk) 08:59, 12 May 2009 (UTC)[reply]

The poster's a nice piece of provocation, and is a response to a similar campaign from the opposite camp. Note that according to I think therefore I am, the bus doesn't exist either. --Dweller (talk) 16:06, 11 May 2009 (UTC)[reply]

Since when has me existing implied buses don't? --Tango (talk) 16:15, 11 May 2009 (UTC)[reply]
I disagree; the logic of cogito ergo sum merely implies that while I definitely exist, the bus might not exist. It makes no more concrete statement as to the degree of certainty of the existence of the bus; except that it is less definitely extant than I am. Nimur (talk) 18:19, 11 May 2009 (UTC)[reply]
Furthermore, I think the cogito ergo sum reasoning proposes several "equally likely" scenarios; one possibility is that the entire universe also exists, and is as it seems; another possibility is that none of it exists, but it appears as such because some entity (God?) is deceiving me; or alternatively, that I am deceiving myself (and that I am the entire universe, including God). My personal belief is that the first case is the most interesting one. This case must be true. If it isn't, then none of you exist, so your opinions of what is true or interesting are irrelevant; and I can single-handedly decided what is true and correct anyway. Nimur (talk) 18:23, 11 May 2009 (UTC)[reply]

<-Chaps, it was a joke, based on the fact that buses can't think. Never mind. --Dweller (talk) 20:30, 11 May 2009 (UTC)[reply]

My English could be failing me but isn't "God probably doesn't exist" actually a categorical statement that "God does not exist in a probable manner" rather than a dubious statement on his (her) existence? Isn't that a truism, since whether or not God exists it could only be in a definite manner? I seem to remember getting sentences starting "Hopefully, xyz" marked through in red at school for this reason....--BozMo talk 16:17, 11 May 2009 (UTC)[reply]
I think that would be "God doesn't exist probably"(note the lack of a comma), or better yet: "God doesn't exist probabilistically". I suppose particles in quantum mechanics can be said to exist probabilistically, so the case could be made. risk (talk) 16:29, 11 May 2009 (UTC)[reply]
Acording to the linguist Steve Pinker there's nothing wrong with saying "Hopefully, xyz". I agree with him. Dauto (talk) 17:43, 11 May 2009 (UTC)[reply]
I think he is at odds with Fowler's_Modern_English_Usage which has to be the higher authority. As you say he is only a linguist not a grammarian, although his book the language instinct is in my top ten books of all time.--BozMo talk 18:19, 11 May 2009 (UTC)[reply]
There's an article, needless to say. --Sean 20:12, 11 May 2009 (UTC)[reply]
However the article seems to be entirely about US usage whereas we were discussing a London bus I thought. Misuse being widespread in the US isn't really relevant or unusual. It is however very difficult to care about it. --BozMo talk 21:03, 11 May 2009 (UTC)[reply]

The use of 'probably' is not in the mathematical sense, it is in the everyday sense of meaning 'most likely'. They're not saying they've done some stats and the probability says X, rather they are saying it is 'most likely' that god doesn't exist. Their reasoning will be based on a huge multitude of factors, not least a lot of what modern science has uncovered about the beginnings of the universe. Anyhoo the point is - this is more likely a advertiser looking for a 'catchy' comment than a statement of fact/reasoning. ny156uk (talk) 16:39, 11 May 2009 (UTC)[reply]

The bus ad uses a rhetorical device that can be called "amplification of slight doubt" that has a venerable tradition in an English operetta:
CAPTAIN: I am never known to quail
At the fury of a gale,
And I'm never, never sick at sea!
CHORUS: What, never?
CAPTAIN: No, never!
CHORUS: What, never?
CAPTAIN:"Hardly ever! Cuddlyable3 (talk) 17:06, 11 May 2009 (UTC)[reply]

The inclusion of "probably" was, I believe, required to allow the advertisement to meet the requirements of the ASA that absolute claims made must be verifiable. To omit it would require proof that God does not exist. Bazza (talk) 13:18, 12 May 2009 (UTC)[reply]

Actually in a radio interview the instigator of the ads said the word "probably" was included to be inclusionist about the atheists supporting it. Many were not "fundamentalist atheists", who contend that it is utterly inconceivable that God exists, but were more moderate atheists who had concluded that the existence of God was unlikely. (If you think about the other unsupported claims that are allowed in adverts then the ASA theory isn't likely). DJ Clayworth (talk) 17:22, 12 May 2009 (UTC)[reply]
It is worth noting that a complaint was made to the ASA and rejected. I don't know if the presence of the word "probably" was significant in that decision, but I doubt it. --Tango (talk) 17:48, 12 May 2009 (UTC)[reply]

Does half the voltage mean half the power?

Does the same lightbulb in a country with a 110 V standard burn half as bright as in a country with 220 V? And does the same go for a battery charger? Will it take twice as long to charge batteries with 110 V instead of 220 V? DirkvdM (talk) 19:03, 11 May 2009 (UTC)[reply]

Actually, if you assume that the lightbulb's resitence is Ohmic The power drawn at 110V should be four times smaller from the formula . In reality the bulb's resistence dependes quite a bit on its temperature which influences the result above. If the battery charger is designed for both 110V and 220V which is quite common nowadays, there should be no change on the time taken to fully charge the battery. Dauto (talk) 19:13, 11 May 2009 (UTC)[reply]

That power formula is for direct current. I believe all countries use alternating current. The power is based on voltage and current. Without knowing the current, it is impossible to make a judgement about power. See AC power. -- kainaw 19:16, 11 May 2009 (UTC)[reply]
The formula I presented works for both direct current and alternating current, provided that the load is an ohmic resistor. As I pointed out, that's not the case here. Dauto (talk) 21:26, 11 May 2009 (UTC)[reply]
(edit conflict with both above) A simple resistive load will draw 1/4 the power when connected to 110 V, as compared to 220 V; see Electric power. But neither of the devices you mention are simple resistive loads. The resistance of a light bulb's filament depends on its temperature, which (circularly) depends on the power it's radiating. See Lamp rerating for some guidelines. A battery charger like this one is a complicated microprocessor-based device, which is in no way a pure resistive load. The one I linked to is designed to accept either input voltage, and its power supply likely uses approximately the same power in either case (drawing twice the current at 110 V, compared to 220 V). It would be designed to apply the same charging current to the batteries in either case, and hence take the same time to charge them. -- Coneslayer (talk) 19:17, 11 May 2009 (UTC)[reply]
A light bulb filament, whether tungsten or carbon, is not an ohmic resistance. When the temperature changes, the resistance changes, quite dramatically. The source [4] indicates that a tungsten filament at half the rated voltage passes just under 2/3 the current it passed at full voltage. The resistance from the chart would be about 2000 ohms at full voltage and about 1530 ohms at half voltage, for the particular bulb referenced. (Milage may vary). Edison (talk) 02:54, 12 May 2009 (UTC)[reply]

What are the benefits of a tree structure?

The Transhumanist    20:22, 11 May 2009 (UTC)[reply]

A tree diagram can show more levels of hierarchy than other types of chart. NeonMerlin 21:04, 11 May 2009 (UTC)[reply]
In computer software, it seems that a vast number of types of data are best stored in a tree-like structure (or one of it's generalizations or specializations). Why this is, is hard to say - but it pops up all over the place. Take, for example, a phone book. A long list of names with associated numbers. Real phone books are essentially just linear arrays - but when you store a phone book on the computer, it turns out to be handy to use a tree structure. The 'root' of the tree might have 26 branches - one for each letter of the alphabet. Each one of those has 26 branches - and so on for perhaps 100 levels corresponding to the longest name + address in the phone book. At the leaves of the tree, we place the phone number that corresponds to the name you spell out as you 'walk' along the tree to get to that leaf. Obviously, the leaf that you get to by following XQWZXGKWRTQ doesn't exist - so we 'prune' the tree to keep only the branches that actually lead somewhere. You'd obviously want branches for numbers and space characters and stuff - but the idea is basically there. Now, when you ask the computer to look up a phone number, it just follows the branches letter by letter and in just a couple of dozen choices, it finds the phone number. Compare that to the number of names you have to look at to find an entry in a paper phone book! Furthermore, when someone new has to be added to the phone book, it's easy to create the extra branches and tack on another leaf. Doing that with a simple linear array would require that you shuffle a large amount of data down the array to make a space for the new name. Furthermore, while you're doing that, the computer can't look up any more phone numbers...but with a tree, one part of the program can be adding or deleting entries while another part is looking up phone numbers. Tree structures are VERY cool. SteveBaker (talk) 02:13, 12 May 2009 (UTC)[reply]

Mutants leaving home to adapt

In what species, if any, have mutants been known to leave their home environments for places to which they were better adapted? NeonMerlin 21:03, 11 May 2009 (UTC)[reply]

Er, humans? David Vetter, for example. Rockpocket 21:31, 11 May 2009 (UTC)[reply]
Actually, Vetter was born in that environment, so perhaps Ted DeVita would be a better example. But there are lots of less dramatic examples of this: people with albinism moving to countries with less sunlight or people with arthritis moving to countries with a warmer climate. Rockpocket 21:41, 11 May 2009 (UTC)[reply]
Zoonoses must have done so at one point. Some species are rather polymorphic and thus may utilize different habitat niches. It's more likely that the "conquest" of a new habitat by a population subgroup with an adaptive trait takes place gradually; it would be hard to pass on your genes if you had no one to mate with in your new home. Also it may take some generations of selection for the organism to refine its adaptations. I'll try to find a specific non-human vertebrate example. In the meantime you may want to see Biological dispersal and rafting event. Sifaka talk 22:53, 11 May 2009 (UTC)[reply]
More a hybrid than a "mutant", but certainly different from your average polar bear: Grizzly–polar bear hybrids might have different habitat preferences than their parent species, but there is no information due to them being very rare. Sifaka talk 23:10, 11 May 2009 (UTC)[reply]
One of his descendants looks like a good bet [5]. Can't give you a species name, though. Since lots of scientists think that the first lifeforms were anaerobic the first bacteria taking steps toward becoming aerobic bacteria would probably also qualify. 71.236.24.129 (talk) 03:41, 12 May 2009 (UTC)[reply]

May 12

cold virus and cold water

we all know that common cold is caused by a virus. Then why do people say that drinking chilled water that is purified earlier will cause cold?--harish (talk) 01:18, 12 May 2009 (UTC)[reply]

  1. We don't all know this.
  2. We don't always act rationally on what we know
Old wives' tales and folk remedies (or folk ailments) are unlikely to be spontaneously extinguished in a fit of rationality. — Lomn 01:35, 12 May 2009 (UTC)[reply]
There is a quality of free association that one finds with pre-bacteriological understandings of disease. "Hmm, mild sicknesses appear to increase in the winter -> must be because of the cold -> other things that are cold must cause illness." --98.217.14.211 (talk) 01:53, 12 May 2009 (UTC)[reply]

using computer and eye problems

The computer monitor is just like any other object in the environment. If we reduce the monitor brightness and use an anti-glare screen, the monitor will have the same brightness as other objects. We don't get eye problems if we gaze at a table for long hours. But if we do the same for a computer, why do we get eye problems?--harish (talk) 01:22, 12 May 2009 (UTC)[reply]

Gazing at anything for long hours may inhibit blinking which may lead to dry eyes and all the common related problems. What specific "eye problems" are you referring to? -- kainaw 01:38, 12 May 2009 (UTC)[reply]
You can get eye strain reading a book for too many hours in a row too. Holding something in front of your face and staring at it intimately for hours and hours is generally not great on the eyes. The problem with computers is that the time flies by pretty dang fast. I don't think it is a problem with brightness per se. --98.217.14.211 (talk) 02:57, 12 May 2009 (UTC)[reply]

By eye problems, i mean myopia or something like that. People warn that using the computer for long hours will result in wearing spectacles--harish (talk) 05:00, 12 May 2009 (UTC)[reply]

People say all sorts of things. Perhaps you should ask them for the evidence.--86.25.193.179 (talk) 05:51, 12 May 2009 (UTC)[reply]
And yet many people who do not use the computer for long hours also end up having to get spectacles... while many people who use computers every day don't. I don't think it's as simple as "don't look at X too much". -- Captain Disdain (talk) 08:26, 12 May 2009 (UTC)[reply]
You can definitely get eye-strain from staring at a monitor for too long - but that's a temporary condition, the muscles are simply getting tired from holding the same (short) focal length for hours at a time. You rest - they get better. Taking some time out every 10 minutes or so to stare out the window - or somewhere else as far away as possible - definitely helps. But I've been using computer screens in my job and at home ever since there WERE computer screens - and although I'm a little short-sighted, I can still pass the eyesight test for my driver's license without glasses. I don't think we have solid scientific evidence on this, either way, but it's certainly not as black and white as some people make out. SteveBaker (talk) 12:02, 12 May 2009 (UTC)[reply]
(anecdotal evidence) I've been using computers since I can remember (2-3 years old) and for LONG hours a day. That includes long hours with those old monitors with very low refresh rates, high radiation and so. Also, I've been reading compulsively both in the computer screen and in paper with poor illumination. Needless to say, I've been repeatedly told that I would damage my eyesight. Now I'm 23 and my eyesight is perfect. Even glasses with very slight correction are incredibly uncomfortable for me. Only when I was very young I remember that sometimes my eyes itched after playing with my old MSX for too long, but, for some reason, it happened like 3-4 times and never happened again. --Taraborn (talk) 13:33, 12 May 2009 (UTC)[reply]

Temperature of the Exosphere

A line in the recent Scientific American (sorry, I know, I'm just bored, and the website is so flashy...) states <a href="http://www.scientificamerican.com/article.cfm?id=how-planets-lose-their-atmospheres&page=2">thus</a>:

The temperature at Earth’s exobase oscillates but is typically about 1,000 kelvins, implying that hydrogen atoms have an average speed of five kilometers per second. That is less than Earth’s escape velocity at that altitude, 10.8 kilometers per second, but the average conceals a wide range, so some hydrogen atoms still manage to break free of our planet’s gravity.

1,000K at the exobase? Isn't that a little bit high? Is this a typo or am I missing something? 219.102.221.127 (talk) 03:40, 12 May 2009 (UTC)[reply]

May be this previous discussion gives some hint about the temperature issue. manya (talk) 04:26, 12 May 2009 (UTC)[reply]
See thermosphere. Looie496 (talk) 04:29, 12 May 2009 (UTC)[reply]

Ah, I see thank you. Darn, I missed it by one sphere. Maybe it should be added to the exosphere article that the temperature in parts can be extremely high, or at least that the fast moving particles of the exosphere have very high energies. 219.102.221.127 (talk) 05:26, 12 May 2009 (UTC)[reply]

dogs and grass

Why do dogs eat grass? Mine seems especially hungry for it in the spring. --Halcatalyst (talk) 04:01, 12 May 2009 (UTC)[reply]

Let me google that for you. :) manya (talk) 04:19, 12 May 2009 (UTC)[reply]
Many dogs are idiots. Tempshill (talk) 04:38, 12 May 2009 (UTC)[reply]
Many animals eat stuff not naturally considered part of their diet. Parrots eat clay, chicken eat pebbles, cats eat grass, birds eat various herbs. Some of these things have been found to help digestion or serve medical purposes. 71.236.24.129 (talk) 08:21, 12 May 2009 (UTC)[reply]

Dogs, if they were still running in packs and actively hunting like wolves, would normally get fiber and veg proteins etc. by eating the intestinal tract of thier prey (partially digested grass and other stuff). I would suspect they eat grass to supplement this now missing fiber/vitamin source.67.193.179.241 (talk) 12:16, 12 May 2009 (UTC) Rana sylvatica[reply]

My vet said some dogs eat grass if they have indigestion; or Grassiscon. Or they may just like the taste of it. Lanfear's Bane | t 12:44, 12 May 2009 (UTC)[reply]

Is the speed of everyones brain similar?

In the same way that computer CPUs can differ in speed, our brains could be thought of as having a clock speed of some kind. Clearly there is some speed, as in practical experience thinking does take time, it is not instantaneous. My question is - is this speed constant among humans - among westerners for example? Or do some people have brain-speeds that are say a half or a tenth of the average? Supplementary question - are there medical conditions or recreational drugs that can significantly slow down brain-speed? 78.145.21.210 (talk) 12:59, 12 May 2009 (UTC)[reply]

from the article myelin "The main purpose of a myelin layer (or sheath) is an increase in the speed at which impulses propagate along the myelinated fiber." It seems that the quality of the myelin varies see: http://www.medicexchange.com/Neurology-Brain-CNS/brain-images-reveal-the-secret-to-higher-iq.html --Digrpat (talk) 13:44, 12 May 2009 (UTC)[reply]
That Myelin article is I guess the closest you'll get to a hardware answer. My own experience is that I seem to have two speeds - very slow for thinking where someone says something and I don't have an answer ready and they're already on to the next thing before I open my mouth, and very fast where I can easily do things like catch a piece of paper dropped between my fingers. Yep I envy those people with a ready wit and repartee. Dmcq (talk) 15:19, 12 May 2009 (UTC)[reply]
I'm not sure clock speed is a good analogy. Whether or not one's brain chemically works at the same speed as others doesn't necessarily translate into speed of thought (associations, combinations of ideas, memory, mathematical operations—all of these are likely quite different neurologically). There are obviously some individuals who are really on the end of the bell curve in terms of some of these functions (John Von Neumann was reportedly a complete mathematical savant). I'm not sure there's any great way to measure the "speed" of the brain in general though. Keep in mind that even if you do measure the speed of a thought, it is not the same thing as the speed of your comprehension of the thought (which is another question altogether, neurological studies have shown—your brain knows things before "you" do). --98.217.14.211 (talk) 15:20, 12 May 2009 (UTC)[reply]
No. I am very slow witted. Some people are remarkably quick-witted, such as Paul Merton.--86.25.193.179 (talk) 16:14, 12 May 2009 (UTC)[reply]
As long as the brain is at normal temperature, the speed of the cellular components is fixed and the same for all people (it only takes a little bit of hypothermia to slow things down, though). But there is a concept of "processing speed" that operates on a higher level than the cellular, and possibly relates to intelligence. It isn't all that well-defined a concept, but you might look at this review for more information. There is a substantial amount of literature on the topic. Looie496 (talk) 16:15, 12 May 2009 (UTC)[reply]
Actually, clock speed might be a very good analogy. People often make the mistake of assuming the speed of a CPU is determined entirely by its clock speed, which is not the case, and the same mistake is being made here. The time it takes to solve a problem depends both on how many things you can do in a second and how many things you need to do. In terms of CPUs, a CPU can do one instruction for every tick of the clock, but different CPUs can do different amounts with each instruction. Some will have a special instruction to solve a given problem in one step while another would have to use general functions and do it in several steps. The same could be true of the brain - one person may be able to solve a problem quickly because they have a good method to do so while another person would take longer because their method is slower. Another factor in both brains and CPUs is memory, both the amount of working memory available and the amount of data already stored. A small child when asked to calculate "ten minus four" may take several seconds because they have to actually calculated it whereas I would answer in a fraction of a second because I have the answer stored in my memory having used it so many times before. The same happens with computers - really fast chess solvers often work by having the solutions to certain positions calculated and stored in advance (particular for the endgame when there are fewer possible positions). --Tango (talk) 18:09, 12 May 2009 (UTC)[reply]
There are people who speak very slowly. Are they slow witted? To some extent, this is a cultural thing. It may also be due to individuals copying others while growing up. Rural people, and people from the Ameriocan South supposedly speak slowly, though I have not noticed it. Some women speak very rapidly. I have heard Italian women rattle off Italian at amazing speed. - GlowWorm. —Preceding unsigned comment added by 98.21.108.29 (talk) 23:18, 12 May 2009 (UTC)[reply]

units

please help me with this question: if the units of force, energy and velocity are 10N, 100J and 5 m/s, find the units of length, mass and time.

(i just don't understand this, the unit of force is Newton. so what's 10N doing here?) please give me hints on how to solve this question —Preceding unsigned comment added by 122.50.131.1 (talk) 13:41, 12 May 2009 (UTC)[reply]

Is it that the question is asking you to suppose that something with that force and energy is travelling at that velocity. On that basis, it's asking you to work out how far it's travelled, what its mass is and how long it's taken to travel there. I have my reservations about this, because my science knowledge is sparse (and a more knowledgable contributor should be along soon) but I think you'd need one of "length" and "time" to work out the other. So perhaps I've misunderstood "length". It can't mean the length of the object, because unless there's more to the question than you've told us, it could be any length from just under infinitely long to just over zero length. --Dweller (talk) 13:57, 12 May 2009 (UTC)[reply]
It is a very confusing question, but here is my guess as to what it means. In SI (as well as in other systems of measurement), units of force, energy and velocity are derived units, defined in terms of the base units of length, mass and time, which are the metre, the kilogram and the second. But you could reverse this logic and make force, energy and velocity your base units, and then define length, mass and time in terms of them - for example, unit of length = unit of energy / unit of force (not a very practical approach, but possible in theory). So I think the question is asking this : in a system of measurement in which the base units are force, energy and velocity, and have magnitudes of 10N, 100J and 5 m/s, what are the magnitudes of the derived units of length, mass and time ? Gandalf61 (talk) 14:16, 12 May 2009 (UTC)[reply]
It's a horribly written question, but that interpretation seems to work. Allow me to explain it slightly differently in case people didn't get that. We define new units, Foo, Bar and Baz. Foo is a unit of force an is equal to 10N, Bar is a unit of energy and is equal to 100J and Baz is a unit of speed and is equal to 5m/s. If order to have a consistent system of units (so all the formulae we know still apply, for example F=ma, without needing to add in constants) we need to define new units of length, mass and time with particular conversions to SI units so that everything works out nicely. What should those new units be? --Tango (talk) 14:29, 12 May 2009 (UTC)[reply]
To me it sounds like a perfectly fair question. It does not require Energy, Force and Velocity to be the base units. All it requires is that their values turn out to be the ones described in the problem. Length, Time and Mass may still be taken to be the base units. I like that problem. Dauto (talk) 15:44, 12 May 2009 (UTC)[reply]
Yeah, it's a perfectly good question, just badly written. It should have said something like "If the units of force, energy and velocity in a new system of units are equal to 10N, 100J and 5 m/s, what are the units of length, mass and time equal to in terms of the appropriate SI units?". (That's assuming we have correctly interpreted the question, which is far from certain.) --Tango (talk) 18:01, 12 May 2009 (UTC)[reply]

molecules and electrons

how can i find out the no. of electrons in 6.022 * 1022 molecules of methane?? —Preceding unsigned comment added by 122.50.131.1 (talk) 15:01, 12 May 2009 (UTC)[reply]

First find the number of electrons in one molecule of methane and than multiply that number by 6.022 * 1022 :) Dauto (talk) 15:51, 12 May 2009 (UTC)[reply]
Then check that you really mean 6.022 * 1022, and not 6.022 * 1023. Algebraist 16:49, 12 May 2009 (UTC)[reply]

What's That Bug, episode 271,828,183

This thing is about 1.5 inches long, lives happily underwater (swims well, etc.), seems pretty happy out of the water, doesn't bite (me). I found it in a small stream in central North Carolina, USA. Anyone know what it is? Thanks. --Sean 15:20, 12 May 2009 (UTC)[reply]

File:BagginsBug1.jpg
What am I?
File:BagginsBug2.jpg
The underneath
Looks like a dragonfly nymph, but no idea what species. Mikenorton (talk) 15:58, 12 May 2009 (UTC)[reply]
Yeah, that's what I was about to say. Looie496 (talk) 15:59, 12 May 2009 (UTC)[reply]
Wow, no kidding; thanks so much. I just got schooled by a 5-year-old. :) --Sean 18:24, 12 May 2009 (UTC)[reply]
It looks like the nymph of some kind of darner. This photo (no species specified), looks pretty close. There are a bunch of other pictures of darner adults and nymphs by the same photographer here, but the nymphs of the different species look rather similar. Deor (talk) 20:40, 12 May 2009 (UTC)[reply]


electrons and molecules

how can i find out the number of electrons in one molecule of methane? —Preceding unsigned comment added by 122.50.131.1 (talk) 16:11, 12 May 2009 (UTC)[reply]

You could start by looking at our article on methane. 65.121.141.34 (talk) 16:21, 12 May 2009 (UTC)[reply]
You'll get close if you start by writing down the names of the atoms, then next to each atom write the number of electrons that atom normally carries. Tempshill (talk) 18:00, 12 May 2009 (UTC)[reply]
Remember to then account for the overall charge on the molecule. (Methane is usually neutral, but if you wanted to do a similar analysis for the sulfate ion, net charge becomes important.) -- 128.104.112.117 (talk) 23:21, 12 May 2009 (UTC)[reply]

Removing COSTAR from Hubble

I don't really understand exactly how COSTAR can no longer be needed. So to begin with, I understand that Hubble had an aberration that required corrective optics in the form of COSTAR. Gradually old instruments have been replaced by ones that correct for the aberration themselves. How have the new instruments bypassed COSTAR? Is COSTAR something that can be turned on and off? Many thanks. RupertMillard (Talk) 18:02, 12 May 2009 (UTC)[reply]

To quote an official-looking page, "All the other instruments, installed since HST's initial deployment, were designed with their own corrective optics. When the FOC is replaced by the Advanced Camera for Surveys (ACS) during Servicing Mission 3B, COSTAR will no longer be needed." So appears that COSTAR has already been bypassed, and the last instrument needing correction is being replaced with one that does not need it.-RunningOnBrains 20:52, 12 May 2009 (UTC)[reply]
Yeah, but this is what I don't get — you stick something in a telescope to fix it. How can some instruments rely on it while others don't? RupertMillard (Talk) 21:09, 12 May 2009 (UTC)[reply]

egg yolks

Are there egg yolks from some bird species that are not normally yellow as per chicken eggs? Examples? 65.121.141.34 (talk) 20:17, 12 May 2009 (UTC)[reply]

This BBC article (http://www.open2.net/everwonderedfood/eggsecrets.html) seems to suggests that 'crimson' yolks are said to occur if a hen is fed coloured maggots. Not quite what you mean but mostly on-topic. ny156uk (talk) 21:02, 12 May 2009 (UTC)[reply]

Common liquids

I realised recently (like...2003, but meh) that there are very few household liquids that I know of; nearly everything is a suspension or solution in water. I can only think of:

  • Water
  • Oil (including petrol/diesel)
  • Mercury

Are there many others that I'm simply ignorant of? 90.193.232.41 (talk) 20:50, 12 May 2009 (UTC)[reply]

That's an interesting observation! Another issue is that most things we use aren't "the pure chemical" but rather are dilute solutions or mixtures even if the material itself might be a liquid. DMacks (talk) 20:56, 12 May 2009 (UTC)[reply]
Mmhm. I noticed it when I realised milk was just water with suspended fat droplets; gravy was the same (water with something); so is pretty much everything I drink. What's blood? 90.193.232.41 (talk) 20:58, 12 May 2009 (UTC)[reply]
Blood is an aqueous solution with lots of cells and other things suspended in it. DMacks (talk) 20:59, 12 May 2009 (UTC)[reply]
At room temperature, there are e.g.:
  • Vinegar (Acetic acid)
  • Alcohol (Ethanol, Methanol, Propanol,...)
  • Carbon tetrachloride
--Stephan Schulz (talk) 21:01, 12 May 2009 (UTC)[reply]

(several ecs)One reason I can think of off the top of my head as to why this is true: if you look at the phase diagram for water, you will see that the green liquid area is the smallest region on the graph; most substances are liquid only for a certain, relatively small range of temperature and pressure. Also, you have forgotten some that are probably in your house right now:

Depending on what you mean by "household", there is also Hydrogen peroxide and Sulfuric acid. There are also many substances in your daily life which are kept as a liquid under pressure (even though they would be a gas if you released them from their container), such as 1,1,1,2-Tetrafluoroethane (probably cooling your refrigerator) and propane. -RunningOnBrains 21:22, 12 May 2009 (UTC)[reply]

Careful...the alcohols (especially drinking) in the home are almost certainly water solutions of these liquid chemicals. Likewise for peroxide and acetic acid. Pure sulfuric acid isn't even available at all. Cooking oil is a good household liquid (and important distinction vs the hydrocarbon oils and fuels). And from it we get biodiesel and similar liquids that are also different from hydrocarbon liquid fuels. OTOH, you'll want to read the glass#Physics of glass article to see what glass probably doesn't belong on a list of liquids. DMacks (talk) 21:34, 12 May 2009 (UTC)[reply]
I know glass isn't a liquid. I don't have "rubbing alcohol" (whatever that is) in my house. Aren't acids basically just a solution in water? =\ I guess I misunderstood chemistry in school...
Though they could be prepared as pure substances, both the acids and alcohols will almost certainly be solutions in water for any likely household use. Dragons flight (talk) 22:14, 12 May 2009 (UTC)[reply]

A lighter may include a butane or naptha liquid. Dragons flight (talk) 22:20, 12 May 2009 (UTC)[reply]

Paint thinner / turpentine are other liquids in some households. Similarly, paints before application are usually liquid. Some paints are oil based, some are water based, and some are neither. Dragons flight (talk) 22:28, 12 May 2009 (UTC)[reply]

magnetic rotation

Can one magnet using the repelling force of another magnet to create circular momentum of one of the magnet —Preceding unsigned comment added by 121.216.203.26 (talk) 21:30, 12 May 2009 (UTC)[reply]

It doesn't matter if it is repelling or attracting. Making one magnet move by use of another magnet is very common. The most visible example that comes to mind is dropping a magnetic cylinder in a liquid and placing it on a spinning magnet. The cylinder will spin also - mixing up the liquid. -- kainaw 22:35, 12 May 2009 (UTC)[reply]
The most common example of magnets attracting each other occurs in electric motors. —Preceding unsigned comment added by 98.21.108.29 (talk) 22:47, 12 May 2009 (UTC)[reply]

I need help understanding this task. I know that the charges on the periodic table go +1, +2, +3, +/-4, -3, -2, -1 (0), where 0 and +/-4 do not ionize (I think). But, what's the deal with the +e-? Honestly, I don't really know why some elements lose and gain electrons and some need that e-. Thank you. Sorry if this is a bit laborious. Thx! —Mr. E. Sánchez (that's me!)What I Do / What I Say 22:53, 12 May 2009 (UTC)[reply]

To a first approximation, atoms want their electron shells to look like noble gases (the octet rule), and they don't care which one. So they take the one that's easiest to get to. That is, the one where they have to gain/lose the least number of electrons, because each electron you gain/lose increases the overall charge, making it that much more energetically costly to gain/lose another. That's why the atoms on the left side of the periodic table tend to lose electrons, and the ones on the right gain electrons. It's the shortest, most energy-efficient path to be noble-gas like. (The whole analysis gets a little complicated in the transition metals, where there are electrons in the d-Orbitals which "don't count", but the general principle of being "noble gas like" still roughly holds.) -- 128.104.112.117 (talk) 23:17, 12 May 2009 (UTC)[reply]